Sie sind auf Seite 1von 50

6ta Edicin

2017

ENAE
Examen Nacional de Enfermera
COMPENDIO DE
BANCOS DE PREGUNTAS

Pedidos: Cel. :RPC. 969340751


Cel. :RPC. 941105128
Correo: publicacionesdc@hotmail.com
Mg. Dany Colca
EXAMEN NACIONAL DE ENFERMERA. Compendio Banco de Preguntas. Lic. Carolina Ayala

1. Hombre de 48 aos que acude a Urgencias por dolor abdominal y


vmitos. Los datos analticos iniciales son: GOT: 80 U/I. GTP 54 U/I.
Leucocitos 21.800/mm3. Amilasa 4.500 U/I. La TAC abdominal
demuestra coleccin peripancretica. Se inicia tratamiento con
fluidoterapia y analgsicos. Seale cul de los siguientes
frmacos aadira al tratamiento, en primer lugar, para mejorar el
pronstico de este caso:
A) Metilprednisolona.
B) Antiinflamatorios no esteroideos. C) D)
Imipenem.
E) Inhibidores de la secrecin pancretica.

2. Cul es el principal factor responsable de la no


cicatrizacin de una lcera pptica?:
A) Estrs.
B) Infeccin por Helicobacter pylori.
C) Consumo de alcohol.
D) No abandono del hbito tabquico. E)
Determinados hbitos dietticos.

3. Cul de las siguientes recomendaciones es FALSA en relacin


con el tratamiento de la peritonitis bacteriana espontnea en un
paciente cirrtico?:
A) El diagnstico se basa en la existencia de ms de
250 PMN/mm3 en el lquido asctico.
B) El tratamiento de eleccin lo constituyen las
cefalosporinas de tercera generacin.
C) La administracin de albmina intravenosa previene el
desarrollo de insuficiencia renal.
D) El tratamiento antibitico debe mantenerse durante
15 das.
E) Una vez resuelta la infeccin, debe iniciarse tratamiento
profilctico con norfloxacino.

4. Paciente de 72 aos, que como nico tratamiento toma


antidiabticos orales, presenta anemia ferropnica crnica, con
hemorragias ocultas positivas. Cual es el mtodo diagnstico ms
indicado para localizar la lesin sangrante?:
A) Trnsito gastroduodenal.
B) Gammagrafa con hemates marcados. C)
Trnsito intestinal.
D) Colonoscopia total. E)

BANCO
Panendoscopia oral.

5. Cul de las siguientes definiciones corresponde con mayor


N001 precisin al concepto de Metaanlisis?:
A) Es una revisin narrativa en la que la bibliografa se busca de
manera no estructurada.
B) Es una revisin narrativa en la que la bibliografa se busca de
manera estructurada.
C) Es una revisin narrativa en la que se presentan tablas
estructuradas de los resultados de los distintos estudios incluidos.
D) Es una revisin en la que se combinan
estadsticamente los resultados de los estudios incluidos.

[1]
Mg. Dany Colca
RESIDENTADO DE ENFERMERA. Compendio Banco de Preguntas. Lic. Carolina Ayala

E) Es una revisin en las que se presentan tablas estructuradas de C) Fstula crnica de lquido cefalorraqudeo tras un
los resultados de los distintos estudios incluidos. traumatismo craneal.
D) Infeccin VIH avanzada.
6. Le encargan el diseo de un ensayo clnico en el que es muy E) Insuficiencia cardaca crnica.
importante que un factor pronstico se distribuya por igual en los dos
grupos de tratamiento. El mtodo de aleatorizacin que usted utilizara 12. Cul de la siguientes vacunas no se incluye en las
es: recomendaciones actuales de un paciente esplenectomizado?:
A) Aleatorizacin simple. A) Vacuna neumoccica. B)
B) Aleatorizacin por bloques. C) Vacuna meningoccica.
Aleatorizacin estratificada. D) C) Vacuna frente a Haemophilus influenzae tipo B.
Aleatorizacin centralizada. D) Vacuna frente al virus de la gripe. E)
E) Aleatorizacin ciega (ocultacin de la secuencia de Vacuna frente al virus hepatitis A.
aleatorizacin).
13. En qu patologa pensara en primer lugar en un paciente de 65
7. En una revista biomdica se publica un estudio, en el que los aos, que presenta disminucin lenta, progresiva e indolora de su
autores notifican el resultado en coste/aos de vida ganados. De qu agudeza visual sin signos de inflamacin ocular?:
tipo de anlisis de evaluacin econmica se trata?: A) Error de refraccin. B)
A) Coste de la enfermedad. B) Distrofia corneal.
Coste-efectividad. C) Papilitis.
C) Coste-beneficio. D) Glaucoma agudo.
D) Coste-consecuencia. E) E) Degeneracin macular senil.
Coste-utilidad.
14. Un paciente diabtico tratado mediante fotocoagulacin focal con
8. La mejor forma de verificar una hiptesis en lser de Argn tres aos antes, presenta una prdida brusca e
epidemiologa es a travs de: importante de visin, sin dolor ni alteraciones en la superficie ocular.
A) Un estudio descriptivo. La causa ms probable de esta disminucin de agudeza visual es:
B) Un estudio experimental. C) Un A) Hemorragia vtrea. B)
estudio caso-control. D) Un Edema corneal.
estudio de cohortes. E) Un C) Glaucoma crnico simple. D)
estudio transversal. Catarata nuclear.
E) Atrofia ptica.
9. Todas estas situaciones, EXCEPTO una, aumentan la probabilidad
de detectar un cncer colorrectal, selela: A) Enfermedad 15. Una de los siguientes signos o sntomas, NO es habitual en
inflamatoria intestinal de larga evolucin. B) Endocarditis por una uvetis anterior aguda:
Streptococcus bovis. A) Midriasis. B)
C) Tabaquismo de ms de 35 aos de duracin. Dolor.
D) Ureterosigmoidostoma hace 20 aos, para corregir una C) Inyeccin ciliar.
malformacin vesical. D) Sinequias posteriores. E)
E) Ingesta crnica de aspirina o antiinflamatorios no Fotofobia.
esteroideos.
16. Cul es el tratamiento de la obstruccin intestinal no
10. Cul de las siguientes caractersticas NO es propia de un quirrgica en la fase terminal del cncer?:
ensayo clnico controlado?: A) Sonda nasogstrica, aspiracin continua,
A) Intervencin experimental. sueroterapia intravenosa.
B) Tratamiento asignado segn el criterio del B) Administracin de vitamina grupo B, laxante y enema de limpieza.
investigador. C) Alimentacin parenteral total y esteroides i.v.
C) Participacin voluntaria del sujeto. D) Morfina, buscapina y haloperidol por va subcutnea. E) Dieta
D) Asignacin aleatoria a las distintas posibilidades de tratamiento absoluta y sueros por va subcutnea.
en estudio.
E) Enmascaramiento de la medicacin. 17. Cul de los siguientes opioides NO es adecuado para el
tratamiento del dolor crnico de etiologa cancerosa?:
11. La vacuna antineumoccica est recomendada para todas, A) Metadona. B)
MENOS una de las siguientes situaciones clnicas, selela: Meperidina. C)
A) Alcoholismo crnico. Morfina.
B) Contactos familiares de un paciente con neumona
neumoccica

[2]
Mg. Dany Colca
RESIDENTADO DE ENFERMERA. Compendio Banco de Preguntas. Lic. Carolina Ayala

D) Fentanilo. E) vesiculosas en glande y adenopatas inguinales bilaterales.


Tramadol. Cul es el diagnstico ms probable?:
A) Herpes genital.
18. La localizacin ideal para hacer una traqueotoma es: B) Secundarismo lutico. C)
A) La membrana crico-tiroidea. B) La Primoinfeccin VIH.
membrana mio-tiroidea. C) Primer D) Infeccin por citomegalovirus. E)
anillo traqueal. Condilomas acuminados.
D) Segundo o tercer anillo traqueal.
24. En cuanto a la neumona por Neumococo en paciente
E) Cuarto o quinto anillo traqueal. infectados por el virus de la inmunodeficiencia humana (VIH) es
19. Cul de estas afirmaciones es FALSA en relacin con la otitis FALSO que:
media secretoria?: A) Es una infeccin frecuentemente bacterimica.
A) Los nios con paladar hendido son ms susceptibles a B) Tiene mayor incidencia que en la poblacin general. C) Junto a
padecerla. Haemophilus influenzae es la causa ms comn de neumona en
B) En casos unilaterales en adultos, es obligada la inspeccin pacientes con SIDA.
del cavum. D) Se recomienda vacuna neumoccica en aquellos pacientes
C) Su perodo de mayor incidencia en climas templados, es de junio a con CD4 < 100 cel/ L.
septiembre, coincidiendo con la poca de baos. E) Esta neumona puede ser vista en pacientes con sistema
D) El 66% de los cultivos de las muestras obtenidas por inmune relativamente intacto.
miringocentesis presentan bacterias.
E) La otoscopia puede mostrar niveles lquidos o 25. La tuberculosis asociada a la infeccin por VIH se caracteriza
burbujas de aire tras el tmpano ntegro. por:
A) Presentacin subclnica de la enfermedad.
20. Una mujer de 68 aos, sin aparentes factores de riesgo B) Aparicin caracterstica en los estadios de
cardiovascular, ingresa en la Unidad Coronaria del Hospital por un inmunodepresin ms severa (>50 CD4/mm3).
cuadro agudo de cardiopata isqumica. En la analtica realizada C) Elevada frecuencia de afectacin extrapulmonar y
a su llegada se objetiva una anemia (Hb 8gr/dl) previamente no diseminada.
conocida. En este caso, la actitud ms adecuada con respecto a la D) Escaso rendimiento de los mtodos microbiolgicos de
anemia es: diagnstico.
A) Actitud expectante, ya que slo se debe transfundir la anemia E) Mala respuesta al tratamiento antituberculoso.
sintomtica.
B) Transfundir hemates. 26. NO es propio del shock txico estafiloccico: A) Fiebre
C) Transfundir sangre total. elevada.
D) Instaurar tratamiento con eritropoyetina. E) B) Lesiones cutneas. C)
Administrar hierro intravenoso. Metstasis spticas. D)
Fracaso renal.
21. Los cuerpos de Howell-Jolly, son inclusiones E) Rabdomilisis.
eritrocitarias de fragmentos nucleares y se observan en: A) Asplenia.
B) Mielofibrosis. 27. Cul de las siguientes permite diferenciar la pielonefritis
C) Leucemia linftica crnica. D) aguda de la cistitis aguda?:
Dficit de G6PD. A) Leucocituria. B)
E) Intoxicacin por plomo. Hematuria. C)
Bacteriuria.
22. Cul de los siguientes hallazgos es el ms D) Antecedentes de infeccin urinaria.
caracterstico de la leucemia mieloide crnica?: E) Fiebre de ms de 38.5C.
A) El cromosoma Filadelfia.
B) El esplenomegalia palpable. 28. Un paciente de 22 aos, que acude para evaluacin de una lesin
C) La disminucin de la fosfatasa alcalina granulocitaria. D) El genital ulcerada, presenta un VDRL positivo a ttulo de 8 diluciones
aumento del cido rico srico. con un FTA-Abs negativo. La interpretacin ms adecuada de estos
E) El reordenamiento del gen bcr/abl. resultados es:
A) Falso positivo de las pruebas no treponmicas. B) Falso
23. Joven de 23 aos, con historia de conducta sexual de riesgo, negativo de las pruebas treponmicas.
consulta por fiebre, malestar general y dolor en glande. La C) Sfilis curada (pendiente la positivizacin del FTA- Abs).
exploracin fsica muestra mltiples tatuajes, "piercings" y la D) Sfilis curada (pendiente la negativizacin del VDRL). E) Sfilis de
presencia de tres lesiones larga evolucin.

[3]
Mg. Dany Colca
RESIDENTADO DE ENFERMERA. Compendio Banco de Preguntas. Lic. Carolina Ayala

29. Seale cul de las siguientes asociaciones de helmintos y su B) Tromboembolismo pulmonar. C)


clnica caracterstica es INCORRECTA: Rotura del msculo papilar.
A) Ascaris lumbricoides - Sndrome de Leffler. B) Giardia D) Rotura de la pared libre ventricular. E)
lamblia Mala absorcin intestinal. Hemorragia cerebral.
C) Ancylostoma duodenale - Anemia megaloblstica.
D) Strongyloides stercolaris - Sndrome de 34. En el tratamiento de la hipertensin arterial, la ventaja de
hiperinfestacin en inmunodeprimidos. los bloqueadores de los receptores de la Angiotensina II con
E) Taenia solium - Convulsiones generalizadas. respecto a los inhibidores del enzima conversor de la Angiotensina
es que:
30. Paciente de 30 aos, seropositivo VIH conocido desde 5 aos A) Son ms potentes.
antes, con antecedentes de neumona por P. jivovecci, que consulta B) Producen menos tos.
por cefalea desde 10 das antes. La exploracin fsica muestra como C) No producen hiperpotasemia. D) Se
datos ms relevantes mnima rigidez de nuca y temperatura de puede dar en embarazadas.
37,5C, fondo de ojo normal, TAC: ligera atrofia cortical. La puncin E) Se pueden dar en sujetos con estenosis de la arteria renal
lumbar da salida a lquido claro con 40 clulas mononucleares, bilateral.
protenas: 90 mgrs%, glucosa:
30 mg% (glucemia: 90 mg%). Sealar, de entre las siguientes, la
causa ms probable: 35. El tratamiento ms eficaz para prevenir recurrencias en el aleteo
A) Herpesvirus tipo 8. B) o flutter auricular comn es:
Listeria. A) Buen control de la hipertensin arterial que con frecuencia
C) Criptococo. D) padecen estos enfermos.
CMV. B) Digoxina asociada a un frmaco que disminuya la conduccin en
E) VIH. el nodo A-V (anticlcicos o betabloqueantes).
C) Ablacin con catter y radiofrecuencia del istmo cavo tricspide.
31. En relacin con Streptococcus pyogenes y la faringoamigdalitis, D) Insercin de un marcapasos con capacidad
cul de las siguientes afirmaciones NO es correcta?: antitaquicardia.
A) En tratamiento de la faringoamigdalitis estreptoccica se efecta E) Amiodarona.
con una sola inyeccin i.m. de 1,200,000 UU de penicilina benzatina.
B) El tratamiento antibitico de la faringoamigdalitis estreptoccica se 36. Un paciente obnubilado con una presin arterial de
efecta con 250,000 UU/6h oral de penicilina V durante 10 das. 80/40 mmHg, un gasto cardaco de 3 l/min, una presin de
C) El tratamiento antibitico de la faringoamigdalitis estreptoccica se enclavamiento pulmonar de 14 mmHg y una presin auricular derecha
efecta con una sola inyeccin i.m. de de 14 mmHg, puede estar sufriendo cualquiera de las siguientes
1,200,000 UU de penicilina procana. situaciones patolgicas con la excepcin de una:
D) El tratamiento antibitico de la faringoamigdalitis estreptoccica se A) Taponamiento cardaco. B)
efecta con amoxicilina oral 500 mg/8h durante 10 das. Deshidratacin.
E) El tratamiento antibitico recomendado de la faringoamigdalitis C) Infarto de ventrculo derecho. D)
estreptoccica en los casos de alergia a la penicilina es un macrlido Tromboembolismo pulmonar. E)
oral durante 10 das. Constriccin pericrdica.

32. A lo largo de los ltimos 10 aos se ha producido un cambio muy 37. La campana del fonendoscopio es ms adecuada que la
importante en la etiologa de la Endocarditis Infecciosa del adulto. El membrana para:
microorganismo ms frecuente en la actualidad, es: A) Distinguir un soplo diastlico de uno sistlico.
A) Microorganismos del grupo HACEK. B) B) Or mejor el chasquido de apertura en pacientes con estenosis
Staphylococcus aureus. mitral.
C) Estafilococos coagulasa negativos. D) C) Valorar la presencia de un tercer y/o cuarto tono. D) Detectar
Bacilos gram negativos. el click mesosistlico del prolapso mitral.
E) Streptococcus viridans. E) Auscultar a los nios pequeos y bebs, en los que la membrana
no capta bien la tonalidad de los ruidos
33. Un hombre de 74 aos con un infarto agudo de miocardio es cardacos.
tratado con estreptoquinasa. Seis horas despus desarrolla un
cuadro de hipotensin arterial severa y obnubilacin. Cul de las 38. Cul de las siguientes respuestas sobre el Delirium
siguientes complicaciones es MENOS probable que sea la causa?: A) Tremens es la correcta?:
Infarto de ventrculo derecho. A) Se trata del estado peculiar de intoxicacin producido por el
consumo de alcohol.
B) Se caracteriza por la aparicin de conductas impulsivas
tras el consumo de una pequea cantidad de
alcohol.

[4]
Mg. Dany Colca
RESIDENTADO DE ENFERMERA. Compendio Banco de Preguntas. Lic. Carolina Ayala

C) Es un cuadro clnico con elevado ndice de complicaciones 43. Un paciente de 67 aos, comienza a presentar lesiones
psiquitricas y con pocas complicaciones orgnicas. eritematoedematosas en brazos y abdomen. Ocasionalmente se
D) Aparece con mayor frecuencia entre los hombres alcohlicos observan ampollas salpicando el rea afectada. No hay afectacin
a partir de los 50 aos. de mucosas. Conserva el estado general. Se realiz biopsia de piel
E) Su tratamiento de eleccin son las benzodiacepinas. donde se observ las imagen de una vescula subepidrmica con
eosinfilos. En inmunofluorescencia directa se observ una imagen
39. Cules son los trastornos perceptivos, de entre los que se de depsito lineal en UDE de IgG. Tras usar la tcnica de separacin
enumeran, ms caractersticos de las esquizofrenias?: con sal, la banda de depsito aparece tanto en el suelo, como en el
A) Ilusiones hipnaggicas. techo de la ampolla. El diagnstico ms probable ser:
B) Alucinaciones visuales zoomrficas. A) Enfermedad de During-Brocq. B)
C) Alucinaciones o pseudoalucinaciones auditivas. D) Fogo Selvagen.
Paraeidolias. C) Enfermedad IgA lineal.
E) Alucinosis auditivas. D) Epidermlisis ampollosa adquirida. E)
Penfigoide ampolloso.
40. Una paciente de 24 aos acude a la Urgencia llevada por un
familiar por vmitos recidivantes. Parece bulimia nerviosa. Cul de 44. Un paciente de 44 presenta lesiones ppulo- erosivas muy
las pruebas de laboratorio es ms til para evaluar la gravedad de los pruriginosas en glteo y rodillas, que comenzaron a salir hace meses,
vmitos?: cursando en brotes que curan espontneamente en una semana.
A) Nivel de Hemoglobina. Se ha realizado biopsia de piel observndose ampolla subepidrmica
B) Nivel plasmtico de Amilasa. C) en tincin de hematoxilina-eosina y un depsito de IgA linea en
Nivel srico de Socio. UDE. La biopsia de vellosidades intestinales en normal. Los
D) Nivel plasmtico de Calcio. anticuerpos antigliadina IgA son negativos. Los anticuerpos
E) Nivel plasmtico de Creatinina. antigliadina IgG y los anticuerpos antiendomisio son positivos.
Presenta adems anticuerpos antitiroideos. El tratamiento adecuado
41. Acude a urgencias un paciente de 37 aos con gran postracin, ser:
fiebre elevada (38,5C) e intensa cefalea. Presenta en miembros A) Corticoides orales porque es una enfermedad IgA
inferiores, tronco y raz de miembros superiores un exantema tenue lineal.
eritemato- violceo, maculoso, escasamente confluente, con B) Corticoides orales porque es una Dermatitis
afectacin de palmas y plantas. Al interrogar a familiares, Herpetiforme con biopsia intestinal negativa.
reconocen haber estado hace siete das en una excursin. C) Sulfonas ms dieta sin gluten, porque es una
Reexplorado el paciente, en cara posterior de la piera derecha Dermatitis Herpetiforme.
aparece una lesin necrtica-costrosa, de 0,5 cm. de dimetro, D) Sulfonas sin dieta, porque es una Dermtitis
rodeada por un halo violceo, edematoso de 0,3-0,4 cm. El Herpetiforme con biopsia intestinal negativa. E)
tratamiento ms indicado sera: Sulfonas ms inmunosupresores.
A) Doxiclina v.o. (200 mgr. cada 12 horas un da 100 mgr. cada
12 horas por 5 das). 45. Un varn de 6 aos presenta de manera brusca un brote de
B) Cloxacilina 1 gr. i.v. cada 6 horas durante 10 das. C) lesiones eritematoescamosas redondeadas, de pqueo tamao,
Ceftriaxona 1 gr. i.v. cada 12 horas durante 10 das. D) TMP-SMX distribudas por todo el tegumento. La semana anterior acudi a
500 mgr. i.v. cada 8 horas por 10 das. Urgencias por un cuadro catarral, con T= 37,5C y amigdalitis. En el
E) Eritromicina 250 mgr. i.v. cada 6 horas por 7 das. cultivo farngeo se aisl un estreptococo. La patologa cutnea ms
probable corresponde a:
42. Un paciente de 27 aos ex-ADVP desde hace tres aos, con A) Pitiriasis rosada. B)
muguet oral y antecedentes de neumona por neumocistis carinii, Eczema numular. C)
presenta en el dedo ndice, en superficie dorsal de la 2. falange, Psoriasis gutata. D)
una lesin nica, lcero necrtica, con crecimiento serpinginoso de Herpes circinado. E)
aproximadamente 4-5 cms. de dimetro, bordes geogrficos y Vasculitis sptica.
algunas reas costrosas, hemorrgicas. Es extremadamente
dolorosa. Su 1. sospecha clnica es: 46. Un joven de 26 aos, con antecedentes familiares de psoriasis,
A) Linfoma B cutneo. presenta un brote de lesiones en placas eritematosas, con centro
B) Ulcera secundaria al tratamiento. C) amarillento y halo perifrico descamativo, ovaladas, bien
Infeccin por herpes simple. delimitadas, en cara anterior y posterior del tronco. La 1. lesin
D) Chancro lutico. haba aparecido haca 1 semana, era de mayor tamao y se
E) Picadura sobreinfectada. localizaba en el tercio superior de la espalda. El

[5]
Mg. Dany Colca
RESIDENTADO DE ENFERMERA. Compendio Banco de Preguntas. Lic. Carolina Ayala

paciente conserv un buen estado general en todo momento. quemadura solar exagerada localizada en cara (respetando regin
Probablemente el cuadro corresponde a: peiorbitaria, retroauricular y submentoniana), dorso de manos y en
A) Psoriasis en pequeas placas. B) ambas piernas hasta la altura de las rodillas. Estaba en tratamiento
Psoriasis tipo Von Zumbusch. C) con un diurtico tiacidico.
Pitiriasis rosada de Gibert. A) Se trata de una reaccin fototxica.
D) Rosola sifiltica. B) Es una reaccin fotoalrgica por mecanismo tipo IV. C) Debe
E) Eczema diseminado. eliminarse el agente causante, poner tratamiento sintomtico
y evitar la radiacin lumnica hasta que remita el cuadro.
47. Un varn de 16 aos comienza a presentar vesculas de D) Es una erupcin polimorfa lumnica. E) Son
contenido claro agrupadas, de localizacin peribucal, que a lo largo correctas A y C.
de 1 semana se van rompiendo formndose erosiones y costras.
Transcurrido este perodo, en el dorso de las manos aparecen 2 52. Una joven de 16 aos de edad presenta varias mculas de color
lesiones ampollosas con vesculas dispuestas perifricamente y blanco lechoso de varios centmetros de dimetro, de distribucin
alguna ppula eritematosa con centro ms oscuro. Se trata de: simtrica sobre codos, rodillas, manos y zona peribucal. En la biopsia
A) Inicio de una varicela. cutnea hay ausencia de malanocitos. Tras fotoquimioterapia
B) Aparicin de lesiones en manos por contagio directo desde la sistmica con psoralenos han pigmentado parcialmente las lesiones.
zona peribucal. El diagnstico de esta paciente es:
C) Imptigo estafiloccico. A) Esclerosis tuberosa. B)
D) Eritema multiforme minor. Lepra.
E) Sndrome de Stevens-Johnson. C) Hipomelanosis guttata idioptica. D)
48. Una mujer de 65 aos viene presentando desde hace 2 aos Vitligo.
brotes de lesiones nodulares < de 2 cm., localizadas de forma bilateral E) Hipomelanosis de Ito.
en MMII, acompaados de livedo reticularis, febrcula y artralgias. La
histologa corresponde a una vasculitis leucocitoclstica. 53. Nio de 10 aos de edad que acude a la consulta por amigdalitis
Probablemente se trata de: pultcea con adenopatias cervicales. Se pone tratamiento con
A) Sndrome de Sweet. penicilina a dosis correctas. A las
B) Eritema elevatum diutinum. C) 72 horas acude de nuevo al no experimentar mejoria. Debemos
Panarteritis nodosa cutnea. D) pensar en:
Vasculitis urticarial. A) Posible Mononucleosis infecciosa.
E) Enfermedad de Kawasaki. B) Posible amigdalitis viral de etiologa diferente al
Estreptococo beta hemoltico del grupo A. C)
49. Un paciente de 63 aos de edad presenta una eritrodermia de Linfoma.
varios meses de evolucin con adenopatas generalizadas y ms D) V. Parainfluenzae. E)
de 10% de clulas con ncleo cerebriforme en sangre perifrica. Rubola.
Tiene adems intenso prurito y edemas pretibiales. Su diagnstico
sera: 54. La presencia de vmitos en la infancia es uno de los motivos ms
A) Eczema seborreico. frecuentes de consulta. Pueden ser debidos a enfermedades
B) Exantema medicamentoso crnico. C) digestivas y extradigestivas.
Eritrodermia psoriasica. Cul de las siguientes patologas produce menos
D) Sndrome de Sezary. frecuentemente vmitos en la etapa de la lactancia?
E) Parapsoriasis en grandes placas. A) Reflujo gastroesofgico.
B) Estenosis hipertrfica de ploro. C)
50. Un nio sin alteraciones cutneas al nacimiento comienza a Apendicitis.
presentar hacia los 3 meses de edad escamas grandes y negruzcas D) Invaginacin intestinal. E)
en tronco y extremidades, afectando pliegues axilares y poplteos. Gastroenteritis.
Tiene un hermano mayor igualmente afecto. Es probable que
se trate de: 55. Ante un nio que de forma brusca presenta hipotensin, vmitos
A) Una ictiosis ligada a X. B) y colapso cardiovascular que no responde a la administracin de
Una ictiosis laminar. drogas vasoactivas o catecolaminas y en la analtica realizada
C) Una ictiosis vulgar. presenta hiponatremia e hiperpotasemia, se debe considerar como
D) Enfermedad de Darier. probable diagnstico:
E) Ninguna de las anteriores. A) Insuficiencia cardaca.
B) Insuficiencia suprarrenal. C)
51. Una mujer acude a nuestra consulta poque tras haber Diabetes juvenil.
estado esa maana expuesta al sol presenta una D) Diabetes inspida.
E) Intoxicacin por monxido de carbono.

[6]
Mg. Dany Colca
RESIDENTADO DE ENFERMERA. Compendio Banco de Preguntas. Lic. Carolina Ayala

56. Un varn de 17 aos se encuentra mareado, con vmitos y en el fiebre de 38C. Cul de los siguientes cuadros cree que
transcurso de unos minutos se halla tumbado en el suelo en coma, presenta el nio?:
con una exploracin neurolgica normal. La causa ms probable sera: A) Catarro habitual descendente con evolucin a asma. B)
A) Ingestin de barbitricos. Aspiracin de cuerpo extrao.
B) Hemorragia subaracnoidea. C) C) Bronquiolitis. D)
Coma etlico. Epiglotitis.
D) Status convulsivo. E) E) Laringitis.
Tumor cerebral.
61. Un nio nacido a trmino de 2.100 gr. de peso presenta
57. Un varn de 4 aos tiene lesiones purpricas palpables, irritabilidad y temblores amplios a las 36 horas de vida. Se alimenta
simtricas, de 3 das de evolucin en las extremidades inferiores. mal y tiene diarrea y obstruccin nasal. Cul es el diagnostico ms
Los estudios hematolgicos revelan: probable?:
Hemoglobina: 10 g/dl; recuento leucocitario A) Hipocalcemia.
16.500/mm3; recuento plaquetario 240.000/mm3 y VSG B) Hipomagnesemia.
de 45 mm/hora. La etiologa ms probable es: A) C) Dficit de piridoxina.
Maltrato infantil. D) Sindrome de abstinencia por adicin materna a herona.
B) Prpura de Schonlein Henoch. C) E) Hipoglucemia.
Enfermedad de Kawasaki.
D) Meningococemia. 62. Lactante de dos meses que presenta llanto agudo, en crisis,
E) Enfermedad de Von Willebrand. desde hace 20 das, sntomas motores y heces normales para su
edad. El diagnstico ms probable ser?:
58. Durante la evaluacin previa al ingreso en el colegio de un nio A) Gastroenteritis aguda.
de 5 aos, se detecta retraso en el habla. Como antecedentes se B) Intolerancia a la lactosa. C)
recogen episodios reiterados de cuadros catarrales sin control Clico del lactante.
mdico. Cul de las siguientes es la casusa ms probabnle D) Otitis media aguda. E)
de esta situacin? Hernia inguinal.
A) Trastorno de dficit de atencin con hiperactividad. B)
Hipoacusia de conduccin. 63. Lactante de tres meses sin antecedentes previos de inters, que
C) Disfuncin de la Trompa de Eustaquio. D) en el examen de salud se le detecta un deterioro en las adquisiciones
Retraso mental. sicomotoras, el resto de la exploracin por aparatos es normal. En la
E) Hipoacusia neurosensorial. anamnesis presenta sacudidas musculares breves de cabeza y
extremidades superiores. Cul sera el diagnstico ms
59. Nia de 7 aos sin antecedentes de inters que acude a urgencias probable?:
por dolor abdominal generalizado y vmitos desde doce horas antes. A) Clico del lactante.
Deposicin normal. No antecedentes quirrgicos previos. A la B) Hemorragia cerebral. C)
exploracin presenta abdomen muy distendido y dolor en zona Sndrome de West.
periumbilical con aumento de ruidos intestinales, Blumberg (-). En la D) Sndrome de Lenaux-Gastaut. E)
Radiografa de abdomen en bipedestacin se observa obstruccin Fenilcetonuria.
a nivel de intestino delgado. El diagnstico ms probable sera:
A) Invaginacin intestinal. B) 64. Ante un lactante de 1,5 meses de edad que presenta ictericia
Brida intestinal. debemos pensar en todos los siguientes cuadros, excepto en:
C) Malrotacin intestinal. D) A) Infeccin urinaria. B)
Divertculo de Meckel. Lactancia materna.
E) Estenosis ileal congnita. C) Atresia congnita de vas biliares. D)
Ictericia fisiolgica.
60. Nio de 5 meses que llevan a la consulta por tos y secrecin nasal E) Hipotiroidismo congnito.
desde hace 24 horas. Toma mal los biberones por presentar fatiga.
Se observa febrcula y retraccion intercostal, con zonas de 65. Un paciente acude por cuadro de dolor intenso epigstrico, de
hipoventilacin y estertores en la auscultacion respiratoria. Existen aparicin brusca, acompaado de vmitos en los que slo consigue
otros familiares con cuadro catarral. Se le pone tratamiento arrojar saliva, gran distensin abdominal alta e imposibilidad para
sintomtico y se aconseja la revisin a las 24 horas. En la nueva pasar ms all del esfago distal con una sonda nasogstrica,
visita el nio ha empeorado, con importante insuficiencia respiratoria, cul sera la principal sospecha diagnstica?: A)
sibilancias, tos continua y Perforacin gstrica.
B) Estenosis pilrica aguda.
C) Vlvulo gstrico agudo.

[7]
Mg. Dany Colca
RESIDENTADO DE ENFERMERA. Compendio Banco de Preguntas. Lic. Carolina Ayala

D) Sndrome de Boerhaave. E) abdominal, nuseas y vmitos. Presenta gran distensin abdominal,


Tricobezoar. intenso dolor a la palpacin y signos de irritacin peritoneal. En la
analtica presenta marcada leucocitosis con desviacin izquierda. En
66. Un paciente de 60 aos refiere dolor en epigastrio desde hace la RX simple de abdomen se observa gran dilatacin del colon. El
unas tres semanas y en menor medida desde meses antes, que paso siguiente es:
se alivia con los alimentos y reaparece 2 horas despus de las A) Intervencin quirrgica. B)
comidas, con irradiacin a hipocondrio derecho. En la gastroscopia Rectocolonoscopia.
muestra una lcera de 1 cm. en curvadura menor, con bordes netos C) Inmunosupresores.
bien definidos, ausencia de islotes de tejido dentro del nicho D) Sonda nasogstrica, sueros y corticoides iv. E) TAC
ulceroso, con mucosa de aspecto normal. Ante lo cual Ud.: abdominal.
A) Decide no practicar biopsia de la lcera por tener caractersticas
de benignidad. 71. Una mujer de 52 aos diagnosticada de lcera gstrica de 2 cm.
B) Slo practicar biopsia del fondo del nicho. con biopsia negativa por malignidad, a las 8 semanas de tratamiento
C) Practicara biopsias mltiples independientemente de las con ranitidina 150
caractersticas del nicho ulceroso. mg cada 12 horas, se somete a control endoscpico encontrndose
D) Algunos de los signos descritos son sugestivos de malignidad, por una lcera de 0,5 cm, con nueva biopsia negativa y totalmente
lo que practicara biopsias mltiples. asintomtica. Cul es la conducta preferida?:
E) Por no tener caractersticas de malignidad claras, no practicara A) Cambiar el medicamento a fenotidina. B)
biopsias. Aadir curalfato a la ranitidina.
C) Remitir por ciruga.
67. Paciente de 84 aos que presenta cuadro de diarrea mucosa con D) Continuar con ranitidina 8 semanas ms. E)
decaimiento generalizado y prdida de peso. Analticamente el Suspender toda medicacin.
paciente presenta anemia con hipopotasemia, hiponatremia e
hipocloremia; al tacto rectal se palpa una masa homognea de 72. Una mujer de 76 aos presenta vmito en posos de caf. En la
consistencia blanda, recubierta de moco, no dolorosa. Este paciente endoscopia se encontr un plipo en cuerpo gstrico. No se identific
presentar con gran probabilidad: ningn otro origen de la hemorragia. El hematcrito es del 28%.
A) Hemorroides. Cul es la mejor opcin teraputica?:
B) Enteropata pierde protenas. C) A) Biopsia con pinzas por estudio histolgico, si es benigna
Leiomioma de recto. ninn tratamiento adicional.
D) Adenoma velloso de recto. E) B) Biopsia con pinzas, si es benigna, antagonistas H2. C)
Hamartoma rectal. Polipectoma endoscpica con asas.
D) Reseccin quirrgica.
68. Mujer de 42 aos que acude al hospital con historia de 10 aos E) Actitud expectante por si se repitiese la hemorragia.
de disfagia, primero para lquidos y posteriormente para slidos, y que
en la actualidad presenta regurgitaciones de carcter principalmente 73. Un paciente de 45 aos con anemia ferropnica y colonoscopia
nocturno. Cul sera la prueba diagnstica esencial?: normal se someti a endoscopia alta observndose un duodeno con
A) Rx de trax. mucosa testoreada sin lesiones sugerentes de hemorragia. La biopsia
B) Trnsito digestivo. C) mostr atrofia total de vellosidades, lo cual sugiere:
Endoscopia alta. A) Glardiasis.
D) Manometra esofgica. E) B) Mucosa normal. C)
TAC torcica. Celaca.
D) Enfermedad de Crohn. E)
69. Un varn de 60 aos sin antecedentes personales ni familiares de Linfoma intestinal.
inters presenta sangre roja mezclada con las heces. El mdico
realiza una inspeccin perianal y un tacto rectal encontrando 74. Una mujer de 65 aos con artritis reumatoide deformante grave se
hemorroides internas grado III. La actitud ms correcta sera: presenta en el hospital por dolor periumbilical de inicio nocturno, con
A) Medidas higinicas ms pomada anti hemorroidal. B) Solicitar aumento rpido de intensidad. La exploracin abdominal era casi
endoscopia alta. normal con hemocultivo positivo, recuento de leucocitos de
C) Solicitar un enema opaco. 20.000 mm3 con desviacin a la izquierda y VSG > 100 mm/h.
D) Solicitar rectoscopia y enema opaco. E) Cul es el diagnstico ms probable?:
Solicitar colonoscopia total. A) Colitis ulcerosa.
B) Enfermedad de Crohn. C)
70. Paciente de 43 aos en tratamiento por colitis ulcerosa con Isquemia intestinal.
corticoides y sulfasalazina; acude a urgencias por malestar D) Parasitosis intestinal.
general, fiebre, distensin E) Angiodisplasia de colon.

[8]
Mg. Dany Colca
RESIDENTADO DE ENFERMERA. Compendio Banco de Preguntas. Lic. Carolina Ayala

75. Varn de 78 aos con debilidad, prdida de peso, diarrea, artritis C) Prolactinoma hipofisario. D)
y fiebre, durante el ltimo ao. En el examen fsico: prdida de Sndrome de Asherman. E) Tumor
masa muscular, linfadenopata y tumefaccin en rodilla izquierda. En cerebral.
pruebas de laboratorio: anemia ferropnica y hemault positivo. Con
Rx abdomen y enema opaco banales. El diagnstico diferencial debe 79. Una mujer de 54 aos solicita tratamiento hormonal sustitutivo por
incluir todos los siguientes, excepto: sntomas neurovegetativos y manifestaciones genitourinarias
A) SIDA. importantes. Refiere que la >FUR fue hace un ao y medio, pero que
B) Enfermedad de Crohn. C) hace 2 meses ha empezado a sangar muy abundantemente. Presenta
Vasculitis reumatoide. exploracin fsica y mamografa normal. Cul es la actitud ms
D) Enfermedad de Whipple. E) apropiada en esta paciente?:
Singellosis. A) Tratamiento con gestgenos. B)
Tratamiento con estrgenos.
76. Se hace colonoscopia en un enfermo, donde aparece un C) Tratamiento con terapia combinada.
plipo de 8 mm en colon sigmoide que se extirpa sin observar otras D) Ecografa transvaginal para valorar lnea media y si existe alguna
lesiones. Cul de los siguientes considera el intervalo de vigilancia duda de patologa endometrial realizar histologa endometrial.
en este paciente?: E) Citologa (triple toma).
A) Seis meses. B)
Un ao. 80. Una mujer de 45 aos con el antecedente de un proceso gripal
C) Dos aos. D) hace dos semanas por el que fue tratado con amoxicilina presenta
Tres aos. E) ahora un cuadro de prurito vulvar y leucorrea. En la exploracin se
Cinco aos. aprecia enrojecimiento y edemas de la vulva y del introito y
secrecin vaginal blanca grumosa de aspecto caseoso. En la mucosa
77. Nia de 6 aos remitida a consulta por hemorragia vginal, que vaginal aparecen unas placas blanquecinas irregulares que se
presenta un desarrollo mamario, en etapa III de Tanner, estatura alta, desprenden con facilidad y dejan ulceraciones rojas superficiales.
con una edad sea de 9 aos (Rx. de mano y mueca izda.), valores La etiologa probable es?:
basales de gonadotropinas y estradiol elevados para la prepubertad A) Candidiasis o moniliasis. B)
con ovarios aumentados para la edad y con mltiples quistes de Herpes genital.
dimetro igual o mayor de 14 mm. Examen neurolgico clnico- C) Tricomaniasis.
radiolgico normal, sin pigmentacin cutnea en mancha de caf con D) Gardnerella vaginalis. E)
leche y estudio hormonal tiroideo y suprarrenal normal. El Clamidias.
tratamiento de eleccin es:
A) Laparocopia diagnstico-teraputica ya que su etiologa es un 81. Mujer de 40 aos que consulta por cuadro de poliartritis simtrica
tumor ovrico. de grandes y pequeas articulaciones de 15 das de evolucin. En la
B) Agonistas de GnRH, ya que estamos ante una PPV, y los analtica destaca un factor reumatoide positivo siendo el resto del
agonistas de GnRH son los nicos que retrasan el desarrollo sexual y estudio inmunolgico negativo. Qu diagnstico realizara?:
la maduracin esqueltica. A) Artritis reumatoide.
C) Danazol. B) Artritis paraneoplsica. C)
D) Acetato de medroxiprogesterona ya que producen un retraso en el Artritis no filiada.
desarrollo sexual y muy buenos resultados en el control del D) Enfermedad de Still del adulto. E)
crecimiento. Poliartritis vrica.
E) No precisa tratamiento, aunque s una vigilancia anual.
82. Mujer de 65 aos que ingresa por cuadro de cefalea, fiebre y dolor
78. Una paciente de 30 aos, que consulta por amenorrea secundaria, con limitacin de ambos hombros y ambas caderas de 2 meses
presenta concentraciones plasmticas basales de FSH 2 MUI/ml., LH de evolucin. El resto de anamnesis y exploracin fsica no aporta
1,5, MUI/ml., prolactina 9 ngr./ml. Tras la administracin de datos relevantes. La analtica pone de manifiesto una gran elevacin
gestgenos 10 mgr./da durante 5 das no se observa sangrado de los reactantes de fase aguda (VSG y PCR). La conducta a
vaginal. En cambio, tras la administracin de estrgenos conjugados seguir sera:
durante 21 das y en los 5 ltimos das gestgenos aparece una A) Diagnosticar a la paciente de polimialgia reumtica e iniciar
menstruacin. De las siguientes causas de amenorrea cul es la que tratamiento.
corresponde con el cuadro clnico?: B) Realizar biopsia de la arteria temporal para descartar la existencia
A) Sndrome de ovario poliqustico. B) de arteritis pues el tratamiento difiere.
Fallo ovrico autoinmune.

[9]
Mg. Dany Colca
RESIDENTADO DE ENFERMERA. Compendio Banco de Preguntas. Lic. Carolina Ayala

C) Iniciar tratamiento con prednisona 1 mg./kg./da para curarse en E) Solicitar analtica, radiografas y gammagrafa con tecnecio y
salud ante la posible existencia de una arteritis. galio ante la sospecha de DSR.
D) Realizar artrocentesis de un hombro o una cadera para descartar
primero la existencia de una artritis sptica. 86. Mujer de 20 aos que acude por dolor y tumefaccin de ambos
E) Iniciar tratamiento con 20 mg./da de prednisona y si no mejorara tobillos junto lesiones cutneas eritematoviolceas dolorosas en
en 15 das subir a 1 mg./kg./da. ambas EEII de 10 das de evolucin. El diagnstico ms probable es:
A) Artritis reactiva. B)
83. Varn de 25 aos afecto clnica y radiolgicamente de una Vasculitis.
sacroiletis unilateral de 2 meses de evolucin. La conducta a seguir C) Sarcoidosis.
es la siguiente: D) Enfermedad inflamatoria intestinal. E)
A) Solicitar analtica con HLA y dar tratamiento con AINE ante la Artropata no filiada.
sospecha diagnstica de espondiloartropata.
B) Adems de lo anterior aadir salazopirina al tratamiento pues 87. Una paciente de 55 aos sin antecedentes patolgicos de inters
probablemente padecer una espondilitis anquilosante. consulta por dorsalgia de inicio brusco y ritmo mecnico de dolor.
C) Adems de lo anterior realizar una buena anamnesis y exploracin Aporta Rx simple de columna en la que se aprecian aplastamientos
fsica para descartar la existencia de psoriasis. vertebrales mltiples. La exploracin fsica es normal salvo por la
D) Realizar anamnesis y exploracin fsica, solicitar analtica presencia de debilidad muscular proximal. Ante la sospecha clnica de
con HLA as como ppD y serologas a brucella. E) Lo primero es osteomalacia, cul de las siguientes exploraciones es ms rentable:
realizar una artrocentesis de la articulacin afecta para A) VSG y hemograma.
descartar proceso infeccioso crnico. B) Determinacin de calcio, fosforo y fosfatasas alcalinas.
C) Densitometra sea. D)
84. Mujer de 45 aos que acude a la consulta por dolor seo Gammagrafa sea.
generalizado desde hace varios aos. La conducta a seguir es: E) Resonancia magntica.
A) Diagnosticarla de fibromialgia y tratarla con analgsicos y
antidepresivos. 88. Una paciente de 23 aos con antecedentes de ulcus duodenal
B) Sospechar la posible existencia de proceso metastsico y realizar presenta una poliartritis simtrica con afeccin predominante de
un completo estudio de bsqueda del tumor primario. manos. En la exploracin fsica, aparte de la poliartritis, presenta aftas
C) Realizar una correcta anamnesis, exploracin fsica y solicitar orales. Se practica analtica general que muestra como nicas
analtica completa con hormonas tiroideas y CPK. alteraciones VSG: 40 mm/1. hora, leucocitos: 3.000/mm3 (1.200
D) Adems de lo explicado en el apartado 3., solicitar una linfocitos) y ANA + 1/320 patrn homogneo. Cul sera su
gammagrafa sea para asegurarse de la existencia de proceso diagnstico?:
inflamatorio articular. A) Artritis reumatoide.
E) Realizar una anamnesis y exploracin fsica correctas. B) Lupus eritematoso sistmico. C)
Solicitar un estudio analtico con VSG y PCR. Cuando exista Gota poliarticular.
sospecha clnica solicitar determinacin de hormonas tiroideas, D) Condrocalcinosis. E)
anticuerpos anti-ANA y creatin- P-quinasas. Artritis reactiva.

85. Varn de 45 aos que acude a la consulta por presentar dolor, 89. Cul es el tratamiento indicado en la paciente de la pregunta
tumefaccin y edema a nivel de la mano izquierda. Entre sus anterior?:
antecedentes patolgicos nicamente destaca la luxacin del hombro A) Antiinflamatorios no esteroideos. B)
izquierdo 1 mes antes. La conducta a seguir es la siguiente: Antipaldicos.
A) Realizar artrocentesis de la mueca para descartar artritis sptica. C) Glucocorticoides orales.
B) Solicitar analtica y radiografas ante la sospecha de artritis de D) Pulsos de metilprednisolona. E)
mueca. Inmunosupresores.
C) Solicitar analtica, radiografas y gammagrafa sea con tecnecio
ante la sospecha de DSR. 90. Un paciente de 45 aos consulta porque en una analtica de rutina
D) Solicitar analtica con inmunologa y radiografas ante se ha detectado una uricemia de 9 mg./dl. No existen antecedentes
la sospecha de inicio de proceso inflamatorio crnico. de artritis ni clculos urinarios. Cul es la actitud correcta?:
A) Iniciar tratamiento con uricosricos.
B) Iniciar tratamiento con uricosricos y colchicina. C) Iniciar
tratamiento con alopurinol.
D) Iniciar tratamiento con alopurinol y colchicina. E) No
precisa tratamiento.

[10]
Mg. Dany Colca
RESIDENTADO DE ENFERMERA. Compendio Banco de Preguntas. Lic. Carolina Ayala

91. Una paciente de 15 aos es remitida por sospecha de fiebre D) Que diga que tiene fro. E)
reumtica: tras un cuadro gripal con artromialgias generalizadas se Uas y pelo frgil.
determinaron las antiestreptolisinas (ASLO), que son positivas. Cul
de las siguientes afirmaciones es verdadera?: 95. Una adolescente de apariencia fsica normal acude por irritabilidad
A) Ante el cuadro clnico de la paciente y las ASLO +, el diagnstico y tristeza. Comenta que decidi comer menos en las comidas porque
de fiebre reumtica es seguro. estaba algo gorda. Haciendo esto consigue adelgazar a temporadas
B) Con las ASLO +, bastara para diagnosticar la fiebre reumtica. pero luego se vuelve a engordar. Ahora est en perodo de transicin,
C) La fiebre reumtica es un tipo de artritis sptica. sigue restringiendo las comidas pero se pasa todo el da picoteando
D) Con estos datos, no se puede realizar el diagnstico de fiebre y en ocasiones come ms de lo que quisiera de forma descontrolada.
reumtica. En esos descontroles efectivamente la paciente come una gran
E) Debemos iniciar rpidamente tratamiento con penicilina. cantidad de alimentos especialmente chocolate, galletas y dulces,
aunque en otras ocasiones son salados. Tras estos atracones se
92. Una paciente de 40 aos presenta una gonartosis con afeccin siente muy culpable e irritada y vomita para evitar engordarse. El
predominante del compartimento interno de la rodilla e impotencia diagnstico ms probable es:
funcional severa. Seale la respuesta verdadera: A) Sndrome de Klein-Levin. B)
A) El tratamiento es reposo absoluto y esperar a que sea ms Depresin bipolar.
mayor para colocarle una prtesis. C) Bulimia.
B) Una osteotoma podra estar indicada. D) Esquizofrenia.
C) Nunca se debe tratar con antiinflamatorios. E) Anorexia nerviosa tipo compulsivo/purgativo.
D) No debe realizar fisioterapia ya que podra daarse ms la
articulacin. 96. Una paciente de 20 aos se presenta en la guardia quejndose al
E) El uso de un bastn est contraindicado. internista de que tiene ataques de corazn y sensacin de ahogo,
sudoracin y sensacin de mareo que se inicia bruscamente mientras
93. Un paciente presenta latencia del sueo de menos de 10 minutos estaba leyendo relajadamente en su casa, le dur unos minutos,
con dificultades para despertarse y episodios de sueo de 18-20 crey morir y senta que lo que le ocurra no era real, temiendo
horas, asociado al despertarse con hiperfagia, hipersexualidad, perder el control o estar volvindose loca. Se le realizan pruebas ECG
aumento de peso, irritabilidad, depresin, comportamiento impulsivo, y auscultacin, detectndose frecuencia cardaca alta y signos
disfuncin vegetativa y alteraciones neurolgicas. Las siestas sugerentes de prolapso mitral. Aun as dada la gran ansiedad de la
diurnas duran varias horas. Estos episodios duran varias semanas paciente se llama al psiquiatra ya que insiste en que se le hagan ms
intercalndose varios meses sin somnolencia. En uno de los pruebas y en que no se quiere quedar sola en casa nunca ms,
episodios de somnolencia el paciente tuvo un grave accidente. El cul es el posible diagnstico?:
diagnstico ms probable es: A) Prolapso mitral. B)
A) Epilepsia. Agorafobia.
B) Narcolepsia. C) Angor.
C) Simulacin o trastorno conversivo-histeria. D) D) Trastorno de angustia. E) A
Sndrome de Klein-Levin. y D.
E) Apnea del sueo.
97. Una paciente de 55 aos acude a consultas por cansancio
94. Una chica de 17 aos acude al servicio de urgencias trada por su especialmente por la maana, falta de concentracin desde hace 1
madre por un desmayo. Cuando la vemos est irritable y dice querer mes. Dice estar por las maanas despierta antes de hora sin poder
irse de alta porque no le pasa nada. Va con ropas holgadas y algo dormir. Ha perdido apetito si bien come como siempre pero
ms abrigada que el resto de pacientes, est delgada y parece ms forzndose, por lo que no ha perdido peso. Pierde el hilo de las
joven de lo que le corresponde por su edad. Dice comer normal conversaciones y est irritable. Ha dejado de hacer las cosas que
y niega usar laxantes o diurticos, su madre dice que no come con le gustaba hacer y no sale de casa ms que lo justo. Viene
ellos porque es muy activa y est todo el da fuera. Rompi con el acompaada por su hermana y nos ruega que no le digamos nada a
novio hace un ao. Dice que est harta de tener a su madre siempre su marido ya que desde que no hace las cosas de la casa tan bien
pendiente de ella. El dato que ms nos ayudara a diferenciar una como antes no quiere darle ms disgustos. Cul es el diagnstico
anorexia nerviosa de otros trastornos sera: ms probable?:
A) Presencia de amenorrea. B) A) Ansiedad y depresin. B)
Peso inferior al 15%. Demencia.
C) Que la paciente diga que tiene miedo a ganar peso o C) Depresin mayor.
que diga que se ve gorda. D) Trastorno de personalidad.

[11]
Mg. Dany Colca
RESIDENTADO DE ENFERMERA. Compendio Banco de Preguntas. Lic. Carolina Ayala

E) Neurosis. siempre est comentando lo que l va haciendo. Cul es el


diagnstico ms probable?:
98. El paciente, que es trado por un familiar, dice no saber porqu A) Esquizofrenia.
est aqu y parece irritado y con agresividad contenida. El familiar nos B) Depresin psictica. C)
hace gestos con los ojos haciendo ver que el paciente no est muy Mana.
bien. Entrevistando a ambos por separado el paciente se muestra D) Demencia. E)
contenido y dice estar bien, mejor que nunca. El familiar dice que Klein-Levin.
ltimamente ha comprado cosas innecesarias y ha hecho algunos
regalos. Dice que quiere iniciar la carrera de psicologa, se ha TEMA B
apuntado a un gimnasio y quiere proponer a su jefe algunos
cambios en la empresa si bien no tiene responsabilidades directivas. 1. Paciente de 64 aos ingresado para ciruga diferida que al da
Est muy hablador y agudo en las conversaciones pero irritable, siguiente del ingreso presenta cuadro de hiperactividad vegetativa,
dominante y con explosiones de mal genio. Debe dormir 4 horas pero temblor, sobresaltos, hiperactividad e hiperalerta pero con confusin,
no est cansado al da siguiente, no hay problemas de apetito. No alucinaciones y convulsiones. Cul es la causa ms frecuente?:
ha habido problemas en su trabajo, pero pasa mucho tiempo en A) Abstinencia a alcohol.
la calle. Cul es el diagnstico ms correcto?: B) Sobredosis de neurolpticos.
A) Esquizofrenia. B) C) Sobredosis de benzodiacepinas. D)
Mana. Sobredosis de anticolinrgicos. E)
C) Depresin ansiosa. Ingestin de barbitricos.
D) Trastorno esquizoafectivo. E)
Trastorno de personalidad. 2. Mujer de 25 aos que acude por intento autoltico con
benzodiacepinas. Es el quinto intento en 7 aos, en esta ocasin tras
99. Paciente trado por la polica por haberlo encontrado andando una pelea familiar. Su madre dice que es muy voluble y caprichosa
descalzo por la carretera. El paciente tiene 28 aos, es de otra que siempre consigue salirse con la suya mediante grandes crisis de
ciudad y por la documentacin que lleva parece que pueda ser nervios y peleas. La madre dice estar harta ya que siempre est
estudiante. Parece deshidratado, completamente desaseado y con regandola mientras que su marido la perdona todo y no la castiga
olor a orn. Pupilas normales. La piel est tostada sin quemaduras en lo suficiente. No tiene novio fijo ya que rompe con ellos en seguida,
las zonas de exposicin al sol. Est ausente, como absorto de manera es mala estudiante y no es capaz de dedicarse a nada fijo ya que se
que al hablarle ms fuerte se sobresalta y nos atiende aburre en seguida, dice sentirse en ocasiones muy vaca y en
momentneamente sonrindonos de forma inapropiada hasta que otras muy inspirada en hacer el bien pero nada le dura mucho,
ladea la cabeza y la gira sin motivo. Murmura algo y mira al quisiera poder tener una relacin con un hombre romntico. Sale
techo, permaneciendo con los brazos extendidos. Cul es el mucho por la noche y en ocasiones no llega hasta el da siguiente
diagnstico ms probable?: por la tarde, llegando ebria en ocasiones. Ha perdido a sus
A) Intoxicacin por LSD. B) amigas, que se quejaban entre otras cosas de que las estaba
Esquizofrenia. llamando continuamente. Le han diagnosticado previamente de
C) Demencia. hepatitis. El diagnstico de esta paciente es:
D) Cuadro manaco. E) A) Esquizofrenia. B)
Histeria. Mana.
C) Trastorno de personalidad lmite o inestable. D)
Trastorno de personalidad por dependencia.
100. Acude la madre de un presunto paciente de 18 aos porque E) Trastorno esquizoide.
su hijo no sale casi nada de casa, parece evitarles. Se hace su
propia comida a partir de conservas generalmente y come aparte. 3. El paciente acude a nuestra consulta porque quisiera dejar de hacer
Nunca usa conservas o alimentos ya empezados, ni acaba los que no algunas de las cosas que actualmente hace ya que le quitan mucho
usa por completo. Alguna vez lo ha visto en el pasillo haciendo tiempo. El paciente tarda mucho tiempo en vestirse ya que ha de
posturas extraas o dando golpes de krate. Fue aficionado a las pensar la ropa que le dar suerte para esa maana. Teme que si no
lecturas de ovnis y ciencia ficcin. Fue regular estudiante. No trabaja coge la adecuada algo podra salir mal. No puede evitar sumar los
ni estudia. Cierra todas las persianas y se queja de que los vecinos nmeros que ve en las matrculas de los coches, as consigue
estn demasiado pendientes de ellos. Por las noches a veces no neutralizar los pensamientos que le vienen sobre la posibilidad de
duerme. Dice que puede or al vecino del tercero (ellos viven en el que el coche tenga un accidente. El sabe que stos son tonteras
primero) hablar mal de ellos y que suyas pero son cosas que no puede quitarse de la cabeza. Es muy
ordenado en algunos aspectos pero se le acumula el

[12]
Mg. Dany Colca
RESIDENTADO DE ENFERMERA. Compendio Banco de Preguntas. Lic. Carolina Ayala

trabajo y no consigue hacer las cosas que tiene preparadas en una 7. Ante una paciente que ingresa en el Servicio de Urgencias con
lista para ese da. Le gusta corregir sus escritos hasta que estn dolor abdominal, elevacin de la amilasa y la glucosa en suero y con
impecables, lo mismo le ocurre con su trabajo, es muy perfeccionista, un pH de 7,1 en qu pensara primero?:
exigente consigo mismo y con los dems, es incapaz de delegar A) Pancreatitis aguda.
trabajo. El diagnstico del paciente es: B) Rotura de embarazo ectpico. C)
A) Esquizofrenia. Quiste de ovario.
B) Trastorno de personalidad dependiente. C) D) Cetoacidosis diabtica. E)
Trastorno obsesivo-compulsivo. Ulcera de estmago.
D) Fobia.
E) Depresin. 8. Un paciente de 18 aos consulta por ictericia sin fiebre y
coluria sin prurito. En las pruebas complementarias presenta: Ac. anti
4. Paciente que acude por no poder salir de casa sola. Dice que le da VHA IgG positivos, Ac. antiHBs positivos, Ac. anti HBc IgG positivos,
miedo salir por sitios solitarios por si le ocurre algo y que en los sitios ecografa heptica normal, hay predominio de la bilirrubina directa
con mucha gente se agobia y se pone muy nerviosa teniendo crisis con monoconjugados, la colecistografa oral es normal y al dar al
de ansiedad con palpitaciones, se imagina as misma necesitada de paciente fenobarbital se observa disminucin de las cifras de
ayuda y sin poder salir. Si se encuentra acompaada se encuentra bilirrubina total. Cul sera su diagnstico de sospecha?:
mejor ya que esa persona podra ayudarla si ocurriera algo. Desde A) Coledocolitiasis.
hace unos meses tiene unas crisis parecidas pero ms cortas estando B) Sndrome de Gilbert.
relajada, incluso acompaada, sin aviso, la primera estando en la C) Sndrome de Dubin-Johnson.
calle distrada viendo un escaparate. Cuando le dan en casa es capaz D) Hepatitis aguda por VHA, con hepatitis aguda por
de salir a la calle para coger aire o bien abre las ventanas. Desde la VHB curada.
primera crisis cogi algo de miedo a estar sola hasta llegar a la E) Sndrome de Rotor.
situacin actual. Se ha comprado un telfono mvil para poder pedir
ayuda si fuera necesario. Si entra en un cine siempre est pendiente 9. Ante un paciente con cirrosis alcohlica en estado C-
de sentarse cerca de alguna salida. El cuadro clnico de la paciente es: 10 de Child-Pugh que ingresa por descompensacin hidrpica, y
A) Agorafobia. B) que se encuentra hipotenso, taquicrdico, oligrico, con un sodio
Hipocondra. srico de 125 mEq/l. y de 5 mEq/l. en orina, con sedimento normal,
C) Trastorno facticio. D) creatinina srica
Trastorno obsesivo. 3 mg./dl. y un aclaramiento de creatinina de 40 ml., en qu pensara
E) Agorafobia con crisis de pnico. usted?:
A) Deficiente tratamiento diurtico.
5. Ante un paciente con dolor epigstrico irradiado en hemicinturn, B) Insuficiencia renal prerrenal secundaria a tercer espacio.
nuseas, distensin abdominal, descenso de los ruidos hidroareos y C) Sndrome hepatorrenal.
ascenso del ST en el electrocardiograma, cul sera su actitud?: D) Glomerulonefritis mesangial IgA.
A) Llamara de inmediato a la Unidad de Cuidados E) Insuficiencia renal prerrenal secundaria a deshidratacin.
Intensivos para tratar el infarto agudo de miocardio.
B) Repetira el electrocardiograma a las 8 horas para confirmar el 10. Paciente varn de 28 aos, VIH(+), en tratamiento con DDI
diagnstico. (Dideoxinosina), que acude al hospital por dolor intenso en epigastrio
C) Lo diagnosticara de pancreatitis aguda. irradiado hacia la espalda, que mejora al flexionar el tronco,
D) Su diagnstico sera aneurisma disecante de aorta. E) El acompaado de nuseas y vmitos. Cul de los siguientes
paciente tiene una pericarditis aguda. diagnsticos es el ms probable?:
A) Pericarditis.
6. Ingresa en el Servicio de Urgencias un paciente con dolor B) Obstruccin intestinal. C)
epigstrico irradiado en hemicinturn y amilasa elevada, que pierde Apendicitis.
de forma brusca visin qu pensara que ha ocurrido?: D) Pancreatitis aguda. E)
A) Retinopata de Purtscher. Endocarditis.
B) Accidente cerebro vascular agudo de la regin occipital
posterior. 11. Tras una semana de ingreso por una pancreatitis aguda, a pesar
C) Desprendimiento de retina. D) del tratamiento mdico sintomtico persiste la fiebre, leucocitosis e
Simulacin. hiperamilasemia. Se aprecia a la palpacin una masa abdominal
E) Glaucoma agudo. localizada en hipocondrio derecho. Cul es el diagnstico ms
probable?:

[13]
Mg. Dany Colca
RESIDENTADO DE ENFERMERA. Compendio Banco de Preguntas. Lic. Carolina Ayala

A) Carcinoma de pncreas. B) 16. Un varn de 13 aos que haba presentado un cuadro febril de
Flemn pancretico. vas respiratorias altas de una semana de evolucin acude a
C) Pancreatitis crnica. urgencias por vmitos sbitos e incoercibles. La semana previa haba
D) Plastn secundario a perforacin duodenal. E) Quiste consumido cido acetilsaliclico para la sintomatologa respiratoria y
hidatdico. tres semanas antes estuvo en contacto con un paciente con hepatitis
aguda por virus B. Dos das despus del ingreso el paciente se
12. El principal diagnstico de presuncin ante una enferma que encuentra estuporoso con convulsiones sin signos neurolgicos de
acude por un cuadro de diarrea acuosa, dos lceras duodenales focalidad y dolor en hipocondrio derecho con hepatomegalia. Cul
resistentes al tratamiento mdico e hipercalcemia es: sera su diagnstico?:
A) Insulinoma. B) A) Cuadro convulsivo en relacin con la fiebre. B)
Gastrinoma. Hepatitis fulminante vrica.
C) Adenocarcinoma de pncreas. D) C) Sndrome de Reye.
Somatostatinoma. D) Reagudizacin del cuadro gripal.
E) Vipoma. E) Intoxicacin por cido acetilsaliclico.

13. Varn de 60 aos que acude por prdida de peso de 17. Un hombre de 21 aos recibi un golpe en el escroto dos horas
10 kilos, dolor sordo en piso abdominal superior de 3 meses de antes de ser examinado en urgencias. Su escroto est tenso,
evolucin, acompaado de ictericia mucocutnea y deposicin de hinhado, y equimtico. No se pueden palpar los testculos. El paso
color blanco desde hace 5 das. A la exploracin destaca masa siguiente es:
palpable en hipocondrio derecho. Qu patologa se sospechara en A) Hacer uretrografa retrgrada.
primer lugar?: B) Hacerle un Doppler de flujo color. C)
A) Adenocarcinoma pancretico. B) Realizar una ecografa del escroto.
Tumor gstrico. D) Tratarle con hielo, reposo y suspensorio. E) Hacer
C) Coledocolitiasis. D) exploracin quirrgica del escroto.
Colecistitis.
E) Hepatitis aguda. 18. Un hombre de 64 aos presenta una hinchazn indolora del
testculo derecho de tres meses de duracin. Los resultados de
14. Varn de 45 aos, etilismo crnico, con dolor abdominal en el los anlisis de orina son normales y la ecografa testicular muestra un
hipocondrio izquierdo, de 3 meses de evolucin, que empeora con la aumento del tamao de dicho teste.
ingesta y se acompaa de deposiciones diarreicas pastosas muy mal El diagnstico ms probable es: A)
olientes. En las exploraciones complementarias destaca: glucemia Linfoma testicular.
de 280, amilasemia en los lmites de la normalidad y en la placa de B) Leucemia linftica crnica. C)
abdomen mltiples calcificaciones a nivel de L2: Seminoma espermatoctico. D)
A) Pancreatitis aguda. B) Teratocarcinoma.
Clico biliar. E) Carcinoma de clulas embrionarias.
C) Pancreatitis crnica. D)
Hepatitis aguda. 19. Una mujer sana presenta de forma aguda polaquiuria y
E) Ulcus gstrico disuria. En el sedimento urinario se observan ms de 5 leucocitos
por campo y el urinocultivo revela
15. Paciente de 60 aos con fibrilacin auricular en tratamiento con 1.000 colonias de E. coli por ml. El diagnstico ms probable
amiodarona desde hace meses, colelitiasis diagnosticada es:
ecogrficamente e insuficiencia renal crnica moderada. Consulta por A) Uretritis por clamydia. B)
cuadro de anorexia, astenia, nuseas, vmitos e ictericia Sndrome uretral.
mucocutnea. En los datos de laboratorio destaca un leve aumento C) Bacteriuria por E. coli. D)
de las transaminasas y una creatinina de Cistitis intersticial.
2,2. Se realiza una biopsia heptica, observndose al microscopio E) Cistitis qustica.
electrnico cuerpos lisosmicos lamelares cargados de fosfolpidos.
Qu proceso patolgico le sugieren estos datos: 20. Un hombre de 74 aos con cncer de prstata metastsico
A) Hepatitis viral aguda. B) conocido presenta dolor agudo de cadera derecha. Hace dos aos se
Clico biliar. le practic una orquiectoma pero no ha recibido ningn otro
C) Pancreatitis aguda. tratamiento. Su estado general es bueno. Una gammagrafa sea
D) Hepatitis txica de origen medicamentoso. E) muestra metstasis difusas y una Rx simple revela una ostelisis en
Hepatitis viral crnica. el acetbulo derecho. La siguiente medida a aplicar es:
A) Flutamida.

[14]
Mg. Dany Colca
RESIDENTADO DE ENFERMERA. Compendio Banco de Preguntas. Lic. Carolina Ayala

B) Anlogos de la LH-RH. C) 26. En un paciente asmtico, cul de los siguientes frmacos est
Radioterapia localizada. D) contraindicado en el tratamiento de la incontinencia?:
Fosfato de estramustina. E) A) La efedrina. B)
Ketoconazol. Sudafed.
C) El propranolol.
21. Un recin nacido presenta hematuria, proteinuria y creatinina D) La fenilefrina.
elevada. Las presiones de la arteria umbilical estn significativamente E) Las anfetaminas.
altas y el paciente desarrolla una insuficiencia cardaca congestiva.
Una gammagrafa renal revela una ausencia de funcin en el rin 27. Ante un paciente de 60 aos con un cncer de prstata
izquierdo. El diagnstico ms probable es: localizado, mal diferenciado y sin metstasis, qu tratamiento le
A) Trombosis de la vena renal. B) recomendara para intentar aumentar su supervivencia:
Necrosis cortical renal. A) Prostatectoma radical. B)
C) Hemorragia adrenal. Anlogos de la LH-RH. C)
D) Rotura de un nefroma mesoblstico. E) Flutamida.
Trombosis de la arteria renal. D) Estramustina. E)
Orquiectoma.
22. Un hombre de 35 aos presenta un clico renal izquierdo. En la
Rx de abdomen se observa un clculo de 3 mm. de dimetro alojado 28. Ante un paciente de 55 aos que presenta una tumoracin vesical
en urter medio. La necesidad de tratamiento quirrgico depende de: que infiltra la muscular y que ocupa la mitad de la vejiga, cul
A) La duracin de los sntomas del paciente. sera el tratamiento que empleara con intencin curativa?:
B) Del nmero de episodios de clicos previos. A) Cistectoma radical y derivacin urinaria. B)
C) Del nmero de intervenciones quirrgicas previas. D) De la Reseccin transuretral vesical.
presencia de fiebre e infeccin urinaria. C) Quimioterapia intravesical. D)
E) De una anormalidad metablica subyacente. Quimioterapia sistmica.
E) Cistectoma parcial.
23. A un paciente se le somete a una intervencin de bypass en el
intestino delgado y presenta una litiasis urinaria. Cul ser la 29. Paciente mujer de 75 aos de edad, que es trada a la consulta
composicin ms probable de la litiasis?: por cambio progresivo de conducta en los ltimos meses.
A) Acido rico. Previamente haban observado fallo de memoria reciente sin poder
B) Urato amnico. C) precisar el momento de inicio. Destacaba a la exploracin lenguaje
Fosfato clcico. D) pobre y fallos de juicio. Cul es el diagnstico ms probable en esta
Oxalato clcico. E) paciente?:
Estruvita. A) Depresin.
B) Sndrome confusional agudo. C)
24. El factor pronstico ms importante en los nios que presentan Demencia tipo Alzheimer.
un tumor de Wilms intracava es: D) Demencia multiinfarto.
A) La histologa. E) Trastorno de la personalidad.
B) El volumen del tumor.
C) La extensin atrial del tumor. 30. Varn de 87 aos, con amaurosis bilateral, que ingresa por
D) La afectacin de ganglios linfticos. neumona basal derecha en la Unidad de Agudos del hospital. La
E) La diseminacin del tumor durante la intervencin quirrgica. noche del ingreso presenta cuadro de agitacin psicomotriz, con
alteracin del nivel de conciencia y desorientacin temporoespacial.
25. Un muchacho de 7 aos presenta de forma sbita dolor escrotal Cul es el diagnstico ms probable?:
derecho de 4 horas de duracin. Se sospecha de una torsin A) Demencia.
testicular. Cul de las siguientes observaciones es la ms probable?: B) Depresin delirante. C)
A) La ausencia del reflejo cremastrico. ACVA.
B) El aumento de la captacin por parte del teste derecho D) Sndrome confusional agudo. E)
despus de la exploracin radioisotpica. Ninguno.
C) La presencia del reflejo cremastrico.
D) Una transiluminacin correcta del compartimento escrotal 31. Paciente de 85 aos de edad con antecedentes de insuficiencia
derecho. cardaca en tratamiento con diurticos, e insomnio que trataba con
E) La estetoscopia con Doppler ser simtrica en ambos lorazepam. Presenta nicturia
compartimentos escrotales. 2-3 veces. Sufre cada al levantarse bruscamente durante la noche
para ir al retrete. Qu factores pueden haber contribuido e la
cada?:

[15]
Mg. Dany Colca
RESIDENTADO DE ENFERMERA. Compendio Banco de Preguntas. Lic. Carolina Ayala

A) Frmacos. 36. Durante los ltimos 3 meses un varn de 80 aos presenta una
B) Hipotensin postural. C) rpida progresin de una demencia acompaada de signos
Factores ambientales. D) extrapiramidales y mioclonas. El diagnstico de sospecha inicial
Ninguno. sera:
E) Todos ellos. A) Demencia senil tipo Alzheimer.
B) Enfermedad de Creutzfeldt-Jakob. C)
32. Paciente de 75 aos de edad, varn, que vive solo. Es encontrado Enfermedad de Huntington.
la maana del da 15 de enero cado en el suelo. A su llegada al D) Hidrocefalia a presin normal. E)
hospital se evidencia afasia motora y hemipleja derecha Enfermedad de Parkinson.
diagnosticndose de ACVA. Cul de las siguientes patologas debe
ser descartada en la valoracin inicial?: 37. Cul de las lesiones cutneas siguientes se asocia ms
A) Rabdomilisis. B) frecuentemente con neoplasia oculta en un paciente anciano?:
Hipertiroidismo. C) A) Penfigoide bulloso. B)
Depresin. Dermatomiositis.
D) Hipotermia. E) C) Eritema multiforme. D)
A y D. Herpes zoster.
E) Pnfigo vulgar.
33. Paciente mujer de 85 aos que ingresa por fractura pertrocantrea
de cadera derecha colocndose traccin. Se evidencia neumona 38. Anciano de 70 aos con un melanoma maligno de reciente
basal derecha que obliga a retrasar la intervencin. La paciente sufre diagnstico y sin otra patologa mdica. Su pronstico vital estar ms
de incontinencia urinaria que se maneja con catter. A los cinco das estrechamente relacionado con una de las siguientes caractersticas:
se observa enrojecimiento en regin sacra que no palidece con la A) Ausencia de regresin.
presin, diagnosticndose de lcera por presin grado I. Cul es el B) Tipo histolgico de la lesin. C)
principal factor de riesgo para esta complicacin?: Grado de invasin.
A) Incontinencia urinaria. B) D) Presencia de ulceracin. E)
Hipoxemia. Lugar de la lesin.
C) Edad avanzada.
D) Inmovilidad. E) 39. Una mujer de 75 aos con historia de diabetes mellitus no
Todos. insulindependiente y epilepsa secundaria a enfermedad cerebro-
vascular padece una inflamacin con retraccin gingival. Se
34. Paciente de 70 aos que sufre cada al suelo golpendose en la encuentra en tratamiento con glipizida y fenobarbital. La causa ms
cabeza. Al cabo de unas semanas sufre trastornos de conducta, probable de su proceso gingival ser:
prdida de memoria y posteriormente alteracin del nivel de A) Caries dental. B)
conciencia. Qu diagnstico debe descartarse en primer lugar?: Edentulismo.
A) Hipotiroidismo. C) Tratamiento con fenobarbital. D)
B) Demencia tipo Alzheimer. C) Dficit de cinc.
Pseudodemencia. E) Pobre higiene oral.
D) Hematoma subdural. E)
Tumor cerebral. 40. Un varn de 76 aos con historia de diabetes mellitus de
larga evolucin y con datos clnicos de polineuropata perifrica,
35. Mujer de 75 aos que consulta por incontinencia urinaria, en la empieza a tomar amitriptilina por prescripcin mdica para las
que predomina la urgencia-miccional y que no sigue ningn parestesias en miembros inferiores. De forma rogresiva nota
tratamiento farmacolgico habitual. A travs de la exploracin fsica disminucin del volumen de diuresis y ocasionalmente escapes
no se objetiva patologa orgnica, siendo el residuo postmiccional involuntarios de orina. En la revisin mdica siguiente se objetiva
normal. El estudio analtico es normal. El tratamiento mdico de deterioro del estado general con insuficiencia renal. Cul es la causa
eleccin sera: ms lgica de su deterioro clnico?:
A) Sondaje vesical intermitente. B) A) Nefrotoxicidad por amitriptilina. B)
Calcioantagonistas. Pielonefritis aguda.
C) Ciruga. C) Infeccin urinaria de vas bajas.
D) Antocolinrgicos. E) D) Retencin urinaria con fracaso renal secundario. E)
Colector externo. Glomeruloesclerosis diabtica.

[16]
Mg. Dany Colca
RESIDENTADO DE ENFERMERA. Compendio Banco de Preguntas. Lic. Carolina Ayala

41. Varn de 45 aos con otalgia derecha y sensacin de supraclavicular. A la exploracin presenta taquicardia y leve
taponamiento auditivo, sin otorrea. A los dos das presenta aumento cianosis, un tiroides agrandado y, en la laringoscopia,
del dolor, que se hace retroauricular, y fiebre en agujas. El Rinne es parlisis de ambas cuerdas vocales en posicin paramediana. La
negativo en odo derecho y el Weber lo lateraliza a la derecha. El actitud que debe seguirse es: A) Bolo de corticoides endovenosos,
diagnstico ms probable es: ante la posibilidad de carcinoma subgltico.
A) Colesteatoma antral invasivo. B) B) Tiroidectoma de urgencia, pues probablemente el tiroides
Petrositis. agrandado comprime la trquea.
C) Tromboflebitis del seno lateral. D) C) Intubacin y observacin.
Otitis externa maligna. D) Traqueostoma de urgencia, pues se trata de una parlisis
E) Carcinoma de odo derecho. recurrencial bilateral secundaria a patologa tiroidea.
E) Administracin endovenosa de espasmolticos.
42. Mujer de 22 aos que presenta parlisis facial perifrica derecha.
A la exploracin presenta otoscopia normal y lengua geogrfica 47. Un nio de 3 aos es trado a urgencias con un cuadro de fiebre
fisurada. La paciente comenta haber tenido otro episodio y tos irritativa "perruna", a lo que se asocia disfona y cierto grado de
anteriormente, aquella vez asociado a edema de labio inferior. El disnea. El cuadro es compatible con todas menos:
diagnstico ms probable es: A) Cuerpo extrao en vas areas. B)
A) Sndrome de Guillain-Barr. Laringotraquetis aguda.
B) Sndrome de Heerfordt-Waaldenstrm. C) C) Epiglotitis aguda.
Dficit de C1-inhibidor. D) Adenoamigdalitis aguda obstructiva. E)
D) Sndrome de Melkerson-Rosenthal. E) Edema alrgico.
Parlisis facial de Bell.
48. Paciente mujer de 25 aos que consulta por prdida de audicin
43. Paciente que presenta hipoacusia neurosensorial izquierda de de comienzo insidioso. Su madre era sorda y gan audicin tras
larga evolucin con cada en agudo, y acfeno persistente. La actitud operarse del odo. Otoscopia normal. Rinne negativo odo izdo.,
correcta en este caso sera: positivo odo dcho. Weber a la izda. Audiometra: hipoacusia
A) No hacer nada, pues se trata de un traumatismo acstico transmisiva izquierda. Reflejo estapedial abolido en odo izdo. y
crnico. presente en derecho. Timpanometra normal. El diagnstico probable
B) Potenciales evocados para descartar neurinoma del es:
VIII. A) Otosclerosis odo izquierdo. B)
C) Instaurar terapia vasodilatadora endovenosa, pues se trata de una Colesteatoma izquierdo.
hipoacusia sbita. C) Fijacin de cadena osicular derecha. D)
D) Intervenir el posible colesteatoma. Timpanosclerosis izquierda.
E) Administrar sedantes vestibulares ante la posibilidad de vrtigo E) Luxacin de cadena osicular izquierda.
de Mnire.
49. Un varn de 30 aos acude a urgencias por vrtigo perifrico
44. Paciente de 65 aos con rinorrea unilateral purulenta, intenso con nistagmus a la derecha y acfeno en odo izquierdo.
dolor hemifacial y epistaxis ocasionales. En Rx se observan lesiones Recuerda haber tenido dficit auditivo izquierdo previo, que le
osteolticas en maxilar. Probablemente se trate de: desapareci. Audiometra: hipoacusia neurosensorial izda. leve.
A) Cuerpo extrao nasal. B) Reflejo estapedial: derecho en 70 dB; izquierdo en 50 dB. Diagnstico
Ocena. probable:
C) Granuloma sangrante de tabique. D) A) Neuronitis vestibular.
Carcinoma de fosa nasal. B) Neurinoma del VIII par con reclutamiento positivo. C) Tumor
E) Coriza. de tronco cerebral.
D) Otitis media secretora con fstula perilinftica. E)
45. Un varn de 60 aos consulta por tumoracin indolora en raz Sndrome de Mnire.
nasal, que desplaza la rbita producindole diplopia. El cuadro se
debe probablemente a: 50. Un nios de 2 aos presenta rinorrea purulenta de larga evolucin
A) Mucocele etmoidal. por fosa nasal derecha, con mala ventilacin nasal. Son diagnsticos
B) Quiste de retencin en seno frontal. C) posibles todos menos:
Enfermedad de Woakes. A) Tuberculosis nasal.
D) Papiloma invertido en techo de fosa nasal. E) B) Cuerpo extrao intranasal. C)
Plasmocitoma solitario en seno frontal. Sinusitis maxilar.
D) Coriza comn.
46. Una mujer de 45 aos acude a la urgencia con un sndrome E) Rinitis alrgica sobreinfectada.
de disnea y estridor importantes, con tiraje

[17]
Mg. Dany Colca
RESIDENTADO DE ENFERMERA. Compendio Banco de Preguntas. Lic. Carolina Ayala

51. Paciente de 30 aos que acude a urgencias presentando una 57. Ante un varn de 32 aos con sndrome febril y prdida visual
ulceracin amigdalar unilateral. Son diagnsticos probables todos unilateral que presenta en fondo de ojo lesiones en queso y tomate.
menos: Cul de las siguientes afirmaciones es falsa?:
A) Amigdalitis tifoidea de Duguet. B) A) Sera aconsejable realizar serologa HIV. B) El
Angina de Ludwig. pronstico vital del enfermo es malo.
C) Carcinoma escamoso de amgdala. C) No necesita tratamiento por ser con frecuencia una alteracin
D) Angina de Plaut-Vincent. E) transitoria.
Agranulocitosis. D) Con bastante probabilidad se deber a CMV. E) La
afectacin suele ser bilateral.
52. Un nios de 12 aos acude a urgencias por obstruccin nasal
bilateral crnica y otitis seromucosa bilateral. Recientemente ha 58. Paciente de 70 aos que presenta metamorfosias, micropsia y
tenido epistaxis importante por ambas fosas nasales. El diagnstico disminucin de la agudeza visual de varias semanas de evolucin,
ms probable es: el diagnstico ms probable es: A) Catarata nuclear.
A) Hipertrofia adenoidea. B) Catarata subcapsular posterior. C)
B) Ototubaritis asociada a rinitis alrgica. C) Degeneracin macular senil.
Poliposis nasal bilateral. D) Pars planitis.
D) Adenocarcinoma de etmoides. E) E) Hemorragia vtrea.
Angiofibroma nasofarngeo.
59. Ante un nio de 5 aos con endotropa que ha seguido
53. Mujer de 75 aos con historia de cefalea presenta prdida sbita tratamiento con correccin ptica y colusiones y cuya agudeza visual
de visin unilateral con edema de papila ipsilateral. Qu medida an no es normal. La pauta a seguir ser:
tomaramos en primer lugar?: A) Correccin quirrgica de la endotropa.
A) TAC. B) Continuar las oclusiones sobre ojo con mejor visin. C)
B) VSG. Continuar las oclusiones sobre ojo con peor visin. D) No ocluir
C) Ingreso hospitalario paratratamiento antibitico iv. D) ms y pasar a otro tratamiento.
Observacin. E) Observacin.
E) Radiografa centrada en el agujero ptico.
60. Varn de 70 aos que presenta prdida visual progresiva
54. Mujer de 20 aos con ojo rojo bilateral, acompaado de quemosis, unilateral, que precisa cambios sucesivos de correccin ptica
folculos conjuntivales tarsales, adenopata preauricular, sin prdida mipica. La causa ms probable es:
de visin. La etiologa ms frecuente ser: A) Degeneracin macular senil. B)
A) Adenovirus. B) Glaucoma crnico simple.
S. aureus. C) Vitritis senil.
C) H. influenzae. D) Coriorretinosis senil. E)
D) Queratitis herptica. E) Catarata nuclear.
Parainfluenzae virus.
61. Varn de 51 aos que sufre sbita prdida visual total e
55. Mujer de 23 aos que presenta prdida brusca de agudeza visual indolora en ojo derecho, aprecindose en el F.O. una retina de color
indolora en ojo derecho, en la exploracin se observa defecto blanquecino con mcula rojo-cereza. El diagnstico ms probable es:
pupilar aferente relativo en ojo derecho, segmento anterior normal y A) Desprendimiento de retina con afectacin macular. B)
F.O. normal. El diagnstico ms probable es: Enfermedad de Tay-Sacks.
A) Neuritis ptica retrobulbar. B) C) Enfermedad de Newman-Pick.
Histeria. D) Obstruccin de arteria central de la retina. E)
C) Compresin quiasmtica por tumor hipofisario. D) Obstruccin de vena central de la retina.
Compresin del globo ocular por tumor orbitario. E) Glaucoma
agudo de ngulo cerrado. 62. Varn de 10 aos de edad que presenta estrabismo, disminucin
de agudeza visual y leucocoria en ojo derecho, sin malformaciones
56. Varn de 50 aos con ojo rojo y doloroso, midriasis arreactiva y oculares. Qu patologa debemos descartar como ms probable?:
cmara anterior estrecha. Cul de las siguientes respuestas es A) Retinoblastoma.
falsa?: B) Catarata congnita.
A) No sera raro que fuera hipermtrope. C) Vtreo primario hiperplsico persistente. D)
B) Puede presentar cefalea con nuseas y vmitos. Enfermedad de Coats.
C) Evitar tomar la PIO por posible etiologa infecciosa. D) El E) Fibroplasia retrolental.
tratamiento definitivo es lser o ciruga.
E) Puede presentar visin en halos de colores.

[18]
Mg. Dany Colca
RESIDENTADO DE ENFERMERA. Compendio Banco de Preguntas. Lic. Carolina Ayala

63. Paciente de 65 aos con Diabetes mellitus tipo II de 67. Varn de 64 aos, veterinario de profesin, bebedor espordico
10 aos de evolucin con mal control metablico presenta que presenta fiebre, cefalea y artromialgias desde hace 7 das.
disminucin de visin brusca unilateral. La causa ms probable ser: Durante las ltimas 48 horas refiere tos productiva, dolor pleurtico en
A) Isquemia macular. el costado derecho y disnea progresiva. Ha sido tratado con
B) Desprendimiento de retina traccional. C) eritromicina, pese a lo que se encuentra febril y desorientado. Se
Hemorragia vtrea. evidencian estertores crepitantes en la base derecha y una
D) Edema macular. hepatomegalia a 4 cm del reborde costal. En las pruebas
E) Obstruccin de arteria central de la retina. complementarias efectuadas, destaca Hb de
14.8 mg/dL, leucocitosis (17.000/L) con desviacin izquierda, GOT
64. Varn de 35 aos sano de carcter nervioso comienza con (AST): 106, GPT (ALT): 82. En la Rx de trax se aprecia un infiltrado
metamorfopsias y escotoma central unilateral, disminucin de visin intersticial en lbulo inferior derecho. Su diagnstico de presuncin
moderada y patrn angiogrfico en chimenea. El diagnstico ms debe ser:
probable ser: A) Neumona neumoccica.
A) Distrofia viteliforme de Best. B) B) Neumonitis por hipersensibilidad. C)
Retinosis pigmentaria. Fiebre Q.
C) Degeneracin macular ligada a la edad. D) D) Legionella.
Enfermedad de Coats. E) Tuberculosis.
E) Coriorretinopata central serosa.
68. Varn de 37 aos que viene presentando durante los tres
65. Una mujer de 35 aos, no fumadora, atleta de fondo y sin ltimos meses astenia, esputos hemoptoicos y disnea progresiva
antecedentes personales de inters refiere en los ltimos 3 meses hasta hacerse de mnimos esfuerzos, con intolerancia al ejercicio.
una menor respuesta al ejercicio habitual, con disnea de moderados Salvo una TA de 90/60, los datos exploratorios son anodinos. Sin
esfuerzos. Tos no productiva pero niega fiebre. En una ocasin embargo, los estudios complementarios nos sorprenden: pH: 7.37,
reciente present la emisin de varios esputos hemoptoicos, que no PaO2: 62, PaCO2: 37, HCO3: 27. Hb: 9.2 y creatinina de 2.3. La
se han vuelto a repetir. La Rx de trax muestra un patrn reticular fino Rx de trax muestra infiltrados difusos parahiliares bilaterales. Ante
difuso bilateral y un mnimo derrame pleural derecho. En las pruebas los hallazgos reseados, se aade la peticin de un sedimento de
funcionales llama la atencin el incremento de los volmenes orina, que muestra microhematuria y proteinuria. De las siguientes,
pulmonares. De los siguientes, cul le parece el diagnstico ms qu prueba diagnstica le parece prioritaria en la evaluacin del
probable?: enfermo?:
A) Lupus eritematoso sistmico. B) A) Test de difusin (DLCO).
Tuberculosis. B) Examen citolgico (esputo o lavado broncoalveolar). C)
C) Sarcoidosis. Anticuerpos anti membrana basal glomerular, c- ANCA.
D) Linfangioleiomatosis. D) Biopsia pulmonar. E)
E) Enfermedad de Hamman-Rich. Biopsia renal.

66. Varn de 50 aos, no fumador, que refiere disnea de moderados 69. En el caso anterior, la capacidad de difusin del CO est
esfuerzos de unos 4 meses de evolucin con tos no productiva. aumentada y el ttulo de anticuerpos antimembrana basal glomerular
Ha recibido tratamiento con diurticos de asa (furosemida) en el es de 1:128. Cul le parece el diagnstico ms probable?:
ltimo mes tras realizarse una Rx de trax (que no aporta). Acude a A) Granulomatosis de Wegener. B)
urgencias por incremento de la disnea y expectoracin de esputos Sndrome de Goodpasture.
claros. Refiere febrcula de predominio vespertino y prdida de unos C) Tuberculosis.
6 Kg de peso. Presenta hipoxemia con hipocapnia (insuficiencia D) Tromboembolismo pulmonar.
respiratoria parcial) y la Rx de trax muestra densidades difusas E) Granulomatosis de Churg-Strauss.
bilaterales, confluentes, mal definidas de predominio parahiliar con un
ndice cardiotorcico en el lmite de la normalidad. FVC: 65%, FEV1: 70. Mujer de 78 aos que en el curso de un postoperatorio por fractura
70%, FEV1/FVC: 75%. DLCO: 60%. de cadera comienza con un cuadro brusco de disnea y febrcula.
A) Insuficiencia cardiaca. Edema agudo de pulmn. B) Exploracin: taquipnea a 30 r.p.m., taquicardia a 130 l.p.m., refuerzo
Enfermedad de Hamman-Rich. del segundo tono, abolicin del murmullo vesicular en base de pulmn
C) Hemorragia pulmonar. D) derecho y extremidades sin edemas, no dolorosas, sin signos
Neumona por CMV. flogticos. Complementarios: GAB: pH: 7.52, PaO2: 56, PaCO2: 30,
E) Carcinoma bronquioalveolar. HCO3: 25. 13.000 leucocitos con desviacin izquierda.
Rx de trax: pinzamiento del seno costodiafragmtico derecho.
ECG: Taquicardia sinusal con bloqueo

[19]
Mg. Dany Colca
RESIDENTADO DE ENFERMERA. Compendio Banco de Preguntas. Lic. Carolina Ayala

incompleto de rama derecha. Seale la actitud ms adecuada: 74. Un varn de 30 aos, fumador de 20 cigarrillos/da desde los
A) Diurticos. 20 a los 25 aos y ex fumador desde entonces, presenta,
B) Toracocentesis. C) en un reconocimiento laboral, un ndulo pulmonar solitario
Antibiticos. (NPS) de unos 2 cm. de diametro en la periferia del LSD. La Rx
D) Corticoides. E) de trax muestra un mediastino normal y no permite
Heparina. identificar calcificaciones en el NPS. El paciente se encuentra
asintomtico y niega la posibilidad de recuperar radiografas
71. Mujer de 37 aos, fumadora, que consulta por presentar durante anteriores antes de 6 meses (por cambio de domicilio). Qu actitud
el ltimo mes fiebre, malestar general, artralgias y una erupcin mantendra ante este enfermo? A) Informar de la baja probabilidad
cutnea dolorosa en ambas piernas. Complementarios. Rx de trax: de malignidad y Rx de trax en 3 meses.
adenopatas hiliares bilaterales, sin afectacin del parnquima B) Fibrobroncoscopia.
pulmonar. Mantoux negativo. Se realiz una FBC con lavado C) Realizacin de una TAC torcica.
broncoalveolar (LBA). Lquido del LBA: 22% de linfocitos con cociente D) Realizacin preferente de una PAAF con control de
CD4/CD8 de 5.2. El diagnstico ms probable es: TAC.
A) Tuberculosis. B) E) Insistir en la recuperacin de las Rx previas y nueva cita en la
Sarcoidosis. C) consulta entonces (6 meses).
Linfoma.
D) Cncer de pulmn . E) 75. En el caso anterior, la TAC no muestra nuevos datos (confirma la
Asbestosis. ausencia de calcificaciones, no adenopatas ni afectacin
mediastnica y no existe afectacin pleural). La familia del enfermo
72. Varn de 47 aos que acude a urgencias por fiebre, tos, artralgias ha localizado las Rx de trax previas (14 meses antes) en la que se
y rinorrea purulenta con ulceraciones de la mucosa nasal de dos identifica el mismo NPS con un dimetro de 1,4 cm. Cul sera la
semanas de evolucin. Inici tratamiento antibitico 7 das antes, al actitud ms adecuada?
ser diagnosticado por su mdico de cabecera de sinisitis A) Actitud expectante y repetir pruebas de imagen en 2 meses.
(opacificacin de ambos senos maxilares), sin obtener una mejora B) Fibrobroncoscopia con citologa en las muestras obtenidas.
clnica. La Rx de trax presenta mltiples ndulos pulmonares C) Fibrobroncoscopia y biopsia transbronquial (BTB). D) PAAF
bilaterales, algunos de ellos cavitados. En los anlisis efectuados con control de TAC.
destaca un sedimento de orina con 8 hemates por campo con E) Toracotoma.
algn cilindro eritrocitario. La biopsia de la mucosa nasal mostr
inflamacin granulomatosa con necrosis. El diagnstico ms probable 76. En caso de que la PAAF obtenga material suficiente y el resultado
es: sea de malignidad, qu tipo histolgico le parece el ms probable?
A) Granulomatosis de Wegener. A) Ca. epidermoide. B)
B) Granulomatosis de Churg-Strauss. Adenocarcinoma.
C) Cncer de cavum con metstasis pulmonares. D) C) Ca clulas pequeas (CCP). D) Ca.
Granulomatosis linfomatoide. bronquioalveolar.
E) Tuberculosis. E) Carcinoide.

73. Un grave problema de las unidades de cuidados intensivos son 77. Paciente de 36 aos con amenorrea de 10 semanas. Tiene un
las infecciones (neumonas y sepsis) por grmenes gramnegativos antecedente de infertilidad por factor tubrico. Refiere episodios de
multirresistentes (pseudomonas, serratias, citrobacter, morganella, dolor clico hipogstrico desde hace aproximadamente un mes. El
acinetobacter, etc.). Ya se han identificado previamente cepas test de embarazo en orina es positivo.
multirresistentes en nuestra UCI. A falta de un antibiograma, cul La prueba diagnstica que solicita a continuacin es: A)
sera el tratamiento emprico de eleccin? Amniocentesis precoz.
A) Ceftazidima, amicamicina y vancomicina. B) B) Ecografa.
Ceftriaxona y tobramicina. C) Laparoscopia.
C) Imipenem o ciprofloxacino. D) D) Triple screening.
Imipenem y amikamicina. E) Biopsia de vellosidades corinicas.
E) Esperar hasta los resultados del antibiograma.

[20]
Mg. Dany Colca
RESIDENTADO DE ENFERMERA. Compendio Banco de Preguntas. Lic. Carolina Ayala

78. Gestante de 16 semanas sin antecedentes de inters que D) Placenta previa.


presenta los siguientes resultados en la analtica de triple screening: E) Prdida de lquido amnitico hemtico.
alfafetoprotena 0,3 MM (disminuido), betahcg 1,7 MM (normal),
riesgo estimado de T21 1/43. A continuacin se le realiza: 83. Tercigesta isoinmunizada que presenta test de Coombs indirecto
A) Ecografa. de 1/10 en semana 30 de embarazo. Se realiza un amniocentesis en
B) Amniocentesis. que se determina la madurez fetal (se confirma la presencia de
C) Biopsia de vellosidades corinicas. D) fosfatidil glicerol en lquido amnitico) y el nomograma de Liley, que
Funiculocentesis. se encuentra en la zona II. La actitud indicada es:
E) Fetoscopia. A) Nueva amniocentesis en una semana.
B) Seguimiento ecogrfico con la paciente hospitalizada. C)
79. Gestante de 26 semanas que consulta por fiebre de Administracin de corticoides para inducir la madurez fetal.
39 C y dolor lumbar unilateral. La analtica de sangre presenta D) Extraccin fetal.
16.000 leucocitos y desviacin izquierda. El tratamiento indicado es: E) Administracin endovenosa materna de IgG contra
A) Abundante ingesta de lquidos. Ac anti D.
B) Analgsicos orales y abundante ingesta de lquidos. C)
Analgsicos endovenosos y forzar diuresis. 84. Gestante de 33 semanas que consulta por dinmica uterina,
D) Antibiticos orales y reposo domiciliario. disminucin de movimientos fetales y febrcula. Al ingreso presenta
E) Antibiticos endovenosos intrahospitalarios. una analtica de sangre con leucocitosis moderada y el resto de los
parmetros normales. En la
80. Gestante de 30 semanas con aumento excesivo de peso ecografa se observa un oligoamnios y un perfil biofsico fetal de 9. El
(ganancia de 18 kg. hasta la actualidad) a la que se realiza un test tratamiento consiste en:
de OSullivan que resulta patolgico. La actitud mdica ha de ser a A) Antibiticos endovenosos. B)
continuacin: Antibiticos intracavitarios. C)
A) Vigilancia fetal estricta, con registros semanales de la frecuencia Induccin del parto.
cardaca fetal. D) Cultivo del lquido amnitico y tratamiento segn
B) Controles de glucemia capilar (BMtest) en desayuno, comida y antibiograma.
cena. E) Administracin de corticoides para favorecer la madurez fetal e
C) Ecografas seriadas para diagnosticar a la mayor induccin del parto en semana 36.
brevedad posible un hidramnios o macrosoma fetal.
D) Confirmar el diagnstico mediante una prueba de tolerancia 85. Gestante de embarazo gemelar que presenta dinmica
oral a la glucosa. espontnea en semana 36. Ambos gemelos se encuentran en
E) Tratar con insulina rpida segn los resultados del presentacin ceflica. El parto del primero se produce sin
test de OSullivan. complicaciones, pero el segundo se encuentra en posicin occipito
iliaca derecha transversa al cabo de 25 minutos desde el nacimiento
81. Gestante de 9 semanas que consulta por metrorragia del primero, sin progresar desde un III plano de Hodge. Para finalizar
menor que una regla y dolor abdominal. Todava no ha acudido a el parto est indicada la realizacin de:
ninguna visita de control por su toclogo. En la exploracin se obseva A) Vacuum. B)
un tero de aproximadamente 8 semanas de gestacin, abdomen Frceps. C)
blando y depresible y crvix permeable a un dedo. Poco despus la Cesarea.
paciente empieza a sangrar abundantemente, mucho ms que una D) Maniobra de Kristeller.
regla. El tratamiento indicado es: E) Cualquiera de las anteriores segn el estado fetal.
A) Ingreso y observacin. B)
Laparoscopia. 86. Primigesta de 36 aos que presenta un aumento de la tensin
C) Legrado. diastlica de 30 mmHg respecto a tomas iniciales, y albuminuria con
D) Laparotoma. edemas generalizados. Actualmente se encuentra en la 34 semana
E) Venoclisis de oxitocina. de embarazo. Sbitamente inicia un cuadro de cefalea y transtornos
visuales. Vd. le inicia tratamiento mdico con:
82. Primigesta de 27 aos, sin antecedentes mdicos de inters, que A) Nifedipina.
consulta por metrorragia insidiosa y recurrente en semana 36 de B) Dihidralacina. C)
embarazo. No presenta dolor abdominal, el tero est relajado y el Alfametildopa.
latido fetal se escucha vigoroso. D) Sulfato de magnesio. E)
El diagnstico ms probable ser: A) Labetalol.
Vasa praevia.
B) Desprendimiento prematuro de placenta.
C) Expulsin del tapn mucoso.

[21]
Mg. Dany Colca
RESIDENTADO DE ENFERMERA. Compendio Banco de Preguntas. Lic. Carolina Ayala

87. En la paciente del caso anterior el estudio fetal descubre un no estaban. Bioqumicamente presenta hiperglucemia de 198
feto afecto de crecimiento intrauterino retardado, en el que la relacin g./dl. Probablemente nos encontramos ante:
entre el rea ceflica y el rea abdominal es mayor de uno. El feto A) Efecto secundario de la ceftacidima. B)
se encuentra en situacin ceflica. Evolucin natural del proceso.
La conducta obsttrica adecuada es: C) Situacin de shock sptico.
A) Expectante, con parto vaginal como via de eleccin. D) Probable asociacin de una hemopata. E)
B) Controles de bienestar fetal (perfil biofsico) Cetoacidosis diabtica.
semanales hasta la semana 40.
C) Valoracin de la madurez fetal e induccin del parto, si el feto es 92. Un varn de 27 aos consulta por presenta en la regin
maduro. balanoprepucial una lcera de 1 cm. de tamao de borde indurado,
D) Cesrea inmediata. no doloroso y que secreta serosidad. Presenta adicionalmente
E) Evaluacin de la funcionalidad placentaria mediante ecografa adenopatas inguinales bilaterales y fiebre. Estaramos obligados a
Doppler. solicitarle en el estudio:
A) Aglutinaciones a salmonella. B)
88. Purpera que acaba de parir mediante parto eutcico un varn Aglutinaciones a brucella.
de 3,450 kg. a los 30 minutos no ha alumbrado, por lo que se realiza C) Serologa de VIH. D)
una maniobra de Cred para extraer la placenta. Transcurridos unos Serologa lutica.
minutos presenta un cuadro de disnea y hemorragia profusa. El tero E) Serologa de micoplasma.
est bien contrado, pero el sangrado no cesa. El diagnstico ms
probable es: 93. Una paciente de 19 aos ha sido diagnosticada de mononucleosis
A) Ruptura uterina. infecciosa, confirmada mediante Paul- Bunnel. Ha sido tratada con
B) Desgarro de crvix. C) paracetamol a dosis de 2,5 g./da. A los cinco das del diagnstico
Hipotona uterina. comienza de forma brusca con un cuadro de abdomen agudo y
D) Retencin de restos placentarios. shock. El proceso ms probable es:
E) Coagulacin intravascular diseminada. A) Rotura espontnea del bazo.
B) Hepatitis por virus de Epstein-Barr.
89. Un paciente de 26 aos de edad, adicto a drogas por va C) Complicacin infecciosa intraabdominal. D)
parenteral, consulta por malestar general, fiebre de 39con tiritona, Complicacin del tratamiento.
escalofros y dolor y tumefaccin en rodilla derecha. En la exploracin E) Otra cualquier causa de abdomen agudo: p. ej. apendicitis
llama la atencin un soplo cardco panfocal que previamente no aguda.
estaba en la historia del enfermo y artritis de rodilla derecha. No se
pudo realizar artrocentesis diagnstica. La cobertura emprica 94. Un lactante de 7 meses de edad, con Tetraloga de Fallot
antibitica ms segura sera: intervenida, desarrolla un cuadro de dificultad respiratoria, sibilancias
A) Vancomicina. y tos. No se termometra fiebre. Adems de las medidas de soporte,
B) Vancomicina y Gentamicina. C) el tratamiento de eleccin sera:
Ciprofloxacina. A) Reintervencin quirrgica de su cardiopata. B)
D) Eritromicina y cefuloxima. E) Esteroides a dosis plenas.
Ceftriaxona. C) Ribavirina en aerosol.
D) Eritromicina intravenosa. E)
90. Un joven de 18 aos sin antecedentes de inters, consulta por Ceftriaxona intravenosa.
tos, fiebre y otalgia. En la Rx de trax presenta infiltrado
intersticial derecho. La exploracin ORL demuestra miringitis 95. Una paciente de 49 aos consulta en un Servicio de Urgencias
ampollosa. Iniciaramos tratamiento con: por dolor, enrojecimiento y tumefaccin de la parte distal de su
A) Ceftriaxona. B) miembro inferior derecho. Por sospecha de trombosis venosa
Vancomicina. C) profunda se le realiza flebografa que resulta ser negativa. La actitud
Norfloxacina. teraputica a seguir es:
D) Isoniazida, rifampicina y pirazinamida. E) A) Tratamiento antiinflamatorio.
Eritromicina. B) Heparina de bajo peso molecular.
C) Cobertura emprica antibitica con Oxacilina. D)
91. Un paciente al que le detecta una infeccin urinaria por Reposo del miembro sin ms.
Pseudomona inicia tratamiento con ceftacidima. Tras 2 das de E) Repetir flebografa pasadas 48 horas.
tratamiento el paciente comienza a encontrarse peor, la fiebre
aumenta hasta los 39 y desarrolla hipotensin. En el hemograma
destaca leucopenia e importante trombopenia que previamente

[22]
Mg. Dany Colca
RESIDENTADO DE ENFERMERA. Compendio Banco de Preguntas. Lic. Carolina Ayala

96. Un adicto a drogas por va parenteral consulta por tumoracin


fluctuante y con signos inflamatorios en la flexura del codo donde ha
realizado inyecciones intravenosas. Se debe realizar:
A) Tratamiento quirrgico de drenaje. B)
Cobertura con ceftriaxona.
C) Cobertura con vancomicina. D)
Cobertura con Oxacilina.
E) Medidas antiinflamatorias.

97. Una pareja de turistas que han regresado a Lima, luego de estar
en la Selva Central del Per, comienzan a presentar deposiciones en
cantidad abundante de caractersticas lquidas, similares al agua, en
nmero de
20-25 al da. No se acompaa de dolor abdominal ni fiebre. El cuadro
obliga a una rehidratacin y sta se consigue con dificultad dada la
gran prdida de agua y electrlitos. El cuadro clnico ms probable es:
A) Gastroenteritis por Salmonella. B)
Disentera bacilar.
C) Disentera amebiana. D)
Clera.
E) Gastroenteritis viral.

98. Un enfermo leucmico, muy inmunodeprimido, desarrolla tras un


tratamiento antibacteriano de amplio espectro, cuadro importante de
insuficiencia respiratoria y fiebre. En la Rx. de trax se aprecia una
masa densa, cubierta por un menisco delgado de aire en el interior de
una cavidad. El diagnstico ms probable es:
A) Tuberculosis.
B) Neumona bacteriana.
C) Masa tumoral sobreinfectada. D)
Neumona por cndidas.
E) Aspergiloma.

99. Un nio de 3 aos desarrolla un cuadro de infeccin respiratoria


de vas altas con fiebre elevada. Posteriormente aparece tos
paroxstica con gallo inspiratorio. El tratamiento de eleccin sera:
A) Ceftriaxona. B)
Cefonicid.
C) Cefalotina. D)
Eritromicina.
E) Amoxicilina-clavulnico.

100. Un paciente de 56 aos acude a su mdico de cabecera por


presentar en el ltimo mes fiebre diaria, con una distribucin de dos
picos, matutino y vespertino. A la exploracin llama la atencin una
hepatoesplenomegalia muy importante. No se objetivan adenopatas
a ningn nivel. El hemograma muestra pancitopenia. La prueba
diagnstica que se debera realizar sera:
A) Estudio de mdula sea. B)
Ecografa abdominal.
C) Marcadores tumorales. D)
Biopsia heptica.
E) Marcadores de hepatitis.

[23]
Mg. Dany Colca
RESIDENTADO DE ENFERMERA. Compendio Banco de Preguntas. Lic. Carolina Ayala

CLAVE DE RESPUESTAS TEMA A

PREGUNTA N 1 RESPUESTA: D
Se trata de un caso clnico, laboratorial y radiolgico tpico de pancreatitis aguda. Luego de iniciar e l tratamiento con dieta absoluta, fluidoterapia
y analgesia. Es beneficioso el tratamiento con Carbapenems (imipenem) por tratarse de pancreatitis Baltazar E (Coleccin), esto ha demostrado ser
capaz de disminuir la incidencia de sepsis y de mortalidad (respuesta D correcta).

PREGUNTA N 2 RESPUESTA: D
Nos preguntan por el principal factor responsable de la no cicatrizacin. Las opciones A, B y C son causas de gastritis por s i mismas y los hbitos
dietticos no se han relacionado con la patogenia de la lcera. El tabaco aumenta la incidencia de la lcera duodenal, empeora la cicatrizacin de
las lceras (opcin D correcta) e incrementa el riesgo de complicaciones y la necesidad de cirug a. Los mecanismos por los que acta son:
aumento del vaciamiento gstrico, disminucin de la secrecin pancrtica de bicarbonato, alteracin del flujo sanguneo o disminucin de la sntesis
de prostaglandinas.

PREGUNTA N 3 RESPUESTA: D
El tratamiento debe comenzarse si la cifra de PMN del lquido es superior a 250/mm3. Los frmacos ms utilizados de forma emprica son las
cefalosporinas de tercera generacin. La duracin del tratamiento es de 7 a 10 das, existiendo trabajos que respalda n los tratamientos de 5 das
como eficaces (respuesta D falsa).

PREGUNTA N 4 RESPUESTA: D
Ante un paciente anciano varn con anemia ferropnica crnica debemos sospechar un cncer de colon, aqu adems nos dan el da to de la sangre
oculta en heces, que es el test de screening en mayores de 50 aos. La actuacin es la misma, realizar una colonoscopia completa (puesto slo el
50% de los cnceres estn al alcance del sigmoidoscopio) para buscar la lesin causante de la hemorr agia crnica que le hace al paciente perder
hierro. Esta prueba es el mtodo de diagnstico ms sensible y siempre de be hacerse ante la sospecha de un cncer de colon.
[24]
Mg. Dany Colca
RESIDENTADO EN ENFERMERA. Compendio Banco de Preguntas. Lic. Carolina Ayala

PREGUNTA N 5 RESPUESTA: D

Para llegar a la respuesta correcta, hay que conocer la diferencia entre una revisin narrativa y una sistemtica. En la revi sin narrativa, el
autor revisa bibliografa sobre un tema y compara los resultados, pero el resultado obtenido depende en buena medida de la voluntad del revisor,
en funcin de qu artculos se incluyan (respuestas A, B y C incorrectas). En la revisin sistemtic a, los artculos se incluyen o no en funcin
de criterios prefijados; es la aplicacin del mtodo cientfico a la revisin bibliogrfica. El metaanlisis es una revisin sistemtica en la que
adems los datos se analizan estadsticamente, combinando los diferentes artculos; no todas las revisiones sistemticas son metaanlisis
(respuesta E incorrecta, D correcta).

PREGUNTA N 6 RESPUESTA: C
La aleatorizacin simple nos asegura una misma probabilidad de estar en un grupo u otro, con independencia del nmero de paci entes que ya haya
en cada grupo y de las caractersticas de base. La aleatorizacin por bloques nos evita la desigualdad numrica, a costa de dejar predeterminada la
asignacin del ltimo elemento del bloque. La aleatorizacin estratificada busca homogeneizar la distribucin de una variable de inters; para ello, la
muestra se divide en grupos homogneos respecto a la variable, y posteriormente se aleatoriza segn nuestras necesidades (respuesta C correcta).
Cabe recordar que la aleatorizacin no asegura una homogenei dad de los grupos, slo tiende a ella, por eso es necesario en la fase de anlisis
verificar que la aleatorizacin ha sido eficaz.

PREGUNTA N 7 RESPUESTA: B
El anlisis en el cual se compara el coste en unidades monetarias con resultados que se miden en unidades natura les o fsicas o unidades de
efectividad (aos de vida ganados, incidencia prevenida, Incapacidad evitada) se denomina anlisis de COSTE/EFECTIVIDAD (respuesta B correcta).

PREGUNTA N 8 RESPUESTA: B
La mejor forma en epidemiologa para corroborar una hiptesis etiolgica es un estudio experimental (respuesta B correcta), y si no, pues un estudio
analtico observacional; los descriptivos slo generan hiptesis causales. Si nos preguntan de los estudios experimentales, el mejor de ellos es el
ensayo clnico controlado y aleatorizado. Si nos preguntaran de los estudios observacionales, el mejor es el de cohortes prospectivas.

PREGUNTA N 9 RESPUESTA: E
Son situaciones asociadas a mayor riesgo de cncer colorrectal: endocarditis por S. bovis (se asocia , pero no es causa),
ureterosigmoidostoma, tabaquismo de larga evolucin, obesidad, dieta rica en grasas e hipercalrica, enfermedad inflamatoria intestinal. Por
lo tanto la opcin falsa es la alternativa E. De hecho puede que el consumo de aspirina disminuya el riesgo de cncer de colon.

PREGUNTA N 10 RESPUESTA: B
El ensayo clnico es la herramienta ms potente para obtener conclusiones. Es un estudio analtico experimental, en el que el paciente, una vez que
firma el consentimiento informado, es asignado a un grupo u otro de tratamiento no por el criterio del experimentador sino por un proceso matemtico
llamado aleatorizacin, que intenta que ambos grupos sean homogneos en cuanto a caractersticas basales (respuesta B falsa). Dichos experimentos
suelen ser ciegos, con enmascaramiento de la medicacin para evitar sesgos de clasificacin, pero eso no es estrictamente indispensable.

PREGUNTA N 11 RESPUESTA: B
Las principales indicaciones de vacunacin antineumoccica son: personas mayores de 65 aos, pacientes con insuficiencia cardaca crnica,
infeccin por VIH avanzada, fstula crnica de LCR, alcohlicos. Es importante recordar tambin su indicacin en pac ientes esplenectomizados.
Por lo tanto en esta pregunta la opcin que no recoge una indicacin aceptada es la B.

PREGUNTA N 12 RESPUESTA: E
Tras la esplenectoma existe una inmunodepresin frente a grmenes encapsulados, por ello ya sabemos que las opciones A, B y C estn
efectivamente indicadas. Por otra parte tambin se administra la vacuna contra la gripe ya que nos encontramos ante un enfermo crnico
con cierta inmunodepresin y adems sabemos que la complicacin ms frecuente de la gripe es la neumona, sob re todo la causada por estos
grmenes encapsulados. Por lo tanto la vacuna que no est indicada es la opcin E, la hepatitis A.

PREGUNTA N 13 RESPUESTA: E
En este caso nos plantean el diagnostico diferencial de la disminucin de la agudeza visual (AV). En primer lugar nos dicen q ue la prdida es
lenta y progresiva, e indolora lo que nos permite descartar el glaucoma agudo y la papilitis (respuestas C y D). Como nos preguntan que es lo ms
probable, descartamos directamente la distrofia corneal ya que no son muy frecuentes (respuesta B). C abe pensar que el error de refraccin, el
paciente lo habra tenido toda su vida, no le va a aparecer a los 65 aos (respuesta A). Por tanto la opcin correcta es la degeneracin macular
asociada a la edad (Alternativa E)

PREGUNTA N 14 RESPUESTA: A
Se plantea una disminucin brusca de la visin, con lo que se descarta el glaucoma crnico simple, la catarata y la atrofia ptica (respuestas C, D y
E) y con la superficie ocular normal, con lo que descartamos el edema corneal (respuesta B). Adems debemo s recordar que la hemorragia vtrea
es una causa frecuente de prdida de agudeza visual en la retinopata diabtica.

PREGUNTA N 15 RESPUESTA: A
[25]
Mg. Dany Colca
RESIDENTADO EN ENFERMERA. Compendio Banco de Preguntas. Lic. Carolina Ayala

PREGUNTA N 5 RESPUESTA: D
En la uvetis anterior aguda la pupila se encuentra en MIOSIS, mientras que en el glaucoma agudo encontramos MIDRIASIS. El re sto de opciones
nos completan la clnica tpica de la uvetis anterior.

[26]
Mg. Dany Colca
RESIDENTADO EN ENFERMERA. Compendio Banco de Preguntas. Lic. Carolina Ayala
PREGUNTA N 16 RESPUESTA: D
En este caso nos estn preguntando por el tratamiento de una obstruccin intestinal por un cncer en fase terminal, es decir, nos estn
preguntando por un tratamiento paliativo y no por uno curativo. De todas las opciones, la nica que cumple estas caractersticas es la opcin D. La
morfina es un opiceo de referencia para el tratamiento del dolor del paciente terminal, debido a su gran potencia analgsica y su alta afinidad y
actividad por los receptores mu, careciendo, prcticamente de techo analgsico. La buscapina (escopolamina) es una anticolinrgico que se
utiliza como espasmoltico con la intencin de disminuir el dolor producido por la contraccin refleja de la musculatura lisa del intestino que
aparece en la obstruccin intestinal. El haloperidol es un neurolptico que acta bloqueando el sistema dopaminrgico y que se usa,
fundamentalmente, por su efecto sedante.

PREGUNTA N 17 RESPUESTA: B
Es una pregunta directa en la que hay que conocer que la meperidina (respuesta B) es un agonista opiceo no recomendado en el tratamiento del
dolor crnico de cualquier etiologa por varias razones. Tiene un metabolito con la mitad de potencia analgs ica y una vida media de 15-20 horas,
tras administraciones repetidas puede acumularse y producir hiperexcitabilidad del SNC con alteraciones del humor, temblor, mioclonas e incluso
convulsiones. El resto de opciones son agonistas opiceos que s se usan en le trata miento del dolor crnico, concretamente la metadona, la morfina
y el fentanilo son de alta potencia, mientras que el tramadol es un opiceo dbil.

PREGUNTA N 18 RESPUESTA: D
La traqueotoma es una maniobra quirrgica indicada en casos de obstruccin respiratoria alta (abscesos, tumores... ), pacientes intubados en
los que se espera mantener la ventilacin artificial durante ms de 48-72 horas, y en pacientes con retencin o riesgo de aspiracin de secreciones.
Se ha de realizar siempre entre el 2 y 3 anillos traqueales (respuesta D correc ta) y nunca en el 1 (respuesta C falsa) por el alto riesgo de
estenosis traqueales residuales (complicacin tarda fundamental, la precoz ms fre cuente es la hemorragia). En la membrana cricotiroidea (opcin
A falsa) es donde se realizan las coniotomas, que slo se han de realizar en caso de extrema urgencia en el medio extrahospitalario.

PREGUNTA N 19 RESPUESTA: C
La opcin A es correcta, ya que es ms frecuente en nios (por la hipertrofia adenoidea), especialmente si tienen malformacio nes velopalatinas, as
como en el sndrome de Down. La opcin B es correcta, ya que toda otitis media secretora unilateral persistente en el adulto obliga a descartar
un cncer de cavum (el 25 % de estos cnceres tienen esta forma de presentacin). La opcin D pu ede plantearnos algunas dudas, pero est
descrito que entre 2/3 y 3/4 de los cultivos de muestras obtenidas por miringocentesis dan positivo. La opcin E tambin es correcta, ya que el
aspecto tpico en la otoscopia es un tmpano ntegro, retrado y opaco, con niveles hidroareos y burbujas en la caja. La opcin C es la falsa, ya
que el dato epidemiolgico que nos presenta es el caractersti co de la otitis del nadador (otitis externa difusa bacteriana).

PREGUNTA N 20 RESPUESTA: B
Esta situacin requiere sin duda alguna una actitud teraputica inmediata, por lo que descartamos la opcin 1 (actitud expectante) y las
opciones D y E, en las que tanto el hierro IV como la EPO requeriran das para comenzar a mejorar la cifra de hemoglobina. La respuesta correcta
es la B (transfundir hemates) porque en el enunciado no se nos dice que la paciente tenga hipotensin o shock hipovolmico por hemorragia,
en cuyo caso s estara indicado transfundir sangre total (opcin C).

PREGUNTA N 21 RESPUESTA: A
Los cuerpos de Howell-Jolly son inclusiones redondeadas, densas y en general nicas, debidas a fragmentos de cromosomas procedentes de mitosis
eritroblsticas anmalas. Se observan en pacientes esplenectomizados, en el hiposplenismo, en el satur nismo y en las anemias megaloblsticas y
refractarias. Por tanto la respuesta correcta es la A, asplenia. Para el mir se deben conocer otros datos morfolgicos del estudio de extensin de la
sangre perifrica. En el dficit de G6PDH son los cuerpos de Heinz, en los procesos hemolticos aparecen los esferocitos.

PREGUNTA N 22 RESPUESTA: E
La leucemia mieloide crnica est claramente relacionada con un marcador citogentico, el cromosoma Philadelphia, que aparece en cerca del 95%
de los casos. Dicho cromosoma consiste en una translocacin del material gentico entre los cromosomas 9 y 22. Dicha translocacin cromosmica
da lugar a la unin del oncogn abl del cromosoma 9 con el oncogn bcr del 22, originando un hbrido anormal bcr/abl, que es el causante de la
enfermedad. De todos modos el cromosoma Philadelphia no es exclusivo de las clulas precursoras de la serie blanca, sino que tambin de la
roja y megacariocitos, aunque es exclusivo de las clulas hematolgica s. Aunque el cromosoma Philadelphia aparece en el 95%, lo caracterstico
es la traslocacin, puesto que es el causante de la sntesis anmala y por tanto la respuesta correcta es la E.

PREGUNTA N 23 RESPUESTA: A
En este paciente, el diagnstico ms factible es el herpes genital, muy probablemente una primoinfeccin, dada la r epercusin sistmica de
la enfermedad. En las recurrencias, no suele haber fiebre ni malestar general. La presencia de las lesiones vesi culosas es muy sugerente de esta
infeccin. Estas vesculas suelen ulcerarse ms tarde; de hecho, la causa infecciosa ms frecuente de lceras genitales es el virus herpes simple.
La opcin B es claramente falsa, pues de un secundarismo lutico no cabra esper ar la aparicin de lesiones de este tipo. Debes asociar a la
sfilis secundaria manifestaciones como la rosola, la lengua en pradera segada y los condilomas planos. La primoinfeccin por VIH puede ser
asintomtica, pero suele cursar como un sndrome mononuclesico, con un rash maculopapular (opcin C falsa). El citomegalovirus puede transmitirse
por va sexual, pero no se incluye en el diagnstico diferencial de los chancros (opcin D falsa). Por ltimo, el condiloma acuminado es una infeccin
por papilomavirus que produ ce unas lesiones de aspecto sobreelevado y verrucoso (opcin E falsa).

[27]
Mg. Dany Colca
RESIDENTADO EN ENFERMERA. Compendio Banco de Preguntas. Lic. Carolina Ayala

PREGUNTA N 24 RESPUESTA: D
Las infecciones bacterianas son una importante causa de muerte en pacientes con infeccin por VIH. El neumococo y el H. influ enzae representan
dos de las infecciones bacterianas ms frecuentes en este tipo de pacientes (respuesta C correcta). Los enfermos con infeccin por VIH muestran
un incremento de seis veces en la incidencia de neumona neumoccica, y de 100 veces en la de bacteriemia por neumococo (respuestas A y B
correctas). Todos los pacientes con infeccin por VIH deben recibir la vacuna antineumoccica desde el momento en que se diagnostica; no
hay que esperar a que tengan un grado determinado de inmunodepresin (respuesta D falsa). Los pacientes VIH positivos, cuando sobrepasan
determinado nivel de inmunodepresin, comienzan a sufrir infecciones por microrganismos oportunistas. Debes conocer muy bien a partir de qu
nivel de linfocitos CD4 comienzan a ser ms probables los distintos grmenes, lo que te facilitar la resolucin de muchos casos clnicos. Por ejempl
o, en el caso del criptococo, el nivel de CD4 suele oscilar en torno a 100 o menos. Sin embargo, otras infecciones no precisan un grado tan intenso
de inmunodepresin, pudiendo aparecer con el sistema inmunolgico relativamente intacto, como es el caso del neumococ o (respuesta E correcta).
Aprovechamos la ocasin para recordarte las complicaciones clnicas que definen la categora C en la infeccin por VIH, entre las cuales se encuentra
el presentar dos o ms episodios de neumona en un ao.

PREGUNTA N 25 RESPUESTA: C
En muchas ocasiones, pueden interrelacionarse, por la especial susceptibilidad de los pacientes VIH positivos a ser infectados por micobacterias.
Este tipo de enfermos suelen padecer formas ms graves de tuberculosis que los inmunocompetentes, debido precisamente a la incompetencia
de su inmunidad celular (opcin A falsa, C correcta). No es necesario un grado de inmunosupresin muy severo para que el paciente VIH sufra una
tuberculosis (respuesta B falsa). El lmite de 50 CD4/mL es donde suelen estar los pacientes con SIDA que son infectados por micobacterias
atpicas. Desde el punto de vista del diagnstico, los mtodos microbiolgicos para el diagnstico de tuberculosis no tienen por qu tener un
menor rendimiento que en pacientes inmunocompetentes. De hecho, el nmero de bacilos en las muestras puede incluso ser mayor en el paciente
con SIDA, por el deterioro de sus defensas (opcin D falsa). En cuanto al tratamiento, la respuesta al mismo de los pacientes VIH positivos es la
misma que en inmunocompetentes, es decir, suele ser satisfactorio (opcin E falsa).

PREGUNTA N 26 RESPUESTA: C
Debes recordar que es una enfermedad producida por toxinas del Staphylococcus aureus, la clnica que produce y su asociacin al uso de
tampones intravaginales. Centrndonos en la pregunta, la respuesta falsa es la C, puesto que este cuadro se debe a la diseminacin hematgena
de toxinas (TSST-1), no de la bacteria en s. Clnicamente, el cuadro tpico consiste en la aparicin de fiebre, exantema con posterior descamacin
(tpicamente palmoplantar), hipotensin y afectacin de al menos tres sistemas orgnicos (digestivo, muscular, renal, heptico, trombopenia o sistema
nervioso), en ausencia de otro diagnstico alternativo. El trata miento debe hacerse con antibiticos antiestafiloccicos (Oxacilina o, si es resistente,
vancomicina) y mantenimiento hemodinmico del paciente.

PREGUNTA N 27 RESPUESTA: E
El diagnstico diferencial entre una ITU alta y una baja es fundamentalmente clnico. Una cistitis aguda suele cursar con ten esmo vesical,
aumento de la frecuencia miccional y escozor al orinar, a veces con hematuria terminal. Puede haber dolor hipogstrico y, ms raramente, febrcula.
En cambio, la pielonefritis se presenta como un cuadro febril, con hipersensibilidad de la fosa lumbar, fiebre, nuseas, vmitos y puo percusin
positiva en la exploracin fsica. De las cuatro primeras opciones que nos presenta la pregu nta, no podramos emplear ninguna para distinguir un
cuadro de otro. La diferencia fundamental es la opcin E, pues en una cistitis es rarsima la presencia de fiebre, mientras que en una pielonefritis
aguda sera excepcional que no apareciese.

PREGUNTA N 28 RESPUESTA: A
Existen dos tipos de pruebas serolgicas, treponmicas (FTA-Abs, TPHA) y no treponmicas. (VDRL, RPR). Las primeras en positivizarse
son las treponmicas, persistiendo positivas de por vida. En cambio, en las no treponmicas puede observarse un a disminucin del ttulo de
anticuerpos cuando se instaura un tratamiento correcto. En el caso que nos presentan, para cons iderar que la lesin ulcerada es consecuencia de
una sfilis, debera tener la prueba treponmica (FTA-Abs) positiva. Al ser negativo, debes pensar en otra causa. La positividad del VDRL tiene un
valor muy relativo, porque es una prueba poco especfica, pu diendo positivizarse por circunstancias muy distintas a la sfilis (lupus eritematoso,
embarazo, ciertas conectivopatas,...). Por tanto, recuerda en adelante que un paciente con FTA-abs negativo debe hacerte pensar que muy
probablemente no padece (ni ha padecido) una sfilis. (Alternativa A)

PREGUNTA N 29 RESPUESTA: C
Centrndonos en esta pregunta, la respuesta incorrecta es la C. El Ancylostoma duodenale es un helminto que se fija a la pare d intestinal, de
cuya sangre se alimenta, gracias a un gancho con el que se fija a la mucosa. El hecho de ser un helminto y poseer un gancho, le ha valido el carioso
apodo de gusano ganchudo. Dado que se alimenta de sangre, el tipo de anemia que produce es FERROPNICA, no megaloblstica; de ah la
falsedad de la respuesta C. Existe otro helminto que s es capaz de producir anemia megaloblstica, por consumo de vitamina B12, llamado
Diphyllobotrium latum.

PREGUNTA N 30 RESPUESTA: C
El caso que nos plantean es el de un paciente con cefalea y leves signos menngeos, aparte de que es VIH positivo y tiene un lquido cefalorraqudeo
con las caractersticas que ya has visto en el enunciado. De las opciones que nos plantean, hay dos posibilid ades compatibles con el anlisis
del LCR que nos dan como dato, que son la B y la C (Listeria y Criptococo). El elemento que nos hace orientar por la segunda es que se trata de
un paciente VIH positivo con un grado muy importante de inmunosupresin (50 CD4/m L), lo que hace realmente probable la infeccin criptoccica
como causa de su meningitis; por otro lado el marcador clnico de la criptococosis cerebral es la cefalea de das de evolucin (Ms de una
[28]
Mg. Dany Colca
RESIDENTADO EN ENFERMERA. Compendio Banco de Preguntas. Lic. Carolina Ayala

semana en elN
PREGUNTA enunciado).
24 El dato que dan Dde la TAC (lig era atrofia cortical) no debe confundirte. Esto es algo relativamente frecuente en los
RESPUESTA:
pacientes VIH positivos, por efecto del propio VIH.

[29]
Mg. Dany Colca
RESIDENTADO DE ENFERMERA. Compendio Banco de Preguntas. Lic. Carolina Ayala

PREGUNTA N 31 RESPUESTA: C
Esta bacteria puede producir infecciones muy variadas, como farngeas, escarlatina, imptigo... Adems, pueden producirse complicaciones no
supurativas en relacin con ella, como la glomerulonefritis postestreptoccica y la fiebre reumtica. El tratamiento de la faringoamigdalitis por S.
pyogenes admite varias alternativas. En caso de alergia a betalactmicos, los macrlidos oral es son una opcin vlida (respuesta E correcta). La
amoxicilina oral es tambin un tratamiento eficaz (respuesta D correcta). Date cuenta que la respuesta dice amoxicilina, y no amoxicilina-
clavulnico. Si se sabe a ciencia cierta que se trata de S. pyogenes, no es necesario aadir cido clavulnico, porque hasta la fecha no se ha visto
que esta bacteria produzca betalactamasas. La nica penicilina activa va oral es la penicilina V (fenoximetilpenicilina), que es til como tratamiento
si se siguen las pautas que dice la opcin B. Para prolongar la vida media de la penicilina en el organismo, una opcin es unirla a una sal, que puede
ser procana o benzatina. La combinacin penicilina G benzatina permite una nica inyeccin intramuscular como tratamiento de esta
infeccin (opcin A correcta). En cambio, la penicilina G procana tiene una vida media considerablemente inferior, por lo que la opcin C no es
vlida.

PREGUNTA N 32 RESPUESTA: B
Los estreptococos eran la causa ms frecuente de endocarditis infecciosa, con gran diferencia , especialmente S. viridans. Sin embargo,
desde los aos 90 ha existido una disminucin de la endocarditis por estreptococos, coincidiendo con un aumento de l as producidas por
estafilococos, que suponen del 30-50% del total. Dentro de este grupo, el S. aureus es 5-10 veces ms frecuente que el S. epidermidis, que s
deberas tener en cuenta si te preguntan por la endocarditis protsica precoz. La alternativa corre cta es la B.

PREGUNTA N 33 RESPUESTA: B
La opcin menos probable es la B, el TEP, que es una patologa trombtica, puesto que la estreptoquinasa, como cualquier tromboltico,
hace muy improbable que se formen trombos tan slo seis horas despus de haberla administrado. Todo lo contrari o ocurre con la opcin E
(hemorragia cerebral), que es una complicacin conocida de estos frmacos. Las opciones A, C y D son tambin posibles puesto que son
complicaciones mecnicas del IMA que pueden cursar con hipotensin y obnubilacin.

PREGUNTA N 34 RESPUESTA: B
Los antagonistas de los receptores de la angiotensina II tienen un efecto y mecanismo de accin similares a los de los IECA, aunque en vez de
bloquear la produccin de angiotensina II, inhiben competitivamente su unin al receptor AT 1 de la angiotensina II. Las indicaciones, utilidad y
tolerancia son similares a las de los IECA, aunque NO producen tos ni angioedema, al no interferir sobre la degradacin de las bradiquininas (opcin
B correcta). La cininasa II es la enzima encargada de metabolizar las bradiquininas, y su estructura es idntica a la enzima convertidora de
angiotensina, por lo que los IECA tambin la inhiben, aumentando los niveles de bradiquininas. Por todo esto se suelen reservar para los pacientes
con intolerancia a los IECA, sobre los que existen ms est udios que prueban el aumento de supervivencia. Recordamos que ambos grupos
farmacolgicos estn contraindicados en embarazadas, pacientes con hiperpotasemia, estenosis renal bilateral o estenosis de la arteria renal con
rin nico.

PREGUNTA N 35 RESPUESTA: C
El flutter se define electrofisiolgicamente como una macrorreentrada a nivel de aurcula derecha, en la que la activacin auricular se produce de
manera crneo-caudal en la pared lateral de la aurcula derecha, para alcanzar la regin del itsmo cavotricuspdeo y propagarse posteriormente de
manera caudo-craneal a nivel del septum hasta alcanzar el techo de la aurcula derecha. Por lo tanto, la ablacin del istmo cavotricuspdeo obtiene
un porcentaje de xitos cercano al 95% en el flutter istmo -dependiente o comn (el ms frecuente), en el que la reentrada utiliza esta estructura
como parte del circuito (opcin C correcta). No obstante, se puede reali zar tambin la prevencin de recurrencias con los mismos frmacos que
se usan en la FA, por ejemplo la amiodarona (opcin E), que en este caso es menos eficaz que la ablacin con catter y radiofrecuencia.

PREGUNTA N 36 RESPUESTA: B
El caso clnico presenta un paciente con disminucin de conciencia e hipotenso, que tiene una presin de enclavamiento dismin uida (<18 mmHg)
y una presin en la aurcula derecha elevada (o presin venosa yugular). Esta situacin se corresponde con una disfuncin del VD, que es incapaz
de bombear sangre al VI por lo que tendremos dos tipos de sntomas: los derivados del defic iente aporte de sangre a los tejidos (obnubilacin e
hipotensin con bajo gasto cardaco) y los secundarios a la sobrecarga retrgrada de lquidos (aumento PVC). La presencia de lquido en el
pericardio, que ocurre en el taponamiento cardaco, y la obliteracin d e la cavidad pericrdica por fibrosis, en el caso de la constriccin pericrdica;
ocasionan ambas un aumento de la presin pericrdica. Esta presin se transmitir primero a las cmaras derechas, por ser stas las que trabajan
a menor presin, lo que producira la d isfuncin diastlica descrita anteriormente, al dificultar el llenado de los ventrculos. Por lo tanto las opciones
A y E son ciertas. El infarto de ventrculo derecho ocurre hasta en un 30% de los IMA inferiores y cursa como en esta pregunta con clnica de
insuficien cia cardiaca aguda (aumento de PVC y disminucin del gasto cardaco). Es importante destacar que el tratamiento de esta situacin sera
la administracin de lquidos intravenosos y frmacos inotrpicos positivos, dato muy preguntado en el MIR. Por lo tanto la opcin C es cierta.
Respecto al TEP (tromboembolismo pulmonar), se produce un aumento de las resistencias vasculares pulmonares debido a la disminucin del rea
transversal del lecho arterial y a la produccin de sustancias neurohumorales como la serot onina. Este aumento de las resistencias produce
hipertensin pulmonar e insuficiencia derecha aguda, que es la causa ms frecuente de muerte inme diata por TEP. Si el VD se dilata, el tabique
puede abombarse hacia el VI, con la consiguiente restriccin al llenado y aparicin de sntomas de bajo gasto (presencia de pulso paradjico).
Opcin D cierta. Adems en este paciente no habra un estado de deshidratacin ya que la PVC est elevada. Por lo tanto la opcin B es la falsa.

PREGUNTA N 37 RESPUESTA: C

[30]
Mg. Dany Colca
RESIDENTADO DE ENFERMERA. Compendio Banco de Preguntas. Lic. Carolina Ayala

En esta ocasin se trata de una pregunta terica acerca del manejo del fonendoscopio, la cual se responda sabiendo que la campa na detecta mejor
bajas frecuencias y la membrana detecta mejor las altas. De este modo, el 3 tono, el 4 tono y el retumbo de l a estenosis mitral se detectaran
mejor con la campana al ser tonos de baja frecuencia. Opcin C correcta. El tercer ruido se p roduce por un llenado ventricular rpido y aunque
puede ser fisiolgico en personas jvenes, generalmente es indicativo de una insu ficiencia

[31]
Mg. Dany Colca
RESIDENTADO DE ENFERMERA. Compendio Banco de Preguntas. Lic. Carolina Ayala

cardiaca. El cuarto ruido es siempre patolgico y se debe a la contraccin de la aurcula contra un ventrculo con una disten sibilidad reducida. El
chasquido de apertura es un tono diastlico de alta frecuencia que se debe a estenosis de una vlvula aurculo-ventricular, por lo que se oira mejor
con la membrana. El prolapso mitral tiene un click al final de la sstole que se sigue de un soplo de alta frecuencia, piante, por lo tanto tambin se
oira mejor con la membrana. Es importante recordar que el soplo del prolapso igual que el de la miocardiopata hipertrfica se hace ms intenso
cuando disminuye la precarga o la postcarga. Opcin B y D falsas. Para determinar en qu momento se produce el ruido o soplo hay que considerar
la relacin con otros fenmenos del ciclo cardaco como el pulso carotdeo, la PVY o el impulso apical, pero no depende de la parte del fonendoscopio
con la que se ausculte. Opcin A f alsa. En nios pequeos se puede utilizar tanto la membrana como la campana, aunque en muchas ocasiones se
utiliza la campana por razn de tamao. La membrana es ms amplia y abarcara gran parte del trax del nio, por lo que no podramos determinar
en qu foc o estamos auscultando un soplo o ruido en el caso en que ste existiese. Opcin E es falsa.

PREGUNTA N 38 RESPUESTA: E
El delirium tremens se trata de un cuadro producido por la abstinencia al alcohol de forma brusca. Cursa con un sndrome conf usional con
desorientacin, alteraciones perceptivas como ilusiones y alucinaciones de gran int ensidad y vivencia angustiosa, sobre todo visuales (tpicas
las microzoopsias). Tambin se pueden producir delirios, inquietud, agitacin y clnica vegetativa. En el t ratamiento, se usan fundamentalmente las
benzodiacepinas (cloracepato, diacepam) y el clometiazol. (Alternativa E) En las formas leves, puede usarse tambin el tiapride.

PREGUNTA N 39 RESPUESTA: C
Dentro del cuadro de la esquizofrenia, es frecuente la existencia de pseudoalucinaciones o alucinacin psquica falsa, caract erizada por tratarse
de una percepcin sin objeto y sin crtica de la misma (al contrario de la alucinosis en la que el paciente duda o crtica la veracidad o autenticidad de
esta), y que el paciente la sita en su espacio interior, siendo sobre todo de tipo auditivo. Por ejemplo, el paciente oye voces en el interior de su
cabeza que le dan rdenes o le incitan a que haga algo. Si estas ocurrieran en el esp acio exterior, el paciente no las oira dentro de su cabeza,
sino que se acercara a las esquinas o mirara al cielo u ot ro lugar desde donde cree que le estn hablando. Las pareidolias, son un tipo de ilusiones
(deformacin de una percepcin real), que se desencaden an por sugestin. La alternativa correcta es la C.

PREGUNTA N 40 RESPUESTA: B
Entre las alteraciones que se exponen, es el aumento de la amilasa srica la nica determinacin til que apoya el diagnstico de sta enfermedad,
la cual adems estar ms aumentada cuanto mayor sea la incidencia de maniobras purgativas, y que se debe a un aumento de la amilasa tanto
salivar como pancretica. Adems, no olvidar las alteraciones ms tpicas por vmitos de repeticin como son la hipopotasemia, la hipocloremia,
hipomagnesemia y alcalosis metablica. La alternativa correcta es la B.

PREGUNTA N 41 RESPUESTA: A
La clnica descrita tanto por el mal estado general, la fiebre, la distribucin y morfologa del exantema, los perodos tanto de pr dromo como de
evolucin, son los tpicos de una fiebre botonosa. Para que no quede duda, tras exposicin en el campo, se observa un a picadura (tache noir).
El tratamiento ms eficaz es con tetraciclinas o ms preferiblemente con doxiciclina a las dosis indic adas.

PREGUNTA N 42 RESPUESTA: C
En la infeccin por VIH, cuando la inmunidad est seriamente daada (y nuestro paciente con esos antecedentes, la tiene) cualquier lcera, sea cual
sea su forma, localizacin, aspecto, evolucin, es una infeccin por herpes simple mientras no se demuestre lo contrario. El dolor intenso y el
crecimiento geogrfico descartan cualquier otra opcin.

PREGUNTA N 43 RESPUESTA: E
La edad de aparicin de las lesiones en la vejez, la morfologa y distribucin de las mismas as como la microscopia ptica, sealan hacia un
penfigoide ampolloso. No obstante, y ante la rara, pero posible confusin con una e pidermlisis ampollosa adquirida, se ha realizado la nica
prueba que las diferencia que es IFD tras tcnica de separacin de la UDE con sal, el antgeno de la Epide rmlisis ampollosa adquirida slo quedar
en dermis, en el penfigoide ampolloso, quedar bien en techo slo, bien como en nuestro caso, en ambos lados.

PREGUNTA N 44 RESPUESTA: C
La edad de inicio, la morfologa y distribucin de las lesiones apuntan a una Dermatitis Herpetiforme. Ningn dato va en cont ra de ella. La afectacin
intestinal es parcheada por lo que no tiene un significado en contra del diagnstico que sea negativa. Los Ac. antigliadina IgA slo aparecen en un
tercio de los casos, y los Ac. antigliadina IgG, menos especficos pero presentes en ms de la mitad de los enfermos s lo son, lo que afirma el
diagnstico. La dieta est indicada en cualquiera de los casos, porque disminuye la cantidad de sulfonas necesaria para controlar la enfermedad.

PREGUNTA N 45 RESPUESTA: C
La vasculitis se descarta por el tipo de lesiones cutneas. El resto de los cuadros s cursan con lesiones eritematoescamosas, sin embargo: el eczema
numular suele ser un proceso muy crnico, en el herpes circinado (tinea corporis) las lesiones aumentan lentamente tanto en nmero como en
tamao, aunque la pitiriasis rosada podra encajar en el cuadro expuesto, es rara en las edades extremas de la vida, y adems es bastante tpico
que los brotes de psoriasis gutata sean desencadenados en la infancia por infecciones estreptoccicas.

PREGUNTA N 46 RESPUESTA: C
La psoriasis tipo Von Zumbusch es un pustulosis aguda generalizada. En la rosola sifiltica es caracterstica la afectacin de palmas y/o plantas. En
el eczema suelen existir vesculas, erosiones y costras. La lesin descrita es caracterstica de la pitir iasis rosada, la
[32]
Mg. Dany Colca
RESIDENTADO DE ENFERMERA. Compendio Banco de Preguntas. Lic. Carolina Ayala

placa inicial correspondera al medalln herldico, adems en la pitiriasis rosada las lesiones se localizan en tronco y regi n proximal de miembros,
lo cual no tiene por qu suceder en la psoriasis.

PREGUNTA N 47 RESPUESTA: D
Las lesiones peribucales corresponderan a un herpes simple, tras el cual se desencadena un eritema exudativo multiforme con lesiones en diana y
lesiones tipo herpes iris de Bateman. No se trata de un Stevens -Johnson (o eritema multiforme major), pues no hay lesiones en mucosas.

PREGUNTA N 48 RESPUESTA: C
Aunque todas las patologas entran en el grupo de las vasculitis necrotizantes, en ninguna de ellas salvo en la PAN la lesin elemental es el ndulo

PREGUNTA N 49 RESPUESTA: D
Varios procesos como el eczema seborreico o psoriasis pueden causar eritrodermia debido a su extensin. Es caracterstico del sndrome de Sezary
la eritroderma con adenopatas y ms de 10% de clulas de Sezary en sangre perifrica. Se acompaa tambin de alteraciones de pelo y uas. La
parapsoriasis en grandes placas es una forma inicial de micosis fungoide.

PREGUNTA N 50 RESPUESTA: A
La ictiosis es una genodermatosis caracterizada por piel seca y cubierta de escamas. La ictiosis vulgar es la ms frecuente, suele comenzar en la
infancia mejorando con la edad. La ictiosis ligada a X la padecen slo los varones, se asocia a un dficit de esterio de sulfatasa, comienza en la
primera infancia y no respeta las flexuras. La ictiosis laminar aparece ya en el momento del nacimiento (bebe coloidon) y suele acompaarse de
ectropin y alteracin de los anejos. La enfermedad de Darier consiste en pequeas ppulas marrones localizadas en reas seborreicas.

PREGUNTA N 51 RESPUESTA: E
La erupcin polimorfa lumnica es una reaccin de inmunidad retardada frente a un antgeno cutneo que suele presentarse anualmente en primavera
o verano y suelen disminuir las manifestaciones a medida que avanza la estacin. Las reacciones fototxicas aparecen tras la primera
exposicin, en horas, se localizan en reas fotoexpuestas y clinicamente son una quemadura solar. Las reacciones fotoalrgicas son por reaccin
tipo IV y tardan ms de 24 horas en aparecer. Clnicamente es una dermatitis eczematosa que desborda reas expuestas. Hay frmacos que
producen ambos tipos de reacciones.

PREGUNTA N 52 RESPUESTA: D
Pensaramos en lepra si estuviramos en un rea endmica y las mculas tuvieran anestesia. La biopsia, localizacin de las lesiones y la respuesta
al tratamiento son caractersticas de vitligo.

PREGUNTA N 53 RESPUESTA: A
El hecho de que un paciente con garganta estreptoccica no mejore con tratamiento en 48 horas har sospechar la posibilidad de una mononucleosis
infecciosa.

PREGUNTA N 54 RESPUESTA: C
La apendicitis es poco frecuente en la etapa de la lactancia, aunque puede manifestarse por vmitos, afectando principalmente a nios en edad
prescolar y escolar. El resto de las respuestas incluye las causas ms frecuentes de vmitos en la lactancia, unidas a las infecciones por ejemplo
urinarias. Los errores dietticos o psicolgicos figuran como una de las causas ms importantes de vmitos en el lactante que crece, gana peso y se
desarrolla normalmente.

PREGUNTA N 55 RESPUESTA: B
La insuficiencia suprarrenal cursa habitualmente de forma gradual, pero puede presentarse de forma brusca con deterioro rpido, hipotensin,
hipoglucemia, hiponatremia e hiperpotasemia. La recuperacin resulta llamativa tras la administracin de corticoides intravenosos, junto con glucosa
al 5% y suero salino para reponer la volemia y la natremia. Para confirmar el diagnstico deben recogerse niveles de cortisol antes de iniciar el
tratamiento, que estarn disminuidos tanto en sangre como en orina.

PREGUNTA N 56 RESPUESTA: C
Los datos clnicos sugieren un coma etlico. La ingestin de barbitricos se acompaa de pupilas miticas, las anfetaminas provocan pupilas
midriticas, el infarto cerebral y la hemorragia subaracnoidea presentan una exploracin neurolgica anormal, el status convulsivo tendra una
duracin mnima de 30 minutos.

PREGUNTA N 57 RESPUESTA: B
La evaluacin de un nio con prpura y petequias de aparicin aguda debe incluir un recuento plaquetario para descartar una trombopenia. Una cifra
de plaquetas normales sugiere vasculitis de pequeos vasos, alteraciones funcionales plaquetarias o trastorno de la coagulacin. La distribucin
simtrica, lesiones palpables y el recuento plaquetario normal son ms compatibles con prpura de Schonlein Henoch.

PREGUNTA N 58 RESPUESTA: B
La recurrencia de cuadros catarrales sugiere la posibilidad de afectacin tica (Otitis medias serosa), que es la causa ms frecuente de problemas
auditivos en la edad prescolar
[33]
Mg. Dany Colca
RESIDENTADO EN ENFERMERA. Compendio Banco de Preguntas. Lic. Carolina Ayala

PREGUNTA N 59 RESPUESTA: D
Un nio fuera del periodo neonatal que tenga una obstruccin intestinal, sin haber sufrido una operacin previa, y que no tenga una invaginacin, lo
ms probable es que presente un divertculo de Meckel.

PREGUNTA N 60 RESPUESTA: C
La bronquiolitis es un cuadro respiratorio asociado con frecuencia al V.R.S. que se produce en nios menores de dos aos con cuadro catarral previo.
Produce insuficiencia respiratoria que puede evolucionar hacia un cuadro grave. La no existencia de estridor, disfagia, la edad, el ser primer episodio,
y no tener antecedentes previos descarta el resto de los cuadros.

PREGUNTA N 61 RESPUESTA: D
El cuadro descrito se encuadra dentro de la abstinencia por adiccin materna a herona, que asocia al bajo peso otros trastornos:
neurolgicos, digestivos y de comportamiento. Se presenta en el 80 % de los hijos de madres heroinmanas.

PREGUNTA N 62 RESPUESTA: C
El clico del lactante aparece en el primer trimestre de vida. Su etiologa y fisiopatologa son desconocidas. El diagnstico es
fundamentalmente clnico caracterizado por llanto de intensidad variable, de predominio vespertino, irritabilidad, distensin abdominal, flexin de
piernas, y falta de respuesta a estmulos calmantes. El desarrollo sicomotor y somatomtrico son normales.

PREGUNTA N 63 RESPUESTA: C
El Sndrome de West se produce en lactantes entre los 2-6 meses de edad. Ocasiona en ellos prdida de adquisiciones sicomotoras, retraso
sicomotor, y presenta un patrn electroencefalogrfico tpico (Hipsarritmia). Hemos de diferenciarlo de los clicos del lactante (que cursa sin prdida
de conocimiento ni retraso sicomotor) por la similitud entre la crisis de ambos. Tambin hay que diferenciarlo de otros cuadros epilpticos mioclnicos
benignos que no afectan al desarrollo sicomotor.

PREGUNTA N 64 RESPUESTA: D
Todas las entidades citadas pueden producir ictericia persistente a la edad del lactante citado, excepto la ictericia fisiolgica. La ictericia fisiolgica
aparece entre el segundo y tercer da desapareciendo hacia el octavo da, suele ser de escasa intensidad cuya etiologa principal es una inmadurez
del sistema enzimtico heptico (Glucuronil transferasa) junto al aumento de la destruccin de hemates.

PREGUNTA N 65 RESPUESTA: C
La perforacin gstrica dara clnica diferente, con un dolor ms difuso, no slo localizado en epigastrio, y s que nos permitira pasar la SNG. La
estenosis pilrica aguda se da en el contexto de una lcera antral o pilrica y tambin permite pasar la SNG. El sndrome de Boerhaave se asocia
con vmitos bruscos previamente, dolor mediastnico tras la ruptura esofgica y, no obstaculizara la SNG. El tricobezoar no da clnica de obstruccin
brusca, sino crnica, con problemas de vaciamiento gstrico de ms evolucin. El vlvulo se caracteriza por dolor intenso, epigstrico y de aparicin
brusca.

PREGUNTA N 66 RESPUESTA: C
Toda lcera gstrica es potencialmente maligna independientemente de su aspecto, por lo que est indicado biopsiar todas ellas y tomando al
menos tres muestras (del nicho y de la periferia).

PREGUNTA N 67 RESPUESTA: D
La presencia de cuadro diarreico con masa palpable nos hace pensar en las respuestas 3, 4 y 5. Estos tres tumores podran ser ms o menos
blandos, y no dolorosos, pero el nico que tpicamente cursa con un cuadro de deplecin electroltica por ser abundamentemente secretor, es el
adenoma tipo velloso de recto.

PREGUNTA N 68 RESPUESTA: D
Todo cuadro de disfagia de larga evolucin sin sndrome constitucional acompaante y sin ninguna otra alteracin hace pensar en su trastorno
motor de esfago, y por tanto tendr una endoscopia normal, y slo podr ser filiado mediante una manomera esofgica.

PREGUNTA N 69 RESPUESTA: E
Todo varn adulto que presenta cuadro de hemorragia digestiva baja, aunque presente causa aparente del sangrado como podran ser las
hemorroides internas, debe hacerse revisar todo el colon mediante colonoscopia, para descartar otra causa asociada, y sobre todo el cncer
colorrectal.

[34]
Mg. Dany Colca
RESIDENTADO EN ENFERMERA. Compendio Banco de Preguntas. Lic. Carolina Ayala

PREGUNTA N 70 RESPUESTA: D
El cuadro descrito se trata de un megacolon txico en un paciente con colitis ulcerosa. Aunque el tratamiento del cuadro suele ser quirrgico
requiriendo panproctocolectoma, el primer paso siempre debe ser conservador intentando revertir el cuadro, y as obviar los riesgos de la ciruga.

PREGUNTA N 71 RESPUESTA: D
Las biopsias gstricas se mantienen negativas y se ha producido una cicatrizacin sustancial. Mientras las biopsias sean negativas debe darse ms
tiempo a que cicatrice la lcera antes de remitir al paciente a ciruga por temor a que la lcera sea maligna. La conducta apropiada, debido a que la
paciente est asintomtica y con buena evolucin en la cicatrizacin, es continuar con el tratamiento con ranitidina 8 semanas ms y despus repetir
la endoscopia. Otra conducta, en especial si la lcera no muestra este progreso satisfactorio, sera cambiar a omeprazol.

PREGUNTA N 72 RESPUESTA: C
Los plipos gstricos son de origen epitelial y de ellos el 90% son hiperplsicos y del 10 al 20% adenomatosos. si bien pueden causar dolor
abdominal a hemorragia, hasta la mitad son asintomticos. La extirpacin total de la lesin mediante polipectoma con asa a fin de obtener el espcimen
total por estudio histolgico es diagnstica y curativa.

PREGUNTA N 73 RESPUESTA: C
Los pacientes con celaca pueden mostrar anemia ferropnica aislada. El festoneo de los pliegues duodenales sugiere esta afeccin y puede ser
uno de sus primeros signos endoscpicos, lo cual viene apoyado por los hallazgos de la biopsia.

PREGUNTA N 74 RESPUESTA: C
El paciente presenta los datos analticos y de exploracin caractersticos del infarto intestinal en evolucin, en probable relacin con arteritis.

PREGUNTA N 75 RESPUESTA: E
El desarrollo de un sndrome txico en personas de edad avanzada, en especial cuando se acompaa de pruebas de afeccin sistmica, debe hacer
pensar en Enfermedad de Whipple.

PREGUNTA N 76 RESPUESTA: D
La recomendacin previa para vigilancia por plipos del colon metacnicos ha sido colonoscopia anual. Datos recientes del National Colon Polyp
Study apoyan un intervalo de 3 aos para el procedimiento, excepto en persona con plipo adenomatoso > 1 cm. de 9, en quienes hay mltiples
plipos y en pacientes con antecedente de plipo maligno. Tambin se ha sugerido que los adenomas tubulares < 1 cm no requieren vigilancia
adicional.

PREGUNTA N 77 RESPUESTA: B
Esta pregunta hace referencia a una pubertad precoz verdadera, con aumento de la talla con una edad sea mayor a la cronolgica, telarqua, as
como valores hormonales ms altos que los que corresponden a la prepubertad. Por otra parte descartar otras causas de pubertad precoz como son
las debidas al sndrome de Albright y alteraciones del tiroides, suprarrenal o primitivas del ovario (tumores ovricos). El tratamiento de eleccin son
los anlogos de GbRH ya que retrasan el desarrollo sexual y la maduracin esqueltica. El acetato de progesterona tiene escasos resultados en
cuanto al control del crecimiento. La respuesta 1 sera la correcta si sospechsemos patologa ovrica tumoral y la 5 en caso de tratarse
exclusivamente de una telarqua precoz.

PREGUNTA N 78 RESPUESTA: E
La ausencia de menstruacin tras la administracin de gestgenos descarta la anovulacin como causa de amenorrea (SOP) y la opcin 3 se
elimina ya que tiene niveles de prolactina normales. La presencia de menstruacin al administrar estrgenos y gestgenos nos orienta a que el fallo
pueda estar a nivel del compartimento II o III-IV, descartando el sndrome de Asherman. Los niveles de gonadotropinas bajos nos permite descartar
la opcin 2 y orientar el caso clnico como opcin correcta la nmero 5.

PREGUNTA N 79 RESPUESTA: D
Ante una metrorragia postmenopusica es esencial realizar una ecografa vaginal que nos permita valorar la lnea media endometrial y si existe
alguna duda de patologa endometrial, debemos realizar legrado fraccionado para diagnstico anatomopatolgico endometrial. Despus de descartar
patologa endometrial podemos valorar la THS ms adecuada.

PREGUNTA N 80 RESPUESTA: A
La cadidiasis vaginal se caracteriza por la aparicin de prurito y edema de vulvas e introito, con secrecin vaginal blanquecina grumosa de aspecto
caseoso. Placas blanquecinas irregulares en mucosa vaginal que se desprenden,

[35]
Mg. Dany Colca
RESIDENTADO EN ENFERMERA. Compendio Banco de Preguntas. Lic. Carolina Ayala

dejando ulceraciones rojas superficiales. Entre las fracturas favorecedoras ms importantes est el tratamiento antibitico prolongado, embarazo,
anovulatorios...

PREGUNTA N 81 RESPUESTA: C
Aunque probablemente se trate de una artritis reumatoide, en este momento no cumple criterios para el diagnstico pues slo lleva 15 das de
evolucin.

PREGUNTA N 82 RESPUESTA: B
Pues aunque la paciente cumple tres criterios para el diagnstico de arteritis de la temporal (edad, VSG y cefalea) debemos realizar una biopsia de
la arteria temporal para confirmar el diagnstico pues en el tratamiento de la arteritis de la temporal las dosis de prednisona son mayores que las que
usaramos en la polimialgia reumtica.

PREGUNTA N 83 RESPUESTA: D
Pues antes de diagnosticar una espondiloartropata debemos descartar los dos procesos infecciosos crnicos capaces de provocar sacroiletis
con frecuencia en nuestro medio, la TBC y la brucella.

PREGUNTA N 84 RESPUESTA: E
Pues las patologas que pueden simular una fibromialgia son el LES de inicio, el hipotiroidismo o la miositis como ms importantes.

PREGUNTA N 85 RESPUESTA: C
Pues el diagnstico ms probable es una DSR (sd. hombro-mano) que por clnica, radiografa y gammagrafa sea se diagnosticar. La
gammagrafa con galio no est indicada pues no se sospecha un proceso infeccioso.

PREGUNTA N 86 RESPUESTA: C
Pues la paciente presenta un eritema nudoso junto a un peritobillo bilateral caracterstico de la sarcoidosis.

PREGUNTA N 87 RESPUESTA: B
El diagnstico diferencial de los aplastamientos vertebrales debe realizarse fundamentalmente entre la osteoporosis, la osteomalacia, las metstasis
seas y el mieloma mltiple. La presencia de debilidad muscular debe hacer sospechar el diagnstico de osteomalacia. En este caso, si el calcio,
el fsforo y la vitamina D son normales, puede descartarse razonablemente el diagnstico; si estn alteradas, la biopsia sea proporcionar el
diagnstico definitivo. En caso de mieloma mltiple, la VSG est elevada y suele existir anemia. La densidad mineral sea est disminuida en
osteoporosis, osteomalacia y mieloma mltiple. La gammagrafa sea y la resonancia magntica son tiles para detectar lesiones subclnicas en
caso de metstasis seas.

PREGUNTA N 88 RESPUESTA: B
La paciente cumple 4 criterios para el diagnstico de lupus eritematoso sistmico (aftas orales, artritis, leucolinfopenia y
ANA +), por lo que se puede establecer el diagnstico de esta enfermedad.

PREGUNTA N 89 RESPUESTA: A
En este caso, sin ninguna manifestacin mayor de lupus eritematoso sistmico, el tratamiento indicado son antiinflamatorios no esteroideos. En caso
de persistencia o empeoramiento de la clnica, podra valorarse la utilizacin de otros tratamientos.

PREGUNTA N 90 RESPUESTA: E
La hiperuricemia asintomtica no es tributaria de tratamiento hipouricemiante a menos que las cifras de cido rico en sangre predispongan al
desarrollo de una nefropata.

PREGUNTA N 91 RESPUESTA: D
La fiebre reumtica es un tipo de artritis reactiva postestreptoccica que se diagnostica con los criterios de Jones.

PREGUNTA N 92 RESPUESTA: D
El tratamiento conservador de la artrosis incluye analgesia y/o antiinflamatorios, descarga articular y fisioterapia. Cuando estas medidas han fallado
y la impotencia funcional es severa, puede plantearse una solucin quirrgica. La osteotoma de alineacin puede retrasar sensiblemente la evolucin
del proceso artrsico.

[36]
Mg. Dany Colca
RESIDENTADO EN ENFERMERA. Compendio Banco de Preguntas. Lic. Carolina Ayala

PREGUNTA N 93 RESPUESTA: D
La latencia de sueo se objetiva mediante el MSLT (mltiple sleep latency test), que en cuanto es inferior a 10 minutos indica hipersomnia. La
dificultad para despertarse y la falta de sueo reparador son caractersticas de la hipersomnia que la diferencian de los grandes dormidores que s
tienen sueos reparadores aunque su tiempo de sueo sea mayor que la media. Una duracin inadecuada del sueo nocturno (inferior a 7 horas)
puede producir sntomas similares a la hipersomnia primaria pero los pacientes recuperan el sueo cuando tienen das libres. En la narcolepsia el
sueo es ms reparador, existen menos dificultades para levantarse, la somnolencia diurna es en forma de crisis y no continua, presentan siestas
cortas, e intrusiones recurrentes de fases REM durante el da. En la apnea del sueo no suele haber un patrn recidivante, suele haber alteraciones
cardiovasculares y obesidad y no suele haber alteraciones emocionales o de conducta descritas que corresponden claramente a las de un sndrome
de Klein-Levin o hipersomnia idioptica recidivante, sin embargo, a veces es necesario hacer un registro polisomnogrfico para diferenciar una apnea
obstructiva de una hipersomnia idioptica. En la epilepsia los fenmenos postcrticos no duran tanto y en las auras o crisis psicomotoras, si bien
existe cierta disminucin del nivel de conciencia no se le puede despertar como a un hipersomne. Siempre que un diagnstico es difcil se plantea
la posibilidad de un sndrome conversivo o simulacin, los estados disociativos se dan sobre todo en relacin con una situacin que contara al
paciente, las lesiones o accidentes que se simulan no suelen ser graves salvo error de clculo. La personalidad del paciente conversivo o disociativo
no es normal entre crisis.

PREGUNTA N 94 RESPUESTA: C
La paciente padece posiblemente anorexia nerviosa. Los pacientes con dicho trastorno suelen negar datos necesarios para el diagnstico, por lo
que ante la evidencia de la delgadez debemos sospecharlo inmediatamente y buscar algn dato objetivo como el peso, presencia de amenorrea,
signos de desnuricin como el fro, la fragilidad de piel y mucosas, etc. Sin embargo estos signos no son especficos. La familia no es tampoco una
fuente fiable de informacin, existe en ocasiones excesiva tolerancia o incomprensin hacia lo que est pasando. La paciente desconfa de los
mdicos a los que ve como entrometidos en su libertad de comer como quiera. Por otro lado la irritabilidad de estas pacientes crea un rechazo por
parte de los profesionales. La nica manera de establecer un diagnstico cierto es mediante un dilogo abierto, dejando a un lado las preguntas
cerradas a fin de averiguar si la paciente est preocupada de forma desmedida por engordar o si se ve gorda a pesar de su delgadez.

PREGUNTA N 95 RESPUESTA: C
Todos los trastornos mencionados pueden presentar hiperfagia. En el sndrome de Klein-Levin la hiperfagia se presenta tras episodios de
hipersomnia. En la depresin bipolar y en la esquizofrenia los sntomas predominantes son los afectivos y psicticos respectivamente y adems la
hiperfagia no es de forma tan recurrente en forma de atracones seguidos de vmitos. En la anorexia nerviosa tipo compulsivo el cuadro es
prcticamente el mismo pero suele haber disminucin de peso, amenorrea y signos de desnutricin que faltan en la bulimia.

PREGUNTA N 96 RESPUESTA: E
El trastorno de angustia suele presentarse en crisis repetidas de breves minutos de duracin, salvo que se tengan varias crisis en salvas, situacin
en la cual la crisis parece durar ms tiempo, pero una exploracin detallada nos revelar altibajos. Las crisis aparecen sin relacin a estar en
situaciones temidas como en las fobias (agorafobia), con un cuadro clnico muy similar al que presenta la paciente. Se desarrolla temor anticipatorio
a sufrir nuevas crisis y generalmente aparece temor a estar en situaciones donde sea difcil obtener ayuda si se tiene la crisis (agorafobia). La
presencia de conectivopatas (prolapso mitral, hiperlaxitud ligamentaria) parece ser mayor en estos pacientes y no slo porque sean explorados
ms por la presencia de quejas somticas. De todas maneras tambin los pacientes psiquitricos pueden tener patologa mdica asociada, en este
caso prolapso mitral. La frecuencia cardaca y la tensin arterial alta debido a la descarga noradrenrgica del locus ceroleous es comn en estos
pacientes.

PREGUNTA N 97 RESPUESTA: D
No hay signos somticos ni psquicos de ansiedad, sino ms bien enlentecimiento psicomotor. En los trastornos de personalidad y en la neurosis
existe un patrn prolongado en el tiempo de quejas muy variadas, a veces remendando un cuadro depresivo. Tanto en la demencia como en la
depresin hay disminucin de concentracin, el inicio de una demencia puede simular una depresin pero no suele existir un inicio tan brusco, salvo
en las vasculares que se suelen acompaar de cadas, dficit neurolgicos o episodios confusionales. La paciente presenta anhedonia (incapacidad
para disfrutar de las cosas que antes le daban placer), este sntoma o bien la tristeza han de estar presentes para poder hacer un diagnstico de
depresin mayor. En este caso la depresin presenta sntomas de tipo endgeno como insomnio de tercera fase, disminucin de apetito, etc. Salvo
que la paciente haya tenido contacto con mdicos o bien presente otra patologa diferente de la depresin no es habitual que la paciente use la
palabra depresin o triste, sino trminos ms descriptivos como estoy por los suelos, lo veo todo negro, estoy ms cansada, etc. De forma similar
el paciente con depresin mayor no viene por s slo sino que ha de ser trado por alguien de confianza al que ha relatado alguna de sus experiencias.

[37]
Mg. Dany Colca
RESIDENTADO EN ENFERMERA. Compendio Banco de Preguntas. Lic. Carolina Ayala

PREGUNTA N 98 RESPUESTA: B
El paciente no presenta al parecer alucinaciones, delirios bizarros o sntomas defectuales que nos puedan hacer dudar con la esquizofrenia o con
un cuadro esquizoafectivo. No existe disminucin del funcionamiento normal como en la depresin, todo lo contrario, el funcionamiento es
superior al normal, si bien en este caso no mejor, ya que su comportamiento le crea problemas (discapacidades) familiares que en un futuro
sern tambin sociales y laborales. Los pacientes ansiosos se saben enfermos y piden ayuda en vez de no ser conscientes de ser enfermos como
es el caso del paciente. Los trastornos de personalidad suelen tener una evolucin prolongada en el tiempo. Los pacientes manacos pueden tener
un nimo expansivo (simptico, de buen humor contagioso) o bien como es este caso al menos durante la consulta, irritable. El funcionamiento
general es superior a la media pero inestable, con realizacin de proyectos cuyo xito es difcil o imposible, los proyectos que se inician difcilmente
se terminan ya que otro inters lo suele desplazar, el habla suele ser rpida en consonancia con la actividad mental. Si el nivel de actividad alto no
le produce alteraciones sociales, laborales o familiares se llama hipomana, si las produce se llama mana y requiere tratamiento con litio y a veces
adems neurolpticos y/o ingreso.

PREGUNTA N 99 RESPUESTA: B
El paciente presenta un aspecto llamativo sugerente de patologa grave. El hecho de que haya estado vagabundeando largo tiempo hasta tener una
piel tostada y su aspecto desaseado hace improbable una histeria o una intoxicacin por LSD que tienen crisis de inicio y duracin cortas. Un
paciente manaco no llega a tener tal deterioro del autocuidado personal. El paciente est ms que con un nivel de conciencia bajo, perplejo y parece
estar absorto oyendo voces, su afecto es inapropiado y su conducta bizarra. Parece que el paciente pueda haberse fugado. Estas fugas pueden darse
en la epilepsia, en la histeria y en las psicosis esquizofrnicas, siendo en los dos primeros casos de duracin breve, no hasta el extremo de
poder llevar varios das con tal grado de abandono de autocuidados.

PREGUNTA N 100 RESPUESTA: A


El paciente presenta un comportamiento muy llamativo, su deseo de aislarse no parece motivado por una depresin, que se acompaara de quejas
de nimo bajo, cansancio, inhibicin psicomotriz, etc. El paciente parece tener ideas de referencia (hablan de l), ideas de vigilancia por los vecinos
y parece tener alucinaciones auditivas comentadoras de la propia actividad (oye a un vecino dos pisos por encima de l comentar lo que hace a
cada momento). Es frecuente que este y otro tipo de pacientes no podamos hacer un diagnstico a partir de sntomas referidos por el paciente sino
a partir de indicios que como en este caso son muy sugerentes.

CLAVE DE RESPUESTAS TEMA B

PREGUNTA N 1 RESPUESTA: A
Las intoxicaciones o sobredosis de frmacos dan un cuadro de disminucin de nivel de conciencia que puede producir un cuadro similar,
concretamente el sndrome neurolptico maligno si bien se acompaara de rigidez. Es improbable que se le haya administrado un exceso
de frmacos mientras est ingresado, salvo las benzodiacepinas, que a veces se suelen dar como hipnticos o como tranquilizantes con excesiva
facilidad a todo tipo de pacientes, incluso ancianos en los que el riesgo de un mayor efecto es ms alto. En cualquier caso el cuadro clnico
sera completamente diferente, con sopor, sin convulsiones, sin temblor. Siendo improbable que sea por algn frmaco que se le est
administrando en exceso parece ms probable que sea algo que haya dejado de tomar, en este caso las sospechas se dirigen especialmente al
alcohol o las benzodiacepinas cuyo cuadro de abstinencia es identico al del paciente incluso aunque la familia diga que no bebe mucho o lo normal.
Otra causa de desorientacin al poco de ingresar son los cuadros demenciales pero no se suelen acompaar de convulsiones o alucinaciones.

PREGUNTA N 2 RESPUESTA: C
La paciente presenta un patrn de comportamiento anormal prolongado en el tiempo caracterizado por inestabilidad emocional en las relaciones en
sus objetivos personales, etc. Muestra inters por relacionarse con la gente al revs que el trastorno esquizoide, si bien este inters es egosta y con
el nico objetivo de evitar la soledad, tener alguien a quien contar los problemas, sin tener conciencia de que estas relaciones han de ser mutuamente
satisfactorias. Busca en general su autosatisfaccin llevando las situaciones al lmite hasta producir la ruptura de las relaciones. No cede a las
exigencias de otros con tal de mantener la relacin como en el caso del trastorno de personalidad por dependencia.

PREGUNTA N 3 RESPUESTA: C
El paciente presenta pensamientos intrusivos, que invaden la conciencia y sin embargo el paciente sabe que esos pensamientos son suyos y no
impuesto por el exterior como por ejemplo en la esquizofrenia, le producen malestar e intenta quitrselos realizando algunos actos conpulsivos
como ponerse cierta ropa, etc. Presenta adems rasgos de personalidad obsesiva como el perfeccionismo, etc. Es frecuente que existan fobias
asociadas a estmulos que desencadenan los pensamientos obsesivos o depresin pero no parece ser el caso de este paciente.

[38]
Mg. Dany Colca
RESIDENTADO EN ENFERMERA. Compendio Banco de Preguntas. Lic. Carolina Ayala

PREGUNTA N 4 RESPUESTA: E
La paciente presenta miedo a espacios cerrados, abiertos o llenos de gente, que tienen en comn que son sitios en los que la paciente piensa que
es ms difcil poder obtener ayuda si le ocurriera algo. El solo hecho de exponerse a esos sitios desencadena una crisis de ansiedad, lo cual es tpico
de las fobias, en este caso agorafobia. Adems presenta crisis de pnico, que fue la primera manifestacin psicopatolgica y que caus
secundariamente la agorafobia. Los trastornos simulados o facticios pueden reproducir la mayora de los trastornos psiquitricos en general con el
fin de obtener un beneficio, sin embargo debemos dejarlos siempre como un diagnstico a hacer una vez excluidos otros ms probables.

PREGUNTA N 5 RESPUESTA: C
En ocasiones en la pancreatitis aguda el electrocardiograma es anormal con alteraciones del ST y onda T semejantes a la isquemia miocrdica.

PREGUNTA N 6 RESPUESTA: A
La retinopata de Purtscher es una complicacin de la pancreatitis aguda relativamente rara que cursa con prdida sbita e importante de la visin.
En el fondo de ojo muestra manchas algodonosas y hemorragias circunscritas a un rea limitada de paila y mcula. Se cree que se debe a la oclusin
de la arteria retiniana posterior con granulocitos agregados.

PREGUNTA N 7 RESPUESTA: D
La cetoacidosis diabtica se suele acompaar de dolor abdominal y aumento de la amilasa srica, pero la lipasa e isoamilasa pancretica
en suero no se elevan. Los pacientes con pH inferior a 7,32 pueden presentar falsas elevaciones de la amilasa srica.

PREGUNTA N 8 RESPUESTA: E
El sndrome de Rotor es un trastorno del metabolismo de la bilirrubina que produce hiperbilirrubinemia mixta, parecido al sndrome de Dubin-Johnson
pero sin pigmento en la clula heptica y con predominio de monoconjugados en la bilirrubina conjugada. La colecistografa oral es normal
y no existe elevacin tarda en la curva de eliminacin de bromosulftanena. Se hereda de forma autosmica recesiva.

PREGUNTA N 9 RESPUESTA: C
El sndrome hepatorrenal es una complicacin grave, propia de pacientes cirrticos con ascitis que se caracteriza por insuficiencia renal
progresiva con marcada retencin de sodio y oliguria en ausencia de cualquier causa especfica identificable de disfuncin renal.

PREGUNTA N 10 RESPUESTA: D
Las manifestaciones clnicas de la pancreatitis son dolor en epigastrio irradiado hacia la espalda que mejora al flexionar el tronco y se acompaa
frecuentemente de nuseas y vmitos. El DDI es una causa de pancreatitis de origen medicamentoso.

PREGUNTA N 11 RESPUESTA: B
Entre las complicacions locales de la pancreatitis aguda, se encuentra el flemn pancretico. Se caracteriza por ser una masa inflamatoria que parece
entre la 2.a y 3.a semana tras el episodio de pancreatitis. Se manifiesta por leucocitosis, fiebre, hiperamilasemia persistente a pesar del tratamiento
correcto.

PREGUNTA N 12 RESPUESTA: B
Dentro del grupo de los tumores pancreticos el gastrinoma (productor del sndrome de Zollinger-Ellison) se caracteriza clnicamente por la existencia
de lceras ppticas resistentes al tratamiento mdico, hipercalcemia y diarrea acuosa.

PREGUNTA N 13 RESPUESTA: A
El adenocarcinoma de pncreas es ms frecuente en varones, mayores de 50 aos, los sntomas iniciales son insidiosos y suele aparecer por lo
menos 2 meses antes de que se diagnostique el tumor. Se caracteriza por dolor sordo abdominal y prdida de peso en ms del 75% de los pacientes.
La ictericia debida a la obstruccin biliar aparece en el 80% de los enfermos.

PREGUNTA N 14 RESPUESTA: C
Las causas de pancreatitis crnica son similares a las de la pancreatitis aguda, siendo el alcoholismo una de las principales. Es caracerstico el dolor
persistente, que empeora con la ingesta y no responde a anticidos. La trada clsica consiste en calcificaciones pancreticas, esteatorrea y diabetes
mellitus.

PREGUNTA N 15 RESPUESTA: D
La amiodarona es un antiarrtmico que puede ocasionar lesin heptica, sta se puede producir tanto al inicio del tratamiento como despus de varios
meses. El cuadro clnico que produce es semejante al de cualquier hepatitis viral (astenia, anorexia, etc.). Es caracterstica que la anatoma patolgica
presente cuerpos lisosmicos lamelares de fosfolpidos al microscopio electrnico.

PREGUNTA N 16 RESPUESTA: C
El sndrome de Reye es una entidad que aparece en la edad peditrica que se caracteriza por la asociacin de encefalopata aguda, rpidamente
progresiva y esteatosis visceral de predominio heptico. La etiopatogenia no es clara pero se ha relacionado con una infeccin viral previa y el
consumo de cido acetilsaliclico.

[39]
Mg. Dany Colca
RESIDENTADO EN ENFERMERA. Compendio Banco de Preguntas. Lic. Carolina Ayala

PREGUNTA N 17 RESPUESTA: C
La ecografa escrotal es la ms til de las pruebas mencionadas para determinar si el testculo est intacto o no. Si presente una rotura,
precisar exploracin quirrgica y reparacin de la lesin. En el caso contrario, podr tratarse de forma conservadora. Si la ecografa es normal, las
posibilidades que se trate de una rotura sern muy escasas.

PREGUNTA N 18 RESPUESTA: A
El linfoma es el tumor testicular ms comn en este grupo de edad, mientras que los tumores de clulas germinales no seminomatosos
son poco frecuentes. Los linfomas testiculares habitualmente son de clulas tipo B. La leucemia linftica crnica es una causa poco comn de
masa testicular.

PREGUNTA N 19 RESPUESTA: C
Los urocultivos de aproximadamente el 15% de las mujeres con bacteriuria revelarn la presencia de bacterias uropatgenas en concentraciones de
1.000 colonias inferior por ml. La historia clnica y los hallazgos bacteriolgicos indican una infeccin significativa que debera tratarse, ms que una
contaminacin de la orina.

PREGUNTA N 20 RESPUESTA: C
El paciente tiene una clara progresin de la enfermedad despus de un tratamiento adecuado. La radioterapia localizada sera la forma ms efectiva
para reducir su dolor seo. Debera considerarse la posibilidad de aplicar otras medidas una vez paliado el dolor. No hay razn alguna para utilizar
un tratamiento con anlogos de la LH-RH despus de la orquiectoma y tras haber fracasado el tratamiento hormonal.

PREGUNTA N 21 RESPUESTA: E
Gracias al uso cada vez ms frecuente del cateterismo arterial umbilical, se ha podido documentar bien la trombosis de la arteria renal como causa
significativa de hipertensin infantil que a menudo provoca insuficiencia cardaca congestiva. Este diagnstico debe sospecharse en todo nio con
hematuria inexplicada, hipertensin o insuficiencia cardaca congestiva.

PREGUNTA N 22 RESPUESTA: D
La necesidad de tratamiento quirrgico en estos pacientes viene dictada principalmente por la presencia de infeccin y fiebre. De no ser as, los
clculos pequeos de 3 mm. se deja que se eliminen espontneamente.

PREGUNTA N 23 RESPUESTA: D
Los procedimientos de by-pass intestinal pueden producir esteatorrea. La esteatorrea provoca precipitacin de calcio de la dieta en la grasa
intraluminal (saponificacin). Esta saponificacin reduce el nivel de calcio inico intraluminal a valores ms bajos de los normales. Normalmente
el calcio inico en la luz intestinal precipita con el oxalato de la dieta. El dficit de calcio inico originado por la saponificacin se traduce en un
aumento de la absorcin de oxalato de la dieta. Como consecuencia de ello, el paciente desarrolla hiperoxalemia, hiperoxaluria y mayor frecuencia
de litognesis de oxalato clcico.

PREGUNTA N 24 RESPUESTA: A
Al igual que en los pacientes con tumor de Wilms, pero sin afectacin de la cava, el factor pronstico ms importante para la supervivencia es el
patrn histolgico. El ndice de supervivencia a los tres aos con histologa favorable era del 86%, frente a un 35% con histologa desfavorable (es
decir, variables anaplsicas).

PREGUNTA N 25 RESPUESTA: A
Existe una correlacin del 100% entre la presencia del reflejo cremastrico ipsilateral y la ausencia de torsin testicular. El reflejo cremastrico es un
reflejo de la piel superficial que consiste en la contraccin del msculo cremster ipsilateral y la elevacin del testculo ipsilateral al golpear la piel de
la cara interna del muslo. La ausencia de dicho reflejo, aunque no es diagnstico de torsin testicular, aumenta las sospechas de torsin testicular e
indica la necesidad de proseguir con ms exploraciones complementarias.

PREGUNTA N 26 RESPUESTA: C
El propranolol aumenta la presin uretral por su efecto bloqueante betaadrenrgico. Sin embargo, puede tambin provocar broncoconstriccin debido
al bloqueo de los receptores bronquiales beta-adrenrgicos. Por ello, este frmaco est contraindicado en pacientes que padecen asma o enfermedad
pulmonar obstructiva.

PREGUNTA N 27 RESPUESTA: A
La nica forma de aumentar la supervivencia en el cncer de prstata es mediante la ciruga radical ante tumores localizados. En principio se
considera que en una ciruga de cncer de prstata, para aumentar la supervivencia el paciente debe de tener menos de
70 aos. El resto de los tratamientos slo consiguen un efecto paliativo de la enfermedad mejorando la sintomatologa, pero no consiguen erradicar
la enfermedad.

PREGUNTA N 28 RESPUESTA: A

[40]
Mg. Dany Colca
RESIDENTADO EN ENFERMERA. Compendio Banco de Preguntas. Lic. Carolina Ayala

Ante un tumor de
PREGUNTA Nesa17 extensin y con afectacin
RESPUESTA: C de la muscular vesical, el nico tratamiento con intencin curativa es la cistectoma radical, y
derivacin urinaria con interposicin de intestino. La cistectoma parcial debido al tamao del tumor no sera viable. La quimioterapia es un tratamiento
paliativo y no curativo en el cncer vesical.

[41]
Mg. Dany Colca
RESIDENTADO EN ENFERMERA. Compendio Banco de Preguntas. Lic. Carolina Ayala

PREGUNTA N 29 RESPUESTA: C
El inicio insidioso sin poder precisar el momento, su curso progresivo y el fallo de memoria reciente como sntoma de presentancin es ms
caracterstico de la enfermedad de Alzheimer.

PREGUNTA N 30 RESPUESTA: D
El cuadro clnico es caracterstico y adems se dan unos factores predisponentes como son la edad avanzada, la deprivacin sensorial, la
infeccin y el cambio ambiental. La presentacin nocturna tambin es caracterstica de este cuadro.

PREGUNTA N 31 RESPUESTA: E
Tanto los diurticos como las benzodiazepinas se asocian a cadas en el anciano. La hipotensin postural es ms frecuente en esta poblacin y
puede verse favorecida por los diurticos y el cambio brusco de posicin. Probablemente la falta de iluminacin contribuy a la cada.

PREGUNTA N 32 RESPUESTA: E
La rabdomilisis es una complicacin frecuente en sujetos que permanecen tiempo tumbados, produce elevacin marcada de la CPK y puede causar
fracaso renal agudo. La hipotermia es un diagnstico dficil si no se piensa en ello. Este paciente ha pasado la noche en el suelo, es invierno y padece
lesin neurolgica que puede deteriorar la termorregulacin.

PREGUNTA N 33 RESPUESTA: D
Todos los dems factores contribuyen a la aparicin, pero el principal siempre es la inmovilidad con la consiguiente presin mantenida sobre la misma
regin anatmica.

PREGUNTA N 34 RESPUESTA: D
Ante un paciente con deterioro cognitivo de inicio subagudo, con alteracin del nivel de conciencia debe investigarse el hecho de un antecedente
traumtico por banal que parezca y descartar mediante TAC de crneo un hematoma subdural. Su importancia radica en que es tratable, y si no se
detecta a tiempo puede ser mortal.

PREGUNTA N 35 RESPUESTA: D
Una vez descartadas las causas transitorias de incontinencia urinaria, la inestabilidad vesical es la causa ms frecuente de incontinencia establecida,
manifestndose de forma caracterstica por urgencia-incontinencia. El diagnstico se realiza a travs del estudio urodinmico. El tratamiento mdico
se basa en la utilizacin de frmacos con accin anticolinrgica. La ciruga estara indicada en el caso de incontinencia de estrs (debilidad del suelo
plvico o alteracin esfinteriana). Obviamente las medidas paliativas del catter vesical o el colector externo no son planteables.

PREGUNTA N 36 RESPUESTA: B
Cuando aparece la enfermedad la evolucin es rpida, asocindose con signos cerebelosos y extrapiramidales, apareciendo mioclonas
inducidas por los estmulos. Los signos extrapiramidales y las mioclonas pueden presentarse tambin en la enfermedad de Alzheimer, aunque en
fases ms avanzadas de la enfermedad. Si bien la enfermedad de Parkinson y de Huntington no presentan un deterioro mental rpido, s que pueden
presentar signos extrapiramidales. Los pacientes con hidrocefalia a presin normal presentan apraxia para la marcha e incontinencia sin mioclonas
o signos extrapiramidales.

PREGUNTA N 37 RESPUESTA: B
La dermatomiositis se asocia en ms de un 60% de los casos a una neoplasia. Habitualmente existe un paralelismo entre la extensin de la
dermatomiositis y la tumoral. El penfigoide bulloso se asocia en un porcentaje muy bajo (5-10%) a una neoplasia oculta.

PREGUNTA N 38 RESPUESTA: C
La incidencia del melanoma maligno se est incrementando rpidamente en todos los grupos de edad. El pronstico de esta lesin es peor en el
grupo de ancianos que en el grupo de los jvenes, lo cual se atribuye ms directamente al grado de invasin en el momento del diagnstico. Si bien
las otras caractersticas (localizacin, histologa, ulceracin, ausencia de regresin), tambin se asocian a un peor pronstico, ninguna de ellas ha
demostrado ser tan buen predictor de la supervivencia como el grado de invasin en el momento de la reseccin quirrgica.

PREGUNTA N 39 RESPUESTA: E
Esta paciente padece una enfermedad periodontal, causada en mayor frecuencia por una inadecuada higiene oral. Algunos pacientes con
enfermedades sistmicas (diabetes mellitus), pueden sufrir una progresin ms rpida de la enfermedad periodontal y ser asimismo ms
severa. Algunos frmacos, como la fenitona, producen hiperplasia gingival, condicionando un mayor riesgo para la enfermedad priodontal. El
edentulismo (prdida de las piezas dentarias) es una consecuencia de la enfermedad periodontal.

PREGUNTA N 40 RESPUESTA: D
Este paciente con historia de diabetes mellitus de larga evolucin tiene muchas probabilidades de padecer una neuropata autonmica asociada.
Como resultado de la neuropata puede existir alteracin de la contractilidad vesical. La adicin de amitriptilina ha podido comprometer an ms la
contractilidad muscular, provocando una arreflexia vesical, lo cual justificara la repercusin sobre el tracto urinario superior y los escapes involuntarios
de orina debido a rebosamiento.

[42]
Mg. Dany Colca
RESIDENTADO EN ENFERMERA. Compendio Banco de Preguntas. Lic. Carolina Ayala
PREGUNTA N 41 RESPUESTA: C
El cuadro es el de una otitis media aguda sin supuracin, que se complica provocando tromboflebitis del seno lateral. Los datos clave son la fiebre
en picos y el dolor retroauricular. Debera hacerse diagnstico diferencial con mastoiditis mediante escner.

PREGUNTA N 42 RESPUESTA: D
El edema de labio inferior y la lengua fisurada asociada a parlisis facial recurrente definen este sndrome.

PREGUNTA N 43 RESPUESTA: B
No se trata de un vrtigo de Mnire porque la hipoacusia no es fluctuante y faltan las crisis de vrtigo. El colesteatoma no produce hipoacusia
neurosensorial. La larga evolucin del proceso excluye la teraputica de choque frente a una hipoacusia sbita. Debera interrogarse la posibilidad
de un trauma sonoro crnico, pero la positividad de este hecho en la historia clnica no excluira la posibilidad del neurinoma del VIII par.

PREGUNTA N 44 RESPUESTA: D
El dato clave es la existencia de lesiones osteolticas en Rx.

PREGUNTA N 45 RESPUESTA: A
El plasmocitoma solitario podra producir el mismo cuadro, pero es infrecuente. La enfermedad de Woakes (poliposis nasal deformante)
es propia de la infancia.

PREGUNTA N 46 RESPUESTA: D
El cuadro es una parlisis recurrencial bilateral por crecimiento tiroideo. Estas parlisis podran intubarse, pero la maniobra puede traumatizar la
glotis y afectar al habla, adems de necesitar traqueostoma diferida para evitar estenosis subgltica por intubacin prolongada.

PREGUNTA N 47 RESPUESTA: E
Las cuatro primeras opciones son compatibles con un cuadro infecciosoirritativo. El edema alrgico de glotis produce disnea y ronquera,
pero rara vez da fiebre o tos.

PREGUNTA N 48 RESPUESTA: A
El colesteatoma puede simular los hallazgos audiolgicos de la otosclerosis, pero la otoscopia es normal. La timpanosclerosis no afecta al
reflejo estapedial; la luxacin de cadena traduce un aumento de la compliance timpnica en la timpanometra.

PREGUNTA N 49 RESPUESTA: E
El cuadro de hipoacusia neurosensorial fluctuante con reclutamiento positivo, unido a acfeno y vrtigo perifrico, es tpico de sndrome
de Mnire. El nistagmo a la derecha traduce hiporreflexia vestibular izquierda, congruente con el resto de los datos.

PREGUNTA N 50 RESPUESTA: C
Los senos maxilares no suelen dar clnica hasta los 4 aos.

PREGUNTA N 51 RESPUESTA: B
Las angina de Ludwig es una inflamacin del suelo de la boca de origen dentario; nada tiene que ver con las amgdalas palatinas.

PREGUNTA N 52 RESPUESTA: C
El dato clave sera la existencia de epistaxis importantes, lo que en principio excluira el resto de diagnsticos; el adenocarcinoma de etmoides es
excepcional en nios.

PREGUNTA N 53 RESPUESTA: B
Ante un edema de papila unilateral en un sujeto adulto es preciso descartar rpidamente una N.O.I.A. en relacin a una arteritis de clulas gigantes,
porque, aunque poco frecuente, el pronstico sin tratamiento es fatal para ambos ojos. La VSG est claramente elevada en la arteritis y no en otras
causas de edema papilar.

PREGUNTA N 54 RESPUESTA: A
La clnica es tpica de conjuntivitis. Los folculos y la adenopata preauricular aparecen en conjuntivitis vricas y la ms frecuente es la causada por
adenovirus.

PREGUNTA N 55 RESPUESTA: A
La prdida de visin de instauracin rpida acompaada de un defecto pupilar aferente nos indica patologa del nervio ptico. En la neuritis retrobulbar
la prdida de visin no se acompaa de otros sntomas o signos, salvo la respuesta pupilar anmala. Tpicamente ni el paciente ni el mdico ven.

PREGUNTA N 56 RESPUESTA: C

[43]
Mg. Dany Colca
RESIDENTADO EN ENFERMERA. Compendio Banco de Preguntas. Lic. Carolina Ayala
Ante un ojo rojoN
PREGUNTA doloroso
41 conRESPUESTA:
pupila en midriasis
C arreactiva y estrechamiento de cmara anterior, el diagnstico ms probable es un glaucoma
agudo de ngulo estrecho. La PIO muy elevada nos confirma el diagnstico.

[44]
Mg. Dany Colca
RESIDENTADO EN ENFERMERA. Compendio Banco de Preguntas. Lic. Carolina Ayala

PREGUNTA N 57 RESPUESTA: C
En la retinitis por CMV aparecen lesiones exudativas y hemorrgicas en patrn de queso y tomate, produce disminucin de la visin si hay afectacin
macular. Se da en pacientes inmunodeprimidos, incluido el SIDA. Requiere tratamiento con Foscarnet o Ganciclovir.

PREGUNTA N 58 RESPUESTA: C
La afectacin macular provoca un sndrome macular caracterizado por metamorfopsias, micropsias, o macropsias junto con disminucin de la agudeza
visual. De las opciones dadas, la correcta es la nica con afectacin macular primariamente.

PREGUNTA N 59 RESPUESTA: B
Hasta los 10-12 aos de edad podemos revertir la ambliopa mediante oclusin del ojo con mejor visin siempre que no exista una patologa orgnica
de fondo. La ciruga sobre los msculos extraoculares se debe retrasar hasta recuperar la visin del ojo amblope.

PREGUNTA N 60 RESPUESTA: E
La catarata nuclear senil provoca disminucin de agudeza visual progresiva y miopizacin del ojo por cambios en el ndice de refraccin
del cristalino (miopa de ndice). Se caracteriza porque empiezan a quitarse las gafas de cerca para trabajos de visin prxima.

PREGUNTA N 61 RESPUESTA: D
La obstruccin de la arteria central de la retina se caracteriza por disminucin brusca de agudeza visual, retina edematosa, blanquecina, contrastando
con el color rojizo de la mcula, que est respetada. Las enfermedades metablicas con mancha rojo- cereza se presentan en la infancia.

PREGUNTA N 62 RESPUESTA: D
Ante un cuadro clnico de estas caractersticas, a esta edad, el diagnstico ms probable es el de enfermedad de Coats, ya que el resto de las
patologas mencionadas se diagnostican ms precozmente, o bien presentan malformaciones signos oculares de ms temprana aparicin.

PREGUNTA N 63 RESPUESTA: C
La hemorragia vtrea es una complicacin frecuente de la retinopata diabtica. La isquemia macular y el edema macular (causas de disminucin de
visin) no aparecen bruscamente.

PREGUNTA N 64 RESPUESTA: E
La coriorretinopata central serosa aparece frecuentemente en varones jvenes-adultos nerviosos, con disminucin de agudeza visual variable,
sndrome mcula (metamorfopsias) y afecta mcula. Constituye un desprendimiento localizado de la neurorretina, producido por disfuncin del EPR.
El patrn angiogrfico en chimenea es poco frecuente pero tpico.

PREGUNTA N 65 RESPUESTA: D
La paciente, una mujer joven, presenta un patrn pulmonar intersticial. La ausencia de sintomatologa sistmica (fiebre, artralgias o sndrome
constitucional) hacen poco sugerente la existencia de una tuberculosis, un LES o una sarcoidosis. Es poco probable que la sarcoidosis presente
exclusivamente una afectacin parenquimatosa (estadio IIIA, afectacin exclusivamente parenquimatosa y IIIB, o IV, fibrosis pulmonar). El sexo
inicialmente nos orientara hacia LES, sarcoidosis y linfangioleiomiomatosis. Es particularmente tpico el incremento de los volmenes pulmonares,
al igual que ocurre en la granulomatosis de clulas de Langerhans (histiocitosis X). Esta peculiaridad se explica, en ambos casos, por la formacin
de mltiples quistes en la va respiratoria distal (en la linfangioleiomatosis, por la proliferacin de las clulas musculares). El derrame pleural y el
neumotrax son complicaciones frecuentes en el momento de establecer el diagnstico.

PREGUNTA N 66 RESPUESTA: E
Se trata de un varn no fumador que presenta un patrn alveolar bilateral sin cardiomegalia. La enfermedad de Hamman-Rich se caracteriza por un
patrn intersticial, y conduce a una fibrosis pulmonar de forma rpida. La ausencia de cardiomegalia, adems de una presin venosa yugular
normal, descarta la insuficiencia cardiaca. No hay datos para sospechar una infeccin por CMV, virus que caractersticamente afecta a individuos
inmunodeprimidos. La espirometra muestra un patrn restrictivo y, dato crucial para descartar la presencia de sangre en el espacio alveolar, la
capacidad de transferencia del CO est disminuida. El carcinoma bronquioalveolar es un tipo de adenocarcinoma (no relacin con el tabaco) con
tres formas clnicas: masa nica (ms de la mitad de los casos), multinodular o difuso (patrn alveolar por diseminacin endobronquial).

PREGUNTA N 67 RESPUESTA: C
El cuadro impresiona de infeccioso y de curso agudo. Las neumonas tpicas, como la neumoccica, presentan un patrn radiolgico de ocupacin
de espacio areo (alveolar) y las manifestaciones extrarespiratorias son raras. Aunque clnicamente es ms frecuente un sndrome gripal, ante toda
neumona atpica con factores epidemiolgicos sugerentes (contacto con ganado y/o sus productos, como granjeros, matarifes o veterinarios) debe
sospecharse la fiebre Q (Coxiella burnetii). La afectacin heptica (con elevacin moderada de transaminasas) apoya este diagnstico. La neumona
por legionella puede producir un cuadro superponible, aunque la falta de respuesta a eritromicina hace menos probable este diagnstico.
Fundamentalmente la agudeza del cuadro y la localizacin del infiltrado sin afectacin ganglionar hacen improbable que se trate de una
primoinfeccin tuberculosa. La neumonitis por hipersensibilidad (aunque en su forma aguda puede provocar fiebre, sin tanta afectacin general)
puede descartarse por la ausencia de antecedente de exposicin.

[45]
Mg. Dany Colca
RESIDENTADO EN ENFERMERA. Compendio Banco de Preguntas. Lic. Carolina Ayala

PREGUNTA N 68 RESPUESTA: C
El caso sugiere la existencia de una hemorragia alveolar difusa (HAD) con afectacin renal. La existencia de siderfagos (macrfagos cargados de
hemosiderina) en esputo o una capacidad de difusin del CO (DLCO) aumentada confirmaran la existencia de una HAD, pero no aporta informacin
sobre la etiologa del proceso. Cualquiera de las tres siguientes respuestas pueden permitir establecer el diagnstico, pero la realizacin de
biopsias suele diferirse en el tiempo. Por el contrario, la determinacin de anticuerpos antimembrana basal glomerular es crucial, ya que puede
permitir el diagnstico del sndrome de Goodpasture. La positividad en los anticuerpos anti-citoplasma de neutrfilo con patrn citoplasmtico (c-
ANCA), dirigidos contra la proteinasa-3, es altamente sugerente de una granulomatosis de Wegener.

PREGUNTA N 69 RESPUESTA: B
En el contexto de una hemorragia alveolar con afectacin renal, tanto tuberculosis como tromboembolismo pulmonar quedan prcticamente
descartadas. Por otra parte, en la granulomatosis de Churg-Strauss puede existir afectacin renal (menos frecuente que en el Wegener), pero son
muy frecuentes los antecedentes de asma y la eosinofilia perifrica. El cuadro clnico es muy sugerente de un sndrome de Goodpasture. Aunque
la granulomatosis de Wegener suele presentar otro patrn radiolgico (ndulos y/o masas mltiples) y la afectacin de las vas areas superiores es
muy frecuente, en principio no puede descartarse el diagnstico sin alguna prueba complementaria. El ttulo de anticuerpos antimembrana basal
glomerular es compatible con un sndrome de Goodpasture. Debe iniciarse lo ms precozmente posible la plasmafresis (adems de corticoides
para mejorar la situacin respiratoria).

PREGUNTA N 70 RESPUESTA: E
El tromboembolismo pulmonar (TEP) es la consecuencia ms grave de la trombosis venosa profunda (TVP). Ambas entidades son aspectos de una
misma enfermedad, la enfermedad tromboemblica venosa. En su patogenia se implican el estasis venoso (inmovilizacin, encamamiento), la lesin
vascular (ciruga de cadera) y las situaciones de hipercoagulabilidad. Clsicamente cursa con disnea de instauracin brusca y dolor torcico. El
signo clnico ms frecuente es la taquipnea. Aunque en un 90-95% el origen del trombo se localiza en miembros inferiores, hasta en un 50% de los
casos no se aprecian signos clnicos de TVP. La Rx de trax suele ser normal (la alteracin ms frecuente es la aparicin de atelectasia o derrame
-en un 10% de los casos, e indica infarto pulmonar-). El ECG ms frecuentemente identificado es normal (lo ms tpico es el patrn SIQIIITIII -que
sugiere sobrecarga ventricular derecha-). Los datos gasomtricos ms sugerentes son la hipoxemia e hipocapnia con un gradiente alveolo-arterial
aumentado. La prueba diagnstica de primera eleccin es la gammagrafa pulmonar de ventilacin-perfusin. El tratamiento consiste en
anticoagulacin con heparina (hay tendencia a iniciar simultneamente heparina y dicumarnicos y, en TEP masivo, pueden emplearse fibrinolticos).

PREGUNTA N 71 RESPUESTA: B
El sndrome de Lfgren es la forma ms frecuente de presentacin de la sarcoidosis aguda, particularmente en mujeres nrdicas e irlandesas (la otra
es el sndrome de Heerfordt-Waldenstrm). Consiste en la asociacin de eritema nodoso (paniculitis septal sin vasculitis) y adenopatas hiliares
bilaterales. Puede existir linfocitosis en el LBA (>15%) de pacientes con sarcoidosis, tuberculosis, asbestosis y linfomas, pero el cuadro clnico es
muy sugerente de sarcoidosis. El cociente CD4/CD8 alto (>3.5) junto a la presentacin tpica son casi diagnsticos de sarcoidosis. Sin embargo, el
diagnstico de certeza exige evidenciar los granulomas no caseificantes en cualquiera de los tejidos afectos. En 2/3 de los casos de sarcoidosis
existe elevacin de la enzima convertidora de la angiotensina (ECA) e hipercalciuria (con/sin hipercalcemia). El pronstico del sndrome de Lfgren
es favorable y, en principio, slo requiere tratamiento sintomtico (AINEs).

PREGUNTA N 72 RESPUESTA: A
El diagnstico de granulomatosis de Wegener se establece por la presencia de vasculitis granulomatosa en la va area superior (sinusitis, rinitis,
otitis media), inferior (ndulos pulmonares) con una glomerulonefritis (tpicamente microhematuria con cilindros eritrocitarios). Adems, distintos
grados de vasculitis en otros tejidos (piel, articulaciones, ojos, etc). El cuadro clnico que se describe es tpico, aunque el diagnstico requiere la
confirmacin histolgica (preferiblemente en tejidos ms accesibles, como la mucosa nasal). Sin embargo, el ttulo elevado de c-ANCA es
altamente sensible (88%) y especfico (95%). El curso clnico y la biopsia descartan la etiologa neoplsica y la tuberculosis (granulomas
caseificantes con bacilos cido-alcohol resistentes). La granulomatosis linfomatoide es en realidad un proceso linfoproliferativo (con infiltrado de
clulas linfocitoides y plasmocitoides atpicas en los vasos) en pulmn, piel, rin y SNC (realmente no es una vasculitis inflamatoria). La
granulomatosis de Churg-Strauss, o angetis alrgica, afecta preferente la va respiratoria inferior (y frecuentemente piel, riones) de pacientes
asmticos. La evidencia eosinofilia (>1.000/L) en un 80% de los casos (aunque ste no es un dato diferencial).

PREGUNTA N 73 RESPUESTA: D
Imipenem (imipenem/cilastatina) es un antibitico beta-lactmico del grupo de los carbepenems. Los carbapenems (imipenem, meropenem) presentan
caractersticas diferenciales de las penicilinas y de las cefalosporinas, que los hace idneos para el problema de la la neumona nosocomial y,
principalmente, de la multirresistencia. Aunque pueden usarse como monoterapia, es recomendable la asociacin (con aminoglucsidos o
ciprofloxacino). La monoterapia con ciprofloxacino es una mala alternativa por la aparicin precoz de nuevas resistencias. La asociacin
ceftazidima, amicamicina y vancomicina es una pauta clsica para el sndrome febril en la neutropenia severa. La ceftazidima es una cefalosporina
con actividad antipseudomona, y probablemente el ms empleado en el caso de neumonas nosocomiales en UCIs, recuperaciones, etc.

PREGUNTA N 74 RESPUESTA: C
La presencia de alguna calcificacin en el NPS es probablemente el dato ms sugerente de benignidad. Su ausencia debe alertar al clnico, aunque
no existan otros datos sugerentes de malignidad (adulto joven, tabaquismo acumulado escaso y tamao menor de 3 cm.). En la actualidad, la
segunda prueba diagnstica suele ser una TAC (aunque la rentabilidad diagnstica en este paciente es, a

[46]
Mg. Dany Colca
RESIDENTADO DE ENFERMERA. Compendio Banco de Preguntas. Lic. Carolina Ayala

priori, muy baja, tambin podramos solicitar citologa de esputo simultneamente). La recuperacin de Rx previas es particularmente importante,
pero no se ha de diferir la realizacin de pruebas complementarias.

PREGUNTA N 75 RESPUESTA: D
La duplicacin del volumen entre 11 das y 2 aos sugiere malignidad (30 y 490 das), aunque el crecimiento lento no lo descarta (el carcinoide, el
adenocarcinoma muy diferenciado y las metstasis pueden crecer muy lentamente). Se estima que existe duplicacin del volumen si se evidencia el
incremento del radio en un 33%. Generalmente la primera exploracin cruenta es la broncoscopia con/sin BTB (y posteriormente la PAAF si no se
obtiene un diagnstico). La rentabilidad de la BTB depende de la capacidad de acceso del fibrobroncoscopio (limitada por el calibre del rbol bronquial)
y el tamao del NPS (dimetro superior a 1 cm.). La localizacin tan perifrica decant hacia la realizacin de la PAAF. La morbilidad de la PAAF
(neumotrax, hemorragia) es muy baja y la rentabilidad diagnstica, en manos experimentadas, alta.

PREGUNTA N 76 RESPUESTA: B
En principio el diagnstico ms probable es el adenocarcinoma. El escaso tabaquismo y la presentacin perifrica hacen menos probables el
carcinoma epidermoide y el carcinoma de clulas pequeas (que tambin se asocia al tabaquismo, pero suele presentarse como masa
central con afectacin mediastnica). Si los hallazgos de la PAAF sugieren un adenocarcinoma hay que intentar descartar que el NPS no se
trate de una metstasis de un adenocarcinoma de otro origen (en principio la falta de otros sntomas no lo sugiere). El carcinoma bronquioalveolar es
un tipo de adenocarcinoma con una presentacin clnica y radiolgica tpica (expectoracin muy abundante y patrn alveolar, aunque es ms frecuente
la presentacin como ndulo o masa nica - 60% de las ocasiones-). En caso de un resultado de la PAAF no concluyente (presenta una rentabilidad
diagnstica en lesiones malignas del 80-
90%), debe realizarse una biopsia del NPS mediante toracotoma (microtoracotoma, y si no es posible, toracotoma abierta) y biopsia intraoperatoria
para decidir la cuanta de la reseccin pulmonar (tumor, segmento, lbulo).

PREGUNTA N 77 RESPUESTA: B
En una gestente con antecedentes de infertilidad y 36 aos de edad est indicado el diagnstico precoz de anomalas congnitas mediante pruebas
invasivas, como pueden ser la amniocentesis o la biopsia del vellosidades corinicas. Sin embargo, en esta paciente es prioritario descartar
una complicacin de la gestacin por el dolor clico hipogstrico que refiere. Dado el antecedente del factor tubrico, hemos de descartar un embarazo
ectpico. La primera prueba necesaria para ello es la ecografa, mediante la cual visualizaremos la vescula embrionaria dentro o fuera del tero.

PREGUNTA N 78 RESPUESTA: A Antes de realizar cualquier tipo de prueba invasiva por un triple screening anmalo como el que se
presenta en este caso clnico es necesario comprobar que efectivamente la gestacin corresponde al tiempo de evolucin que le suponemos. Es
decir, si no hay una ecografa previa que nos d la edad gestacional, hay que realizarla. Si entonces se confirman los datos de que disponemos, est
indicada una amniocentesis ante la probabilidad de una comosomopata.

PREGUNTA N 79 RESPUESTA: E
Esta paciente presenta una pielonefritis, complicacin mdica bastante frecuente en la gestacin. El tratamiento es antibitico endovenoso,
hasta conseguir un mnimo de 48 horas sin fiebre. Luego, dependiendo del estado general de la paciente puede concluir el tratamiento oral en su
domicilio. El primer diagnstico diferencial es con el clico nefrtico, en el que no se presenta fiebre ni signos analticos de infeccin.

PREGUNTA N 80 RESPUESTA: D El test de OSullivan es un mtodo de screening que se aplica a todas las embarazadas, y es aproximativo,
pero no diagnstico de la diabetes gestacional. Una vez obtenido un resultado patolgico, es necesario confirmar la enfermedad mediante una curva
de glucemia (prueba de tolerancia oral a la glucosa, basal y horaria en tres horas), que si es excesiva en al menos dos resultados diagnostica la
diabetes.

PREGUNTA N 81 RESPUESTA: C
Por la clnica que se define en el enunciado, podramos incluir este cuadro entre las metrorragias del primer trimestre. Para realizar el diagnstico
diferencial entre las causas ms habituales de metrorragia, debemos tener en cuenta que la enfermedad trofoblstica produce sangrado
indoloro con tero mayor que la edad gestacional; el embarazo ectpico produce una hemorragia interna con escaso sangrado al exterior, y
habitualmente el crvix est cerrado. En este caso el diagnstico es aborto, por lo que se debe realizar legrado quirrgico para su tratamiento.

PREGUNTA N 82 RESPUESTA: D
La causa ms frecuente de metrorragias en el tercer trimestre es la placenta previa. Adems, en este caso la ausencia de contraccin uterina y las
caracetersticas de la metrorragia apoyan el diagnstico. El tapn mucoso se diferencia claramente de la sangre por su consistencia, asi como el
lquido amnitico hemtico. En cuanto a los Vasa Praevia, es muy infrecuente su rotura espontnea, que siempre se acompaa de afectacin fetal.

PREGUNTA N 83 RESPUESTA: D
En un feto cuya madre est isoinmunizada, el seguimiento supone determinaciones seriadas del test de Coombs indirecto, que cuando presenta
valores ascendentes implica afectacin fetal. El nomograma de Liley valora la presencia de bilirrubina de predominio indirecta, que en este
caso se encuentra en un escaln intermedio. La indicacin es de esperar la madurez fetal y extraer el feto; la madurez viene marcada por la presencia
de fosfatidil glicerol en lquido amnitico, hecho que se ha confirmado. Por ello el siguiente paso es finalizar el embarazo.

[47]
Mg. Dany Colca
RESIDENTADO DE ENFERMERA. Compendio Banco de Preguntas. Lic. Carolina Ayala

PREGUNTA N 84 RESPUESTA: C
El cuadro clnico que se describe se corresponde con una corioamnionitis. El feto est afectado (perfil biofsico = 9), por lo que se debe finalizar la
gestacin lo antes posible, adems de realizar tratamiento antibitico endovenoso a la madre en el parto y postparto. El resto de las opciones se
invalidan al no contemplar esta posiblidad.

PREGUNTA N 85 RESPUESTA: B
Entre el nacimiento de un primer y segundo gemelo deben transcurrir entre 5 y 20 minutos. Por encima de este tiempo debe realizarse la
instrumentacin indicada segn la posicin y el plano que alcanza la ceflica fetal. En este caso podemos finalizar el parto va vaginal ya que
se ha llegado al tercer plano de Hodge, y el frceps es el instrumento ms indicado dada su capacidad rotadora. La maniobra de Kristeller est
proscrita dadas las complicaciones maternas y fetales que puede acarrear.

PREGUNTA N 86 RESPUESTA: D
En el caso de esta paciente debemos prevenir la aparicin de una eclampsia, para lo que el nico medicamento til de esta lista es el sulfato de
magnesio. El resto son hipotensores que se utilizan principalmente para disminuir los valores de la tensin arterial diastlica en las crisis hipertensivas.

PREGUNTA N 87 RESPUESTA: C
En la primiparidad aosa y la toxemia materna es ms frecuente el crecimiento intrauterino retardado de tipo II o extrnseco disarmnico
(neonatos distrficos), en el que la cabeza es mucho mayor que el abdomen por la redistribucin del flujo hacia los tejidos nobles. En este tipo de
CIR se recomienda la extraccin del feto tan pronto como sea maduro, pues parece ser que se desarrolla mejor fuera de la madre; la placenta est
muy deteriorada y no es capaz de suministrar el aporte necesario de oxgeno. El realizar una cesrea inmediata independientemente del estado fetal
sucede nicamente cuando est en peligro la vida de la madre. En el caso que planteamos se aconseja la induccin del parto y su finalizacin va
vaginal de ser posible.

PREGUNTA N 88 RESPUESTA: E
El desarrollo del cuadro con la aparicin de disnea y hemorragia incoercible orienta a una embolia de lquido amnitico, complicada con una
coagulacin intravascualar diseminada (CID). El pronstico es nefasto, prcticamente con un 100% de mortalidad, y se debe al paso de lquido
amnitico al torrente circulatorio materno, lo que desencadena la cascada de la coagulacin intravascular y una embolia pulmonar casi
simultneamente. El diagnstico diferencial con las otras situaciones descritas se basa en la sintomatologa pulmonar, si bien sta puede ser
minusvalorada o incluso ignorada.

PREGUNTA N 89 RESPUESTA: B
El cuadro clnico ms probable en este paciente sera una endocarditis y artritis sptica de rodilla. El germen ms frecuentemente implicado es un
estafilococo aureus, por lo que la cobertura ideal sera vancomicina y tobramicina (la cual es sinrgica con la vancomicina).

PREGUNTA N 90 RESPUESTA: E
Muy probablemente no hallemos ante una neumona por Mycoplasma. Una de las complicaciones relativamente frecuentes es la miringitis ampollosa.
El tratamiento de eleccin es la eritromicina.

PREGUNTA N 91 RESPUESTA: C
El cuadro clnico corresponde a un shock sptico en el que es frecuente encontrar leucopenia, sobre todo si est originado por grmenes
gramnegativos y trombopenia, as como consumo de factores de coagulacin. La hiperglucemia se produce por la descompensacin
infecciosa y estrs. Probablemente est originado por una pseudomona resistente a la ceftacidima.

PREGUNTA N 92 RESPUESTA: D
La descripcin de la lcera hace pensar en un chancro duro de origen sifiltico. El chancro aparece en la primoinfeccin lutica y posteriormente
desaparece. La prueba de eleccin sera una serologa sifiltica.

PREGUNTA N 93 RESPUESTA: A
La rotura espontnea de bazo es una complicacin infrecuente pero caracterstica de la mononucleosis infecciosa. Cursa con un cuadro de
abdomen agudo y shock hipovolmico por sangrado.

PREGUNTA N 94 RESPUESTA: C
La primera causa de bronquiolitis en un lactante es el virus sincitial respiratorio. El tratamiento es de soporte con oxgeno y humedad. En caso de
patologa de base importante, se debe iniciar tratamiento con ribavirina en aerosol.

PREGUNTA N 95 RESPUESTA: C
Una vez descartado proceso vascular trombtico, de forma cierta con la flebografa, el diagnstico diferencial ms probable sera celulitis del miembro.
En la mayora de las ocasiones est producida por estafilococo, por lo que sera conveniente la cobertura emprica con Oxacilina y ver evolucin.

[48]
Mg. Dany Colca
RESIDENTADO DE ENFERMERA. Compendio Banco de Preguntas. Lic. Carolina Ayala

PREGUNTA N 96 RESPUESTA: A
Los abscesos en pacientes ADVP deben ser drenados cuando tienen las condiciones para ello. Esto es, cuando comienzan a fluctuar. El
tratamiento antibitico es inefectivo si no se acompaa de drenaje.

PREGUNTA N 97 RESPUESTA: D
El cuadro clnico ms probable es el de clera por la ausencia de fiebre, dolor abdominal y la gran prdida de lquidos. Estas
caractersticas no se dan en el resto de las entidades.

PREGUNTA N 98 RESPUESTA: E
El aspergiloma es una forma clnica de la aspergilosis. Esta enfermedad sistmica fngica suele aparecer en individuos
inmunodeprimidos que, adems, son tratados con antibiticos de amplio espectro o tratamiento antifngico al cual son resistentes.

PREGUNTA N 99 RESPUESTA: D
El cuadro clnico parece corresponder a la evolucin natural de una tos ferina. El tratamiento de eleccin es la eritromicina.

PREGUNTA N 100 RESPUESTA: A


A falta de confirmar el diagnstico mediante un medulograma, el cuadro clnico parece corresponder a una Leishmaniasis visceral o Kala-azar, la
cual cursa con importante esplenomegalia con pancitopenia secundaria. Caractersticamente presenta fiebre con dos picos diarios. El diagnstico se
hace mediante la visualizacin directa de las leishmanias en la mdula sea. Se pueden utilizar adicionalmente medios de cultivo y serologa
especfica.

[49]

Das könnte Ihnen auch gefallen